ORTHOPEDIC MCQS ONLINE OB 20 RECONSTRUCTION 1A
ORTHOPEDIC MCQS ONLINE OB 20 2A RECONSTRUCTION
OrthoCash 2020
-
A 65-year-old woman with painful knee arthritis and the deformity seen in Figure A, is scheduled to undergo a total knee arthroplasty. All the following are risk factors for a post-operative peroneal palsy EXCEPT:
-
Pre-operative flexion contracture >10 degrees
-
History of lumbar laminectomy
-
Female gender
-
Valgus deformity of >12 degrees
-
Epidural anesthesia
Corrent answer: 3
The clinical presentation is consistent with end-stage arthritis in a valgus knee. All of the factors listed are risk factors for peroneal nerve palsy EXCEPT female gender, which is not a risk factor.
Peroneal nerve palsy is a potential serious complication of TKA in patients with a pre-operative valgus knee deformity. Peroneal nerve palsy is likely caused by lengthening of the lateral aspect of the knee and subsequent traction on the peroneal nerve. It is generally recommended that patients be evaluated
carefully for symptoms postoperatively. If peroneal nerve palsy symptoms are discovered, the knee should be flexed to relax the tension that is effectively being placed on the nerve. If peri-operative nerve exploration or decompression is undertaken, the posterior border of the biceps-femoris tendon is the proper site of identification.
Idusuyi et al. published a retrospective review of 32 postoperative peroneal nerve palsies in thirty patients in which they identified possible risk factors. Prior proximal tibial osteotomy, lumbar laminectomy (thought to be a “double-crush” phenomenon), and preoperative valgus alignment of 12 degrees or more were all identified as risk factors. Other concerns included epidural anesthesia for postop pain control, preoperative flexion contractures and tourniquette time greater than 120 minutes also increased concern.
Favorito et al reviewed valgus total knee arthroplasty and reported that the most common complications of patients with a valgus deformity include: tibiofemoral instability (2% to 70%), recurrent valgus deformity (4% to 38%), postoperative motion deficits requiring manipulation (1% to 20%), wound problems (4% to 13%), patellar stress fracture or osteonecrosis (1% to 12%), patellar tracking problems (2% to 10%), and peroneal nerve palsy (3% to 4%).
Figure A demonstrates and AP radiograph of the knee showing end-stage arthritis with severe lateral compartment narrowing.
Incorrect Answers:
Answer 1: Pre-operative flexion contracture >10 degrees is a risk factor for postoperative peroneal nerve palsy due to stretching the nerve, causing neurologic ischemia.
Answer 2: History of lumbar laminectomy is thought to place patients at risk for postoperative peroneal nerve palsy because of the "double-crush" phenomenon.
Answer 4: Valgus deformity >12 degrees increases the risk for postoperative peroneal nerve palsy due to stretching the nerve beyond functional tolerance postoperatively.
Answer 5: Epidural anesthesia has been found to be significantly associated with post-operative peroneal nerve palsy. Idusuyi et al postulate that the decrease in proprioception and sensory stimuli that accompany epidural anesthesia postoperatively allow the limb to rest in an unprotected state, thus placing the limb at risk for neurologic ischemia from local compression.
OrthoCash 2020
-
-
An 82-year-old woman falls and sustains the fracture shown in figure A. She denies any history of dislocation or prodromal pain prior to her fall. What is the most appropriate treatment?
-
Toe-touch weightbearing
-
Open reduction internal fixation with a cable plate
-
Revision of the femur with a long, cementless stem
-
Revision of the femur with a long, cemented stem
-
Girdlestone resection arthroplasty Corrent answer: 3
The radiograph demonstrates a periprosthetic femur fracture extending to the tip of the stem. The long spiral fracture is consistent with a loose implant. The bone stock is sufficient. Therefore, this fracture pattern would classify as a B2 using the Vancouver classification system. The Vancouver classification for periprosthetic femoral fractures is simple yet incorporates all the pertinent factors such a location, stem fixation, and bone stock. Type A is a trochanteric fracture- lesser or greater. These can be treated non-operatively usually and ORIF if symptomatic. Type B fractures are around or just below the stem and are subdivided into three types. Type B1 is a fracture with a well fixed stem.
The treatment is cable plating or allograft struts or a combination of the two. Type B2 is a fracture with a loose stem with good bone stock. The treatment is a cementless porous coated long stem atleast two diameter length past the
fracture site. Type B3 is a fracture with a loose stem and comminution. For younger patients, use cementless porous coated long stems with allograft struts. For older patients, consider a tumor prosthesis. Cement fixation is sometimes necessary Type C is a fracture well below the stem tip. These can be treated independently of the prosthesis.
Springer et al showed optimal outcomes with revision involving long extensively-coated femoral stems for Vancouver B fractures.
Masri et al review the classification and treatment of periprosthetic femur fractures.
OrthoCash 2020
-
-
A 67 year-old woman sustained an ACL tear while playing basketball when she was 35 years-old. She has noted progressive leg deformity and episodes of giving way, and now has pain preventing activity. Non-operative management has failed to provide relief. Treatment should consist of?
-
Opening wedge high tibial osteotomy with autograft
-
Closing wedge proximal tibial osteotomy
-
Medial interpositional arthroplasty
-
Medial unicompartmental knee arthroplasty
-
Total knee arthroplasty Corrent answer: 5
The radiograph seen in Figure A reveals varus alignment of the knee, with medial tibial deficiency; from this X-ray the patient appears to have unicompartmental arthritis. Treatment options for unicompartmental arthritis include high tibial osteotomy, interpositional arthroplasty, unicondylar knee replacement and total knee replacement. Interpositional arthroplasty became popular in the 1950’s when early outcomes analysis seemed to indicate good results; long term follow up in one study found 0/12 excellent results, with all patients requiring conversion to TKA. This procedure is no longer recommended due to the poor long term outcomes.
While an osteotomy is still used for young and active patients, unicompartmental or total knee arthroplasty have largely replaced this treatment in older patients. Advantages of UKA and TKA include more predictable relief of pain, quicker recovery, and better long-term results. Criteria for UKA include limited unicompartmental disease, no more than a fixed 10 degrees of varus or 5 degrees of valgus deformity from neutral and an intact anterior cruciate ligament with no signs of medial lateral subluxation of the femur on the tibia; this patient is therefore not a good candidate for this procedure.
Total knee arthroplasty can be used to provide predictable pain relief in a patient with unicompartmental and tricompartmental degenerative disease and varus malformation of the knee and for this patient is the best option.
OrthoCash 2020
-
-
A 65-year old healthy male has just undergone primary total knee arthroplasty. Which of the following is associated with use of a closed suction drain in this procedure?
-
Increased incidence of wound dehiscence
-
Increased incidence of transfusion
-
Decreased incidence of infection
-
Decreased incidence of hematoma formation requiring return to OR
-
Decreased pain scores on post-op days 1 and 2 Corrent answer: 2
The cited meta-analysis by Parker et al evaluated 18 studies with 3495 patients (3689 wounds) and demonstrated that closed suction drainage increases the transfusion requirements after elective hip and knee arthroplasty (relative risk, 1.43; 95% confidence interval, 1.19 to 1.72). They found no significant effect on wound hematoma, infection, or operations for wound complications.
OrthoCash 2020
-
-
A 75-year-old man underwent total hip arthroplasty 10 years ago. He now reports mild groin pain which has been increasing lately. What is the most likely explanation for the finding in Figure A indicated with the arrows?
-
Osteosarcoma
-
Galvanic corrosion of the modular components
-
Polyethlene wear particles tracking through the effective joint space
-
Joint sepsis
-
Occult fracture
Corrent answer: 3
Osteolysis of the pelvis is a common complication associated with total hip arthroplasty. Osteolysis affects sockets with and without cement, and has been attributed to the biologic reaction to wear debris. With well-fixed cementless sockets, an expansile pattern of osteolysis is usually seen.
The radiographic appearance has a radiolucent area that starts at the implant-bone interface and expands into the cancellous bone away from the implant.
This pattern of osteolysis can be explained with the concept of effective joint space. This concept states that joint fluid and wear particles will flow according to pressure gradients and follow the path of least resistance.
The Level 5 review article by Chiang discusses osteolysis in further depth.
OrthoCash 2020
-
-
All of the following are risk factors for post-operative total knee arthroplasty periprosthetic supracondylar femur fractures EXCEPT:
-
Rheumatoid arthritis
-
Parkinson's disease
-
Chronic steroid therapy
-
Revision knee arthroplasty
-
Male gender
Corrent answer: 5
Rheumatoid arthritis, Parkinson's disease, chronic steroid therapy, osteopenia, and female gender have all been found to be risk factors for postoperative periprosthetic supracondylar femur fractures. Male gender has not been found to be a risk factor.
Su et al discuss risk factors for supracondylar periprosthetic femoral fractures which include rheumatoid arthritis, neurologic disorders such as Parkinson's disease, chronic steroid therapy, and revision knee arthroplasty. Analysis of the Mayo Clinic joint registry by Berry found that females are at increased risk of postoperative periprosthetic fracture, likely due to the increased incidence of osteoporosis. There is controversy regarding anterior cortical notching (Illustration A) and increased risk for periprosthetic fracture.
Lesh et al performed a biomechanical study on the consequences of anterior femoral notching. Using cadaveric matched femora with and without full thickness anterior cortex defects above TKA implants, they found that notching decreased both bending and torsional strength in the supracondylar region of the femur. They also found that fracture orientation differed between the two groups following the application of a bending load.
Ritter et al in a series of 670 total knee arthroplasties, of which 27% had notching (<3 mm), only 2 patients sustained supracondylar femoral fractures (1 with notching, 1 without). The authors argued that remodeling and stress redistribution around the implant accounted for the low incidence of fractures despite a high incidence of intraoperative notching.
Illustration B shows the typical fracture pattern seen with notching, and Illustration C shows the fracture pattern without.
OrthoCash 2020
-
-
A 64-year-old woman with osteoarthritis underwent bilateral total knee replacement 3 years ago. Current radiographs are shown in Figure A. She reports a 3-month history of bilateral knee pain while at rest and increasing swelling in the knees. Her ESR and CRP are elevated and bilateral knee aspiration cultures reveal Staphylococcus aureus. What is the most likely outcome if the patient undergoes simultaneous, bilateral knee resection arthroplasty with cement spacer and a course of intravenous antibiotics?
-
Prosthesis reimplantation with need for multiple surgical debridements at 2-year follow-up
-
20% risk of above knee amputation
-
Retention of antibiotic cement spacer and low chance of successful prosthesis reimplantation at 2-year follow-up
-
50% rate of conversion to knee fusion following resection arthroplasty
-
Successful prosthesis reimplantation at 2-year follow-up with less than 20% revision rate
Corrent answer: 5
This patient presents with bilateral total knee arthroplasty infection.
Wolff et al report Level 4 evidence of 18 patients followed an average of 5 years after bilateral TKA infection. Eleven patients were initially treated with attempts to salvage the original prosthesis (polyethylene l liner exchange, I&D, IV antibiotics and chronic oral suppressive antibiotics. With prosthesis retention, 9/11 (81%) developed recurrent infection at a mean of 15 months. The other 10 patients initially underwent resection arthroplasty with cement spacer and a course of IV antibiotics. Seven of the 10 (70%) underwent reimplantation at a mean of 3 months (6 weeks to 5 months) and none of the patients required revision at mean of two years follow up. Satisfaction rates were significantly higher among this group of patients. The authors advocate the protocol of bilateral TKA resection arthroplasty with cement antibiotic spacer and course of IV antibiotics followed by prosthesis reimplantation.
OrthoCash 2020
-
-
During insertion of a cementless femoral stem, a nondisplaced fracture is noticed along the femoral calcar. Which of the following is the most appropriate next step in surgical management?
-
Continued insertion of the stem, cerclage wiring around the fracture site, and non-weight bearing x6 weeks
-
Continued insertion of the stem, reduction of the hip, and non-weight bearing activity restrictions following surgery
-
Removal of the stem, cerclage wiring around the fracture site, and re-insertion of a stem
-
Removal of the stem and conversion to a cemented femoral stem
-
Removal of the stem, open reduction internal fixation of the femur with planned delayed femoral stem insertion following fracture healing
Corrent answer: 3
Appropriate care of an intraoperative fracture during total hip arthroplasty requires removal of the stem to adequately evaluate the fracture. The fracture should then be stabilized with cerclage wiring, and a long stem should be inserted to ensure stability of the stem in the postoperative period.
Tsiridis et al review the identification, classification, and management of intraoperative and postoperative periprosthetic hip fractures. Postoperative fractures around stable components may be treated with open surgical fixation. All intra-operative fractures should be considered inherently unstable, and should be treated with a long stem that bypasses the femoral fracture as well as cerclage wiring.
Incorrect Answers:
Answer 1: If there is a fracture while inserting the final femoral stem, it should be removed, a cerclage wire should be placed, then the final stem should be inserted.
Answer 2: The fracture creates an unstable situation with the femoral stem, and this should be stabilized intraoperatively to prevent settling, continued pain, and possible instability.
Answer 4: Simple conversion to a cemented stem with a proximal fracture, without cerclage placement, will lead to a loss of hoop stresses as the fracture can continue to displace during pressurization.
Answer 5: There is no need to delay femoral implant insertion to a second stage.
OrthoCash 2020
-
-
A 72-year-old male presents 2 years status post fixation of an impending pathologic right femur fracture due to metastatic renal cell carcinoma. He is minimally ambulatory due to pain. Despite radiation therapy, there has been progression of the lesion with extensive cortical bone loss, which is shown in Figure A. A proximal femoral replacement arthroplasty is performed without complications, and is demonstrated in Figure B. Which of the following is true regarding this patients post-operative course?
-
Deep prosthetic infection is the most common complication
-
Mean Harris Hip score will likely not improve
-
The patient will most likely continue to be minimally ambulatory
-
Aseptic failure rate at 5 years is >50%
-
Pre-operative radiation decreases the risk of infection post-operatively Corrent answer: 1
Deep prosthetic infection is the most common complication after hip arthroplasty performed for salvage of failed internal fixation after pathologic proximal femoral fracture secondary to malignancy.
Jacofsky et al reviewed the complications in 42 patients with a mean age of 63 who were treated with hip arthroplasty for salvage of failed treatment of a pathologic proximal femoral fracture. Multiple different constructs were used.
The most common complication was deep prosthetic infection, which occurred in nearly 10% of the patients studied. All infections occured in patients whom had previously received radiation. The mean Harris Hip score improved from 42 to 83 points post-operatively, and 41 of the 42 patients were ambulatory at follow-up. Implant survivorship free of revision for any reason at 5 years was 90%, and free of revision for aseptic failure or radiographic failure was 97%.
Figure A shows a lytic lesion of the proximal femur with an intramedullary implant. Figure B shows a proximal femoral replacement.
OrthoCash 2020
-
-
All of the following are true for a patient who underwent a metal-on-metal total hip arthroplasty (THA) EXCEPT?
-
they will have production of ionically charged wear particles
-
there is a higher cancer risk than with metal-on-polyethylene THA
-
they will have elevated levels of cobalt and chromium in the serum
-
they will have elevated levels of cobalt and chromium in the urine
-
there is a higher frictional torque than with ceramic on ceramic THA Corrent answer: 2
Metal-on-metal articulations in THA are characterized by ionically charged wear particles. Elevated serum and urine concentrations of metallic elements including chromium, cobalt, and molybdenum are found in patients with metal-on-metal joint replacements as compared with controls. To date, there is no correlation between metal serum levels and cancer risk. As such, the link between metal on metal arthroplasty and an elevated cancer risk has not been supported by hard data. Finally, metal-on-metal THA has higher frictional torque than ceramic on ceramic THA.
The reference by Brockett et al is a biomechanical analysis of the friction of various hip arthroplasty components. Ceramic on ceramic was found to have the lowest coefficient of friction, followed by ceramic on metal.
OrthoCash 2020
-
-
A 62-year-old woman is undergoing a revision total knee arthroplasty for aseptic component loosening. The surgeon has all the trial components in place and recognizes that the soft tissues are balanced in the coronal plane, but the knee is 10 degrees from reaching full extension. He proceeds to correct the contracture by
making an additional 2mm cut off of the tibia and is successful in achieving full extension. What is the most likely effect of this additional resection?
-
Loss of full flexion
-
Flexion instability
-
Extension instability
-
Valgus instability
-
Varus instability
Corrent answer: 2
This patient presents with asymmetric gapping because she is tight in extension and balanced in flexion. Ries discusses that resection of the proximal tibia in this situation is a common pitfall in surgical technique as it “will resolve the flexion contracture but produce instability in flexion”. The preferred method of restoring the distal femoral joint line to achieve full extension and maintain flexion stability is to cut “more of the distal part of the femur, as this will not affect the flexion space”. Similarly, there is an asymmetric gap if full extension is achieved, but flexion is limited. The lack of full flexion can be treated with distal femoral augments and a thinner tibial insert.
OrthoCash 2020
-
-
A 67-year-old diabetic male presents 4 months status post right total knee arthroplasty (TKA) complaining of pain and stiffness for the last four weeks. A clinical photograph is shown in Figure A. Radiographs and a bone scan are shown in Figures B, C and D. Blood work shows an ESR of 14mm/hr (normal 0-12mm/hr) and a CRP of 2mg/L (normal 1-3mg/L). Knee aspiration yields WBC of 1000, 30% PMNs, and a negative gram stain. He finished a 14-day course of antibiotics prescribed to him by his primary care physician one week ago. Which of the following is the most appropriate next step in management?
-
Broad-spectrum, empiric oral antibiotics
-
Repeat aspiration after one week
-
Irrigation and debridement of the right knee with a polyethylene liner exchange
-
One-stage irrigation and debridement of the right knee with a component exchange
-
Two-stage component removal, antibiotic spacer placement and subsequent revision
Corrent answer: 2
The clinical scenario describes a patient with an equivocal presentation of a periprosthetic joint infection (PJI) and recent history of antibiotic use. As such, a repeat aspiration in one week is indicated.
The work-up of a suspected PJI after TKA includes an evaluation of radiological (x-ray +/- bone scan and PET scan) and laboratory (ESR and CRP) parameters as well as analysis of joint aspirate fluid (cell count and differential, culture, gram stain +/- PCR).
Barrack et al. evaluated the utility of routine aspiration of a symptomatic TKA before reoperation and found aspiration to have a sensitivity of 75%, specificity of 96%, and accuracy of 90%. Previous antibiotic use increased the
risk of a false negative result, and reaspiration at a later date was found to significantly improve the value of this test in such cases.
Parvizi et al. published an AAOS Clinical Practice Guideline (CPG) on the diagnosis of PJI of the hip and knee using evidence from the literature. They found sufficient evidence to make strong recommendations for the use of ESR, CRP, joint aspiration, intraoperative gram stain, frozen sections of peri-implant tissues, multiple intraoperative cultures and withholding antibiotics until after cultures have been obtained.
The Workgroup Convened by the Musculoskeletal Infection Society proposed diagnostic criteria for PJI after the evaluating the available evidence and suggested that a definite PJI exists when: (1) there is a sinus tract communicating with the prosthesis; or (2) a pathogen is isolated by culture from 2 or more separate tissue or fluid samples obtained from the affected prosthetic joint; or (3) when 4 of the following 6 criteria exist: (a) elevated serum erythrocyte sedimentation rate and serum C-reactive protein (CRP) concentration, (b) elevated synovial white blood cell count, (c) elevated synovial polymorphonuclear percentage (PMN%), (d) presence of purulence in the affected joint, (e) isolation of a microorganism in one culture of periprosthetic tissue or fluid, or (f) greater than 5 neutrophils per high-power field in 5 high-power fields observed from histologic analysis of periprosthetic tissue at ×400 magnification.
Figure A is a clinical photograph demonstrating a swollen, erythematous right knee with a well-healed incision from a previous TKA. Figure B and C are AP and lateral radiographs of the right knee with no obvious acute findings. Figure D is a bone scan demonstrating increased uptake in the post-operative knee, which is consistent with the 4 month follow-up.
Incorrect Answers:
Answer 1, 3, 4 & 5: Broad-spectrum antibiotics, I & D +/- liner exchange, one stage and two stage revision would not be appropriate at this time point as the diagnosis remains unclear.
OrthoCash 2020
-
-
Internal rotation of the femoral component during total knee arthroplasty can result in which of the following?
-
Increased need for lateral release
-
Decreased post-operative pain
-
Increased polyethylene thickness
-
Decreased post-operative Q angle
-
Elevation of the native joint line Corrent answer: 1
Internal rotation of the femoral component during total knee arthroplasty causes increased lateral patellar subluxation forces, which effectively increases the Q angle. Femoral component rotation, in isolation, does not affect the position of the joint line or dictate the necessary polyethylene thickness.
Internal rotation of the femoral component can be a source of increased pain post-operatively. Sodha et al compared the rates and results of lateral release before and after femoral component placement. The rates of lateral release in internally rotated femoral components was 24% for varus deformities and 33% for valgus deformities. When the femoral component was externally rotated, based off the transepicondylar axis in 246 TKA's, lateral release rates of 7% in varus deformities and 29% in valgus deformities were noted.
Illustration A demonstrates internal rotation of the femoral component, and increased lateral patellar subluxation.
OrthoCash 2020
-
-
The schematic shown in Figure A displays a ceramic-on-ceramic total hip arthroplasty articulation with impingement. Which of the
following modifications would increase the primary arc range of motion?
-
Addition of a collar on the femoral head
-
Exchanging the ceramic liner with a hooded polyethylene liner
-
Increasing the femoral head size
-
Increasing the femoral offset
-
Increasing the acetabular anteversion Corrent answer: 3
The assessment of hip stability involves four major areas: component design, component alignment, soft tissue tensioning, and soft tissue function. The primary determinant of primary arc range is the head-neck ratio, which is defined as the ratio of the femoral head diameter to the femoral neck diameter. Increasing the size of the femoral head will increase the excursion distance of the femoral head to dislocate, thus making the hip more stable.
Illustration A shows how a greater head-to-neck ratio may improve range of motion before impingement. Increasing femoral component offset increases the abductor moment arm and reduces the resulting hip joint reactive force but does not affect primary arc range of motion impingement.
The article by Yoon et al reports that ceramic-on-ceramic constructs are susceptible to osteolysis resulting from particulate debris. The histologic reaction to the smaller ceramic particles was similar as the reaction to larger particles such as polyethylene. The debris in the listed study was found to be largely from the articulation and was also thought to be secondary to a decreased head-neck ratio leading to impingement.
OrthoCash 2020
-
-
A patient who has previously undergone a high tibial osteotomy 10 years prior is scheduled for a total knee arthroplasty (TKA). Which of the following factors is most likely to be present and may complicate the arthroplasty?
-
Collateral ligament instability
-
Patella alta
-
Patella baja
-
Patellar tendon insufficiency
-
Severe varus deformity Corrent answer: 3
TKA after a high tibial osteotomy (HTO) can be more difficult to perform than a primary knee replacement because of a shift of the proximal tibial articular surface in relation to the medullary canal, retained hardware, previous skin incisions, scar tissue, and altered patellofemoral mechanics caused by patella baja and contraction of the patella tendon. The frequency of valgus deformity is greater following HTO.
Parvizi et al reviewed 166 TKA's performed following a high tibial osteotomy. A higher rate of component loosening was observed with 8% revision at 5.9 years follow-up. Male gender, preoperative limb malalignment, young age, and collateral ligament instability were associated with higher rates of failure.
Meding et al reviewed 39 patients who had bilateral TKA performed following unilateral high tibial osteotomy. There were no differences between the two
groups including postoperative complications, range of motion, revision surgery, and patient satisfaction scores.
OrthoCash 2020
-
-
Osteopenia has what effect on the strength of the bone-cement interface in comparison to normal bone?
-
no effect
-
improved mechanical integrity (higher fracture resistance)
-
diminished mechanical integrity (low fracture resistance)
-
reduced depth of cement penetration into bone
-
less affected by cement pressurization Corrent answer: 2
The increased porosity seen in osteopenia and osteoporosis actually helps create a stronger bone-cement interface. Graham et al studied the effects of bone porosity, trabecular orientation, cement pressure, and cement penetration depth on fracture toughness at the bone-cement interface in bovine femora. They found that improved mechanical integrity (higher fracture resistance) is correlated with increased bone porosity (worsening osteopenia) and maximum cement penetration depth. The authors also found that with increased cement pressurization, the cement penetration depth was increased and the fracture resistance was also increased. In conclusion, "a lack of porosity is associated with reduced mechanical integrity of the cemented interface and may contribute to the relatively poorer results of cement fixation in young male patients." The fracture resistance of the bone-cement interface is greatly improved when the ability of the cement to flow into the intertrabecular spaces is enhanced."
OrthoCash 2020
-
-
Figure A demonstrates a total knee prosthesis design. Which of the following motions is constrained in this particular design:
-
Complete anterior-posterior translation constraint only
-
Partial varus-valgus angulation constraint only
-
Partial varus-valgus angulation and partial internal-external rotation constraint
-
Complete internal-external rotation constraint only
-
Complete varus-valgus angulation and anterior-posterior translation constraint
Corrent answer: 3
Figure A demonstrates a non-linked, constrained total knee arthroplasty prosthesis. This drawing depicts the degree of coronal plane and rotational constraint provided by the tall, wide tibial spine in the deep femoral box. This design constrains varus-valgus (allows 2°-3°) and internal-external rotation (allows 2°). A linked, rotating-hinge prosthesis (Illustration A) constrains anterior-posterior translation in addition to varus-valgus and internal-external rotation.
The article by Scuderi reports that in revision TKA, the goal is to restore the original anatomy, restore function, and provide a stable joint. To this point of stability, it is preferable to implant the prosthesis that provides adequate stability with the least mechanical constraint possible to avoid bone-implant stresses that may cause early loosening. Therefore, it is preferable to use a posterior-stabilized (cruciate substituting) articulation (Illustration B) if the knee remains stable without constrained components.
McAuley et al suggest that more predictable results are obtained with the use of cruciate-substituting components. However, if there is functional loss of the medial collateral ligament or lateral collateral ligament, inability to balance the flexion and extension spaces, or a severe valgus deformity, then a constrained
condylar prosthesis is needed.
Rodriguez et al reports Level 4 evidence of 44 patients revised with varus-valgus constrained implants followed for an average of 5.5 years. There was a theoretical concern that the increased constraint of the prosthesis would lead to component loosening, however their series had only one femoral component and no tibial components that loosened.
OrthoCash 2020
-
-
A surgeon is planning to revise a left hip resurfacing component to a total hip arthroplasty. He wishes to decrease the joint reaction force of the left hip by increasing the femoral offset. Which of the following labeled measurements found in Figure A best describes femoral offset?
-
Line 1
-
Line 2
-
Line 4
-
Line 5
-
Line A
Corrent answer: 3
In total hip arthroplasty, the femoral component offset is measured as distance between the center of the femoral head and a line drawn down the center of the femoral shaft(Line 4 shown in Figure A). Increased femoral offset is also shown in Illustration A.
The review article by Bourne et al states that offset is relevant to soft tissue balancing around the hip and the forces generated at the hip joint.
Lateralization of the femoral shaft restores offset, reduces femoropelvic impingement, and increases abductor muscle tension leading to a decreased joint reaction force. However, increasing femoral offset may have the unwanted effect of increasing rotational torque on the stem leading to aseptic loosening and increasing trochanteric bursitis.
OrthoCash 2020
-
-
A 62-year-old woman presents for her 1-year follow-up after a revision right total hip arthroplasty. She has no complaints of pain and has returned to all her activities of daily living. An AP radiograph is shown in Figure A. The black arrow in the radiograph indicates she is at higher risk for which of the following?
-
Aseptic loosening
-
Aseptic lymphocytic vasculitis-associated lesions (ALVAL)
-
Dislocation
-
Third body wear
-
Catastrophic ceramic bearing failure Corrent answer: 3
The radiographs reveal a constrained system by the metal ring of the constrained liner, and subsequent broken ring representing a dissociation of the liner. Ring failure is associated with increased risk of hip dislocation. The incidence of dislocation ranges from 0.5% to 10% after primary and up to 28% after revision THA. Procedures described to treat this instability include reorientation of femoral or acetabular component position, trochanteric reattachment or advancement, capsulorrhaphy, the use of an elevated acetabular liner, conversion to a bipolar prosthesis, lengthening of the femoral neck, resection arthroplasty, or the use of a constrained acetabular component.
In the Level 4 study by Shapiro et al, 85 constrained THAs were implanted during revision THA for chronic instability. There was a 2.4% dislocation rate in this cohort and both of these were secondary to constrained liner dissociation. Illustration A shows a radiograph of a constrained hip dislocation secondary to
fracture of the constraining ring on the neck of the liner. An example of a broken constraining ring is shown in Illustration B.
OrthoCash 2020
-
-
Which of the following characteristics of stromelysin is incorrect?
-
Belongs to the family of proteolytic enzymes called metalloproteinases
-
Secreted by chondrocytes
-
Inhibited by Tissue Inhibitor of Metalloproteinase
-
Inhibited by plasmin
-
Degrades cartilage and is thought to play a role in degenerative joint disease
Corrent answer: 4
Stromelysin is not inhibited by plasmin.
Metalloproteinases (MMPs) are a family of proteolytic enzymes which utilizes a metal during the catalytic process. Stromelysin and plasmin are two examples of metalloproteinases, both secreted by chondrocytes, which have degradative action against cartilage. It is believed that these metalloproteinases play a role in articular degeneration and degenerative joint disease. Tissue inhibitor of metalloproteinase inhibits the degradative action of stromelysin. Tissue inhibitor of metalloproteinase (TIMPs) counteract the proteolytic enzymes produced by chondrocytes.
Tetlow et al performed an experiment on the superficial zone of cartilage in in osteoarthritis specimens. They found cells that immunostain for IL-1beta, TNFalpha, and 6 different MMP's which support the concept that cytokine-MMP associations reflect a modified chondrocyte phenotype and an intrinsic process of cartilage degradation in OA.
OrthoCash 2020
-
-
What preoperative knee deformity puts a patient at most risk for a postoperative peroneal nerve palsy after total knee arthroplasty?
-
Valgus deformity only
-
Valgus and flexion contracture
-
Varus and flexion contracture
-
Varus deformity only
-
Flexion contracture only Corrent answer: 2
Conditions that have been associated with an increased prevalence of peroneal nerve injury include a significant fixed valgus deformity and flexion contracture. Immediate treatment of a peroneal nerve palsy post-operatively includes dressing removal and flexion of the knee 20-30 degrees.
Ayers et al report a 0.58% cumulative prevalence of peroneal nerve palsy
after TKA in their review article. They state that possible mechanisms of nerve injury include traction during correction of a valgus deformity, ischemia when stretching of the surrounding soft tissue causing occlusion of small vessels, and compression by a tight dressing or splint.
In a more recent review article, Nercessian et al report a peroneal nerve palsy incidence of 0.3-1.3% after primary total knee arthroplasty. Their reviewed studies reported a preoperative valgus deformity of 18-23.3 degrees, and flexion deformity of 15.5-22 degrees as being risk factors for peroneal nerve palsy after TKA.
OrthoCash 2020
-
-
Osteolysis occurs because there is a histiocytic response by macrophages to wear debris. What size particles are implicated in osteolysis?
-
less than 1 micron (submicron)
-
approximately 10 microns
-
approximately 100 microns
-
approximately 1000 microns
-
approximately 5000 microns
Corrent answer: 1
Osteolysis is the histiocytic response by macrophages to wear debris particles, which are often less than 1 micron in size.
Osteolysis is a particle-induced biological process occurring at the bone-metal or bone-cement interface around total joints resulting in rapidly expanding focal lesions that may or may not cause loosening. Its slower counterpart, aseptic loosening, involves the identical biological process. Wear particles generated within the joint space are phagocytosed and stored within cells in the joint capsule. Sub-micron particles are retained within macrophages and are implicated in osteolysis.
Campbell et al. described an isolation method to recover ultra-high-molecular-weight polyethylene (UHMWPE) particles from tissues around failed total hip replacements. This process yielded particles that had rounded or elongated shapes. Additionally, the majority of particles isolated were reported to be submicron in size.
Mckellop reviews four topics in wear including Modes, Mechanisms, Damage and Debris. Four Modes that creates debris are described. Wear Mode 1 occurs
when the two bearing surfaces are articulating against each other in the manner intended by the implant designer. Mode 2 occurs when a bearing surface articulates against a non-bearing surface. Mode 3 occurs when third-body abrasive particles have become entrapped between the two bearing surfaces, and Mode 4 occurs when two non-bearing surfaces are wearing against each other.
Incorrect Answers:
As reported by Campbell et al. the majority of particles recovered from prosthetic joints with osteolysis were submicron in size. Answer choices 2, 3, & 4 contain values greater than a micron and are therefore incorrect.
OrthoCash 2020
-
-
A 62-year-old female underwent a primary total knee arthroplasty of the left knee 10 days ago. She presents to clinic with skin necrosis of the midline incision. There is no deep infection present upon aspiration of the knee joint. She undergoes superficial irrigation and debridement and is left with exposed patellar tendon as shown in Figure A. What is the most appropriate next step in management?
-
Split thickness skin grafting
-
Twice daily wet-to-dry dressing changes with Dakin's solution until healing by secondary intention
-
Latissimus dorsi free flap transfer
-
Vacuum-assisted closure device until healing by secondary intention
-
Medial gastrocnemius muscle flap transfer and skin grafting Corrent answer: 5
Medial gastrocnemius muscle flap transfer and skin grafting is the most appropriate choice of the options listed (postoperative image shown in Illustration A).
Level 4 evidence by Ries describes 9 patients sustained skin necrosis after total knee arthroplasty. Seven of these cases were over the patella tendon or tibial tubercle, of whom 6 were treated with medial gastrocnemius flap coverage. Successful wound healing and salvage of the TKA was achieved in all cases. Ries concluded that necrosis of the proximal wound including the area over the patella can be treated by local wound care and skin grafting.
However, skin necrosis over the tibial tubercle or patellar tendon requires muscle flap coverage to prevent extensor mechanism disruption and deep infection.
OrthoCash 2020
-
-
A 58-year-old man has significant pain and stiffness after undergoing right total knee arthoplasty 6 months ago. A current radiograph and bone scan are shown in Figures A and B. Labs show an ESR of 45mm/hr (normal 0-20) and a CRP of 13.5 mg/l(normal <10). Knee aspiration reveals a WBC count of 850 cells/mm(3) with 70% polymorphonuclear cells and no growth on culture. What is the next most appropriate step in management?
-
Two-stage component removal, antibiotic spacer placement and subsequent revision
-
Observation with repeat ESR and CRP in one week
-
Surgical debridement and polyethylene exchange only
-
Repeat aspiration and culture
-
One-stage irrigation and debridement with exchange of all components Corrent answer: 4
The clinical scenario describes a patient with an equivocal presentation of an infected total knee. The radiographs are normal and the bone scan shows uptake as would be expected 6 months out. A repeat aspiration is indicated in cases of equivocal laboratory aspiration data.
Mason et al in 2003 reviewed 440 revision TKA's of which 86 had preoperative aspirations. The aspirations yield 55 aseptic failures and 31 septic failures. The mean WBC of the aseptic group was 645 cells/mm(3) compared to 25,951 cells/mm(3) for the septic group (P=<.001). The mean percentage of polymorphonuclear cells (PMNs) was statistically higher in the septic group compared with the aseptic group (72.8% vs 27.3%; P=<.001). With these results, the authors concluded that aspirates with a WBC count greater than 2,500 and 60% PMNs are highly suggestive of infection.
Ghanem et al presents Level 3 evidence of 161 infected TKA's compared to 268 aseptic failures and discovered synovial fluid aspiration with WBC of
>1100 cells/mm3 and PMN > 64% are suggestive of infection. When both tests were below these respective values, the negative predictive value was 98.2%.
OrthoCash 2020
-
-
Figures A and B are pre-operative and intra-operative radiographs of a 67-year-old male that has undergone a left total hip arthroplasty under general anesthesia. The patient had no motor deficits preoperatively. During the operation, the trial acetabular and femoral components were positioned and reduced with no complication. Intraoperative leg lengths were equal. Before implanting the real components, the surgeon and anaesthesiologist performed a wake up test, which revealed that the patient was unable to dorsiflex the left foot. What would be the most appropriate next step in the management of this patient?
-
Urgent electromyogram and nerve conduction study
-
Continue with sized trial components and observe the motor function in surgical recovery area
-
Remove all implants and insertion of cement spacer
-
Perform a shortening subtrochanteric osteotomy
-
Urgent neurology consult Corrent answer: 4
This patient has undergone a left THA with significant leg lengthening. The biggest concern is stretch to the sciatic nerve. The most appropriate step at this stage would be to perform a subtrochanteric osteotomy to decrease leg length and sciatic nerve stretch.
Patients with DDH that have undergone a large limb-lengthening procedure are at a greater risk due to the significant stretch of the sciatic nerve. Intraoperative procedures that have been shown to prevent this outcome include good pre-operative planning, limb lengthening <3 cm, subtrochanteric osteotomy, intra-operative wake-up test, neurophysiologic intraoperative monitoring and downsizing implant components if presenting with deficits peri-operatively.
Chen et al. retrospectively reviewed 22 THA procedures in 20 patients with Crowe type IV developmental dysplasia of the hip who underwent the wakeup test during THA. None of the patients who showed no deficits in motion of the feet during the intraoperative wake-up test had signs of postoperative nerve injury.
Krych et al. retrospectively reviewed 24 patients with Crowe type-IV developmental dysplasia of the hip that underwent arthroplasty with a subtrochanteric shortening osteotomy. There were no neurological deficits with this procedure. Two subtrochanteric osteotomies went onto nonunion. There was one instance of isolated loosening of the femoral stem. and ionacetabular component loosened. Four hips dislocated postoperatively.
Figure A and B show pre- and peri-operative images of a patient with severe hip dysplasia treated with primary total hip arthroplasty.
illustration A shows one technique that can be used to perform a subtrochanteric osteotomy.
Incorrect Answers:
Answer 1: Urgent electromyogram and nerve conduction study would not be appropriate intra-operatively.
Answer 2: Motor deficits due to limb lengthening procedures should not be observed when there is a potential to decrease the stretch on the nerve.
Answer 3: There is no indication to remove all implants and insert a cement spacer.
Answer 5: There is no indication to consult neurology.
OrthoCash 2020
-
-
A 45-year-old woman is scheduled to undergo a TKA. Which of the following implant designs theoretically reduces poylethylene wear and reduces bone-implant-interface stress?
-
Mobile-bearing TKA
-
Posterior stabilized fixed bearing TKA
-
Cruciate retaining fixed bearing TKA
-
Constrained TKA
-
Mobile-bearing hinged TKA Corrent answer: 1
Total knee arthroplasties continue to be performed in patients who are younger and more active. As a result of this trend, better wear performance is imperative for long-term durability. Research continues to be done to determine optimal wear characteristics of different polyethylene and metal surfaces. Mobile-bearing knee systems are distinguished from conventional, fixed-bearing systems in that they allow dual-surface articulation between an ultra-high molecular weight polyethylene insert and metallic femoral and tibial tray components. This results in increased sagittal femorotibial conformity of most mobile-bearing implants, which reduces polyethylene shear stresses and should lessen polyethylene wear rates. By allowing more contact area, the surface and subsurface stresses in the poly bearing are significantly reduced (recall that pressure = force / area).
The ICL by Callaghan et al review the early findings of studies of fixed versus mobile bearing implants. Research is progressing as long term data continues to be collected and analyzed. Despite theoretical advantages, there has been no documented improvement in survivorship between mobile and fixed bearing TKA's in short and intermediate term studies.
OrthoCash 2020
-
-
Which of the following intra-operative techniques during total knee arthroplasty (TKA) decreases the need for lateral retinacular release?
-
Internal rotation of femoral component
-
External rotation of femoral component
-
Internal rotation of tibial component
-
Lateralization of patellar component
-
Insertion of a posterior cruciate retaining device Corrent answer: 2
The only answer choice above that decreases the need for a lateral release during TKA is external rotation of the femoral component. Internal rotation of the femoral component increases lateral subluxation forces on the patella, and will increase the need for a lateral release.
Akagi et al looked at the relationship of femoral component rotation on lateral releases on 44 consecutive patients undergoing TKA. Twenty-two patients had femoral component set parallel to the posterior condylar axis, while twenty-two patients had femoral components set at 3-5 degrees of external rotation to the posterior condylar axis. Only 6% of patients in the externally rotated group required a lateral release, vs 33% of the neutrally aligned group.
Parker et al showed that extensor mechanism failure is the most common reason for revision TKA. They discuss the morbidity of patellar tracking which can be caused by either internal rotation of the femoral or tibial component. Furthermore, they recommend intra-operative assessment of patellar tracking with both trial and final implants. If maltracking is present in the presence of an inflated thigh tourniquet, they recommend tourniquet deflation before lateral release as this can alter patellar maltracking. A lateral release should only be considered if lateral tilt or maltracking continues in the presence of properly aligned femoral and tibial components.
OrthoCash 2020
-
-
When placing acetabular screws to supplement cementless acetabular fixation in total hip arthroplasty, placing screws in which zone poses the highest risk to damaging the external iliac vasculature?
-
anterior-inferior zone
-
anterior-superior zone
-
posterior-inferior zone
-
posterior-superior zone
-
oblique zone
Corrent answer: 2
The acetabulum is divided into four quadrants with two bisecting lines. One from the ASIS to center of acetabular socket and the second is perpendicular to it. This is a source of repeat questions concerning the danger/safe zones of various quadrants for placement of acetabular screws. anterosuperior quadrant may injure the external iliac artery and vein. The anteroinferior quadrant may injure the obturator artery, nerve, or vein. The posterosuperior quadrant may injure the sciatic nerve, superior gluteal nerve and vessels and is considered the "safe zone". Posteroinferior quadrant may injure the inferior gluteal, internal pudendal structures. In general, posterior quadrants are safe except if long screws are placed posteroinferiorly. See illustration A. Wasielewski et al conclude "quadrant system provides the surgeon with a simple intraoperative guide to the safe transacetabular placement of screws during primary and revision acetabular arthroplasty."
OrthoCash 2020
-
-
Which of the following factors MOST places the knee at risk of patellar maltracking in total knee arthroplasty?
-
Thickness of patellar resection
-
Cruciate retaining component
-
Medial placement of patellar component
-
Preoperative patellar tilt
-
Lateral placement of patellar component Corrent answer: 5
Level 4 evidence by Kawano et al found that lateral patellar component position has been shown to directly correlate with lateral subluxation and maltracking. The study also found that there was no significant influence of the thickness of the patellar resection and preoperative patellar tilt on postoperative patellar tracking.
Avoiding implantation of the patellar component in a lateral position is paramount to tracking. Lateral positioning of the patellar component is shown in Illustration A.
An ideal percentage for patella component placement was calculated as 40-45% with the following equation: Distance of medial resected edge to central peg/length of patellar resection surface *100.
OrthoCash 2020
-
-
During a primary total knee arthroplasty, trial of components demonstrates a knee that is balanced in flexion and loose in extension. Which of the following will balance the flexion and extension gap?
-
Distal femur resection only
-
Distal femur augmentation and use of the same size polyethylene
-
Downsize femoral component and use a thinner polyethylene insert
-
Proximal tibia resection only
-
Distal femur augmentation and thicker polyethylene insert Corrent answer: 2
The goal in sagittal balancing of TKA is to obtain a gap that is equal in flexion and extension. General principles to remember: 1. Changing the distal femur only affects extension, 2. Changing the femoral component size only affects flexion, and 3. Changing the proximal tibia/polyethylene insert affects both extension and flexion. In the above scenario, distal femoral augmentation will correct the "looseness in extension" without changing the "balanced flexion".
The above principles are reviewed by Ries et al along with soft tissue balancing principles for stability in the coronal plane.
OrthoCash 2020
-
-
A 69-year-old female 16 years status post total knee arthroplasty complains of knee pain. A radiograph is provided in Figure A. Which of the following is true regarding the pathogenesis of the bony abnormality seen in the distal femur?
-
It is related to the toughness of the polyethylene liner
-
It is more likely to occur with highly cross-linked polyethylene compared to conventional polyethylene
-
It is caused by macrophage activation by polyethylene particles
-
It is most frequently caused by infection
-
It occurs more frequently in patients taking immunosuppressive medications.
Corrent answer: 3
The radiograph demonstrates polyethylene wear and osteolysis around the femoral component of a total knee replacement. Osteolysis is caused by macrophage activation from polyethylene particles. Ingham et al reviews the pathologic role of macrophages in osteolysis. Answer #1 is incorrect because toughness of the polyethylene is not related to wear rate, but does affect its overall mechanical strength. Answer #2 is incorrect because highly cross-
linked polyethylene liners have lower wear rates compared to conventional polyethylene. The listed reference by Huang concludes that there is an increased rate of osteolysis in mobile bearing TKA. This is a contradictory finding as mobile-bearing designs were created to decrease the stress and subsequent wear of the polyethylene
OrthoCash 2020
-
-
A 41-year-old male has steroid-induced avascular necrosis of the hip and decides to undergo metal on polyethylene total hip arthroplasty. His 80-year-old, sedentary father had a total hip replacement 5 years ago. With comparison to his father, the patient should be informed of the following risk?
-
Increased risk of sciatic nerve palsy
-
Increased longevity of prothesis
-
Increased risk for polyethylene wear and osteolysis
-
Reduced range of motion
-
Lower likelihood of revision surgery Corrent answer: 3
A younger, active patient will sustain more polyethylene wear and osteolysis due to greater activity levels and more years of use.
Kim et al prospectively studied 98 consecutive patients with osteonecrosis of the femoral head with an average follow-up was 9.3 years. Although there was no aseptic loosening of the components, they reported a high rate of linear wear of the polyethylene liner and a high rate of osteolysis in these high-risk young patients (16% in cemented femoral stems, 24% in uncemented stems).
OrthoCash 2020
-
-
In evaluating methods of polyetheylene sterilization for hip arthroplasty, gamma-irradiation in air compared to irradiation in an inert substance results in which of the following?
-
No difference in regards to outcome
-
Higher rate of cross-linking when irradiated in air
-
Lower rate of oxidation when irradiated in air
-
Accelerated wear and failure when irradiated in air
-
Better wear resistance and longevity when irradiated in air Corrent answer: 4
The standard of care is irradiation of polyethylene (PE) in an inert gas (e.g. argon, nitrogen or vacuum packaging). Irradiation of PE in air (i.e. oxygen present) results in oxidized PE while irradiation in the absence of oxygen results in greater cross-linking.
The quoted studies by McKellop et al and Sychtez et al both demonstrate that irradiation in air results in early PE delamination and cracking and accelerated failure due to increased oxidation.
OrthoCash 2020
-
-
A 67-year-old man who underwent total hip arthroplasty (THA) 4 years ago fell on to his right hip. His pre-injury right hip film is seen in Figure A while films of his current injury are seen in Figures B and C. Prior to the fall he had no thigh or hip pain. His ESR and CRP are within normal limits. During intraoperative assessment, the acetabular and femoral stems are found to be well fixed. What is the next best course of action?
-
Revision of the acetabular component and ORIF of the femur with locking plates and cerclage wires
-
Revison of the femoral component, bypassing the fracture by two cortical diameters
-
Revision of the femoral component with impaction grafting and cerclage wires
-
Revision to a cemented component, bypassing the fracture by two cortical diameters
-
ORIF of the femur with locking plates and cerclage wires Corrent answer: 5
This patient has a periprosthetic hip fracture at the level of the stem with a stable prosthesis, indicated open reduction and internal fixation as the treatment of choice.
The Vancouver Classification can be helpful in clinical decision-making regarding fixation versus revision of periprosthetic hip fractures of the proximal femur. A stable implant, by nature, does not need to be revised in the setting of adequate bone stock for fixation, but the ultimate test of stability should be in the operating room. Many fixation strategies are appropriate, but many implants include locking plate fixation for concerns of stress-shielded bone around the implant as well as use of unicortical fixation at the level of the stem.
Pike et al review the current trends in treating B1 fractures including locking plates with strut allografts, minimally invasive plate osteosynthesis (MIPO) and locking plates spanning femoral THA and TKA stems in selected patients. The authors conclude that no studies currently provide evidence establishing one
technique over the other and recommend treatment on a case by case basis.
Illustration A shows a possible fixation construct for this patient's fracture. The Vancouver Classification is seen in Illustration B and Illustration C represents an algorithm for treatment options.
Incorrect Answers:
1-4: All other answer choice include revising the implants, which is unnecessary based on this question stem.
OrthoCash 2020
-
-
While performing a cementless total hip arthroplasty in a healthy 68-year-old female, the surgeon notes an audible change while impacting the final broach. The broach is removed and a 1cm longitudinal crack originating at the calcar is visualized. Bone stock is otherwise preserved. What is the next best step in management?
-
Insert standard press-fit stem, weight bearing as tolerated postoperatively
-
Apply cerclage wire, insert standard press-fit stem, weight bearing as tolerated postoperatively
-
Insert long porous-coated stem, touch down weight bearing postoperatively
-
Insert long cemented stem, weight bearing as tolerated postoperatively
-
Insert long porous-coated stem, augment with cortical allograft and cerclage wires, touch down weight bearing postoperatively.
Corrent answer: 2
The patient has sustained an intraoperative proximal femur fracture and should be managed with placement of cerclage wire to prevent propagation of the fracture, insertion of the press-fit stem as planned, followed by weight bearing as tolerated postoperatively.
Intraoperative periprosthetic femur fractures occur in 1-18% of primary total hip arthroplasties (THA). Risk factors include the use of minimally invasive
techniques, press-fit cementless stems, revision surgeries, female sex, metabolic bone disease, Paget disease and intraoperative technical errors. Management of these fractures depends on timing of recognition (intraoperative or postoperative) and appropriate classification of the fracture (Vancouver classification for intraoperative fractures; Illustration A), which is dictated by fracture location, bone quality and implant stability. fMinimally displaced fractures at the calcar (Type A2) occur most often during broaching and are managed with removal of the broach, application of a cerclage wire around the fracture followed by insertion of the implant. Weight bearing does not need to be restricted postoperatively, as these minimally displaced calcar fractures are stable following cerclage wiring and implant placement. If implant stability is compromised or bone quality is poor (Type A3), a long diaphyseal stem may be used to bypass the defect. Minimally displaced fractures at the implant tip discovered immediately postoperatively may be managed with touch down weight bearing alone.
Berry reviewed management of perioperative fractures during THA. Minor cracks can be managed intraoperatively with cerclage fixation. Fractures noted postoperatively that do not affect implant stability or femoral integrity may be successfully managed with limited weight bearing and observation. Unstable implants or loss of femoral integrity require fracture fixation with either cerclage, strut grafts, plates or conversion to a long-stem implant.
Zhao et al investigated risk factors for intraoperative periprosthetic femoral fractures during cementless THA. A Corail stem (compared to Synergy), the anterolateral approach (compared to posterolateral), advanced age and a low Metaphyseal-Diaphyseal Index score (MDI score; Illustration B) were associated with increased risk of fracture. The MDI score was 25.89 (+/-8.11) in the fracture group versus 32.94 (+/-14.22) in the non-fracture group (p = 0.016). All fractures were treated with cerclage wire application and cementless implant insertion, followed by protected weight bearing postoperatively for 6 weeks, with no revisions required.
Illustration A depicts the Vancouver classification for perioperative periprosthetic femur fractures. Type A involves the proximal metaphysis [labelled A-C], type B involves the diaphysis [D-F]and type C fractures are distal to the stem tip and not amenable to insertion of the longest revision stem [G]. Each type is further sub-classified into type I if there is only a cortical perforation, type 2 is there is a nondisplaced crack and type 3 is there is a displaced unstable fracture pattern. Illustration B is an image from Zhao et al demonstrating radiographic measurements. The MDI is calculated by (D/F) / (G1+G2) where D = canal width 20mm above the mid-lesser trochanter line, F
= canal width 20mm below the mid-lesser trochanter line, G1 and G2 = two
cortical thicknesses at the same level as line F.
Incorrect Answers:
Answer 1: A cerclage wire should be placed prior to insertion of the stem, to prevent fracture propagation, loss of metaphyseal fit and ultimately stem subsidence.
Answer 3: Long porous-coated press-fit stems are usually reserved for periprosthetic fractures with extensive proximal bone loss (type A3) in which metaphyseal fixation is not possible, and therefore is not the best choice for this patient.
Answer 4: A long cemented stem is unnecessary for this periprosthetic fracture pattern. However, if a standard cemented stem is chosen, a cerclage wire should first be applied to reduce the fracture and prevent cement from entering the fracture site and potentially causing a nonunion.
Answer 5: Augmentation with cortical allograft is reserved for unstable periprosthetic fractures with diaphyseal bone loss (type B3) and therefore is not appropriate for this patient.
OrthoCash 2020
-
-
During revision total hip arthroplasty (THA), adjunctive motor-evoked potentials (MEPs) and electromyography (EMG) are utilized to monitor the sciatic and peroneal nerves. During the procedure, a conduction abnormality arises in the sciatic nerve. Which of the following actions would decrease tension on the sciatic nerve?
-
Provide traction to the leg
-
Pulsatile irrigation in the wound to remove blood clots
-
Flex the hip
-
Extend the hip
-
Extend the knee Corrent answer: 4
The only answer choice that would decrease tension on the sciatic nerve is hip extension.
Satcher et al used motor-evoked potentials (MEPs) and electromyography (EMG) monitoring during 27 consecutive total hip revision cases to identify intraoperative events that caused conduction abnormalities of the sciatic and peroneal nerves. Leg positioning was the most commonly associated factor that increased sciatic nerve pressure, causing changes in monitored parameters in 4 patients. The position that caused the most conduction abnormality was hip flexion during posterior acetabular retraction in these patients.
Incorrect Answers:
1,2,3,5: During hip flexion, the nerve can impinge on the acetabular retractor. Providing traction to the leg, pulsatile irrigation, hip flexion, and knee extension would all increase sciatic nerve pressure.
OrthoCash 2020
-
-
In animal models, which of the following is true when comparing hydroxyapatite(HA)-coated femoral stems to identical non-HA porous-coated stems after implantation?
-
Grit-blasted stems have decreased rates of loosening
-
Hydroxyapatite-coated stems have shorter time to biologic fixation
-
Harris hip scores are higher after porous-coated stem insertion
-
Transient thigh pain is increased after hydroxyapatite-coated stem insertion
-
Porous-coated stems show increased rates of calcar atrophy Corrent answer: 2
Hydroxyapatite-coated femoral stems have shown shorter times to biologic fixation in animal models, however clinical studies have yet to support their superiority to other stem designs.
Eckardt et al evaluated the influence of a proximal hydroxyapatite coating in comparison with a grit-blasted titanium surface of an anatomic hip stem in a canine model. Radiographically, animals with uncoated prostheses showed characteristic signs of loosening more frequently. Histomorphometrically, an average of 65% of the surface of HA-coated implants had bone contact, but this was present on only 14.7% of the surface of grit-blasted prostheses.
Kim et al followed 50 patients who underwent simultaneous bilateral hip arthroplasty in which a a proximally porous-coated titanium stem with hydroxyapatite coating was implanted on one side, and a proximally porous-coated titanium stem without hydroxyapatite coating was implanted on the other side. At a mean follow-up of 6.6 years, there was no difference in the rate of thigh pain, Harris hip score, or severity of calcar atrophy.
More recently, Camazzola et al performed a prospective randomized trial comparing hydroxyapatite-coated and non-hydroxyapatite-coated femoral total hip arthroplasty components in 61 patients. At 13 year follow-up, All femoral stems were well fixed on x-ray with no evidence of loosening. There was no statistically significant difference in the revision rates or in the Harris hip score between the two groups, and all femoral stems were well fixed radiographically. They concluded that there is no clinical advantage to the use of a hydroxyapatite coating on the femoral component for primary total hip arthroplasty.
OrthoCash 2020
-
-
A 60-year-old male tennis player undergoes a unicompartmental knee arthroplasty (UKA) shown in Figures A and B. Which of the following statements regarding this procedure is true?
-
Compared to total knee arthroplasty (TKA), UKA more closely approximates native knee kinematics
-
Patients undergoing a UKA and TKA have equivalent blood loss and pain
medication requirements
-
Compared to their TKA counterparts, UKA patients have a slower return to function
-
There is no difference in range of motion at short or long term follow-up when compared with TKA
-
Postoperative hospital stay is equivalent for UKA and TKA patients Corrent answer: 1
Figures A and B depict radiographs of a unicompartmental knee arthroplasty (UKA). UKA kinematics have been shown to most closely approximate native knee kinematics.
In an in vitro cadaver study, Patil et al found that TKA significantly changed knee kinematics while the unicompartmental replacement preserved normal knee kinematics.
Fisher et al performed a retrospective study comparing the short-term outcomes of small-incision unicompartmental knee arthroplasty (UKA) with standard total knee arthroplasty (TKA) in 91 consecutive patients older than 70 years. They found: 1) Blood loss was significantly more for the TKA group, as was the need for blood transfusion. 2) Patients with unicompartmental replacements had a much quicker return of function and discontinuation of pain medication. 3) While knee scores and ROM were similar preoperatively, both were better in the unicompartmental group at each postoperative time interval. 4) Narcotic use and length of hospital stay were also significantly less for the unicompartmental group. Therefore answers 2,3,4 and 5 are false.
OrthoCash 2020
-
-
With regard to unicompartmental knee arthroplasty, all of the following are true EXCEPT:
-
Females have a higher revision rate
-
BMI greater than 32 is not a risk factor for early implant failure
-
Presence of osteopenia contributes to premature implant failure
-
Lateral compartment arthroplasties have higher failure rates than medial compartment arthroplasties
-
Progressive arthritis within the remaining compartments of the knee is low 5 years post-operatively
Corrent answer: 4
Lateral compartment arthroplasties have not been shown to have higher failure rates than medial compartment arthroplasties.
Heck et al determined survivorship and risk factors for failure in their study of 294 UKA's with an average follow-up of 6 years. No statistically significant difference in the need for revision was demonstrated between those knees in which a medial as compared with a lateral compartmental arthroplasty had been performed. Female gender had a RR of revision of 1.7 compared to men. They also found that the average patient requiring revision had a BMI of 32.6 kg/m2, and an association between obesity (wt >81kg) and revision was statistically significant. However more recent data, summarized below, has called this particular finding into question.
Pandit et al sought to determine whether potential and previously described contraindications to UKA should apply to patients with a mobile-bearing UKR. With regards to BMI, they found no significant clinical or functional outcome difference, failure rate or survival between 551 UKRs performed in ideal weight patients (44-82kg) compared to non-ideal (82-185kg).
Weale et al evaluated the radiographic changes in 50 UKA's at 5 years postop. They found no correlation between the post-op tibiofemoral angle and the extent of recurrent varus recorded at five years, and stated that changes in alignment may be indicative of minor polyethylene wear or of subsidence of the tibial component. They also found that the incidence of progressive osteoarthritis within the knee was very low after UKA.
OrthoCash 2020
-
-
Which of the following factors is most likely to increase the risk of hip dislocation after a total hip arthroplasty (THA)?
-
Large head-to-neck ratio
-
Use of a skirted femoral head
-
Femoral component in 15 degrees of anteversion
-
Acetabular cup in 15 degrees of anteversion
-
Acetabular cup in 50 degrees of abduction Corrent answer: 2
The use of a skirted femoral head actually decreases the head to neck ratio as seen in illustration A, and leads to increased risk of hip impingement and dislocation after THAs. Illustration B shows an example of a smaller head-to-neck ratio causing decreased hip arc of motion before impingement occurs.
Barrack looked at implant design and orientation and its role in hip impingement and dislocations after THAs. Ways to minimize the risk of impingement and dislocation included avoiding the use of skirted heads, maximing head-to-neck ratio, and using chamfered acetabular liners whenever possible. With the use of computer modeling studies, he found that optimal femoral component anteversion is 10-20 degrees, while optimal acetabular component positioning is 10-20 degrees of anteversion and 45-55 degrees of abduction.
Illustration A shows how a skirted femoral head decreases the head to neck ratio. Illustration B shows an example of a smaller head-to-neck ratio causing decreased hip arc of motion before impingement occurs.
OrthoCash 2020
-
-
During total hip arthroplasty, which of the following techniques increases range of motion prior to impingement?
-
Using implants with a smaller femoral head
-
Using implants with a larger femoral head to neck ratio
-
Using a ultra high molecular weight polyethylene liner on the acetabulum
-
Decreasing femoral offset
-
Cementing the femoral stem Corrent answer: 2
Using implants with a larger femoral head to neck ratio increases range of motion prior to impingement and improves stability.
The efficacy of using a larger size diameter femoral head to improve stability has been recognized since the early 1970s. With the larger head (larger head to neck ratio), the distance to travel before subluxation and dislocation is greater, and more ROM is allowed before the neck impinges on the shell wall and levers the head from the shell.
Amstutz et al. evaluated the outcomes of 140 THAs using size 36mm femoral heads or larger. Patients were divided into 3 groups: revision for dislocation, revision for reasons other than dislocation, and primary THA. Six cases required revision surgery for instability and all were found to have mal-oriented acetabular components. After revision, all the hips were stable and none required the use of a constrained acetabular liner. The authors concluded that large diameter femoral heads provide additional stability not only for patients with recurrent dislocations, but for any revision.
Sikes et al. compared 52 THA cases at high risk of dislocation to a matched cohort. The high risk patients were all treated with a large diameter metal on metal components while the matched group received the standard metal on poly. The large head group had 0 disclocations compared to 2 in the standard head size. Ultra high molecular weight polyethylene liners (answer #3) are used in almost all metal on plastic THA today and have greater resistance to wear than prior generation of liners. However, they have no effect on ROM and impingement. Decreased femoral offset (#4) would result in decreased tension in the abductors and could result in increased risk of dislocation, but has no effect on impingement of the femoral neck on the acetabular cup. Cemented (#5) versus press fit stems should have no effect on ROM and impingement.
OrthoCash 2020
-
-
Which of the following motions shows the greatest difference between a normal and ACL deficient knee?
-
Posterior femoral translation at 30° flexion
-
Posterior femoral translation at 60° flexion
-
Axial rotation in full extension
-
Axial rotation at 50° flexion
-
Varus angulation at 30 ° flexion Corrent answer: 4
The study by Dennis et al, found a different axial rotation pattern in ACL deficient (ACL-D) knees compared to normal knees after 30° of knee flexion. Axial rotation was the same between the two groups in less than 30° of flexion. They also found normal and ACL deficient (ACL-D) knee patients demonstrated a similar pattern of posterior femoral translation during progressive knee flexion (0-120°). Additionally, the study showed increased variability in knee kinematic patterns observed in ACL-D knees as compared to the normal knees. Posterior femoral translation is substantially greater laterally than medially in both normal and ACL deficient patients, creating a medial pivot type of axial rotation pattern. With knee flexion, the normal tibia typically internally rotates relative to the femur and conversely, externally rotates with knee extension (i.e., screw home mechanism)
OrthoCash 2020
-
-
Figure A shows a ceramic head removed during a total hip revision. The component shows damage to the femoral head which was most likely caused by which of the following?
-
Third body debris
-
Chronic infection
-
Impingement of the femoral stem neck on the acetabular socket
-
Lift-off separation of the femoral head during hip range of motion
-
Insertion of the head on the femoral stem at time of initial surgery
Corrent answer: 4
Ceramic-on-ceramic articulation has been an attractive alternative to metal-on-polyethylene articulation because it exhibits low-friction, load-tolerant behavior with satisfactory wear characteristics. Stripe-wear as found in Figure A is a distinct type of impingement from the classic impingement of the femoral head on the acetabular socket found in episodes of instability (ie. lift-off separation) during gait.
Yammamoto et al in a retrieval study of 3 ceramic bearings and found significant stripe scars/wear at the rim of the alumina, but not at the weight bearing portion of the head. They concluded that stripe wear is caused by the femoral head making contact with the rim of the socket when the head undergoes lift-off separation from the socket.
Manaka et al found that the locations of the stripes were similar in retrieved and simulator ceramic heads. However, the stripes from the simulator were narrower than the short-term retrievals and much narrower than some longterm retrievals.
OrthoCash 2020
-
-
A 57-year-old man complains of knee pain that is exacerbated with weight bearing and ambulation. He underwent surgery on his knee 10 years ago following a motor vehicle collision. On physical exam he has medial and lateral joint line tenderness and no instability. Radiographs are provided in figures A and B. Conservative therapy with NSAID's and viscosupplementation is initiated. If he continues to develop further degenerative changes and needs arthroplasty what type of implant should be utilized?
-
Unicompartmental mobile bearing knee arthroplasty
-
Posterior cruciate retaining total knee arthroplasty
-
Posterior stabilized total knee arthroplasty
-
Constrained nonhinged total knee arthroplasty
-
Constrained hinged total knee arthroplasty Corrent answer: 3
The radiographs and clinical presentation are consistent with a patient who has undergone a previous patellectomy and is now developing degenerative arthritis of the knee. Patellectomy is an indication to use a posterior stabilized implant. The PS implant will offer better femoral rollback and reduce the risk of potential anteroposterior instability that may occur with use a cruciate retaining prosthesis.
Paletta et al review a series of patients undergoing TKA following patellectomy and compared them to a series of TKA patients who did not have a previous history of patellectomy. Most importantly they showed better outcomes in patellectomy patients who had a posterior-stabilized implant placed at the time of TKA.
Incorrect Answers:
Answer 1: UKA is not suitable for a patient with medial and lateral pain nor a patient with previous patellectomy
Answer 2: Posterior cruciate retaining knee following patellectomy risks anteroposterior instability
Answer 4 & 5: Constrained knee options are not necessary for patellectomy as there is no loss of varus/valgus stability.
OrthoCash 2020
-
-
A 66-year-old male is undergoing a total knee arthroplasty using a fixed bearing posterior stabilized component. During intraoperative trialing of the components it is noted that the flexion gap is loose, and extension gap is appropriate. If this is not corrected, what postoperative complication is this patient most at risk of having?
-
Spin out of the polyethylene
-
Periprosthetic fracture
-
Posterior knee dislocation
-
Osteolysis
-
Patellar instability
Corrent answer: 3
A posteriorly stabilized knee has a post built into the polyethylene bearing that articulates with the box of the femoral component in flexion to act as a cam
mechanism. If the knee is too loose in flexion, it is possible for the femoral component to "jump the post", causing a posterior dislocation.
Clarke and Scuderi review flexion instability as a mode of failure in knee replacements. They describe how this is usually due to lack of adequate balance at the time of surgery. They also report that revision surgery is usually the only way to correct symptomatic flexion instability.
OrthoCash 2020
-
-
A 56-year-old gentleman presents to your office one year after undergoing total hip arthroplasty with the implant seen in Figure A. He is concerned about the potential complications given the recent media attention his implant has received. He is currently asymptomatic. Which of the following statements is accurate regarding his prosthesis and future care?
-
He should have bi-annual LFTs measured, as metal ions are metabolized by the liver.
-
His risk of developing cancer is dramatically increased.
-
There is no correlation between activity level and serum levels of metal ions.
-
His prosthesis design is safe in women of child-bearing age as the ions cannot be transmitted via pregnancy.
-
His prosthesis design puts him at an increased risk for dislocation.
Corrent answer: 3
There is currently much debate over metal-on-metal (MOM) hip replacements and the optimal management of these patients in the post-operative period.
While data is currently limited, it has been shown that activity level does not affect serum metal ion levels.
Heisel et al. in their article from JBJS 2005 present level II evidence where they looked at the relationship between patient activity and cobalt and chromium ion levels. They found no correlation between patient activity and serum levels of cobalt or chromium, or urine levels of chromium.
Incorrect answers:
-
Metal ions are excreted by the kidneys, and there is not currently a concern regarding liver function in patients with MOM hip replacements.
-
Increased risk for developing cancer has not been shown.
-
Metal ions are capable of placental transmission and therefore should not be used in women of child-bearing age.
-
Increased incidence of hip dislocation is not a known complication of MOM hip replacements.
Illustration A shows a picture of ALTR (adverse local tissue reactions) from a metal-on-metal hip replacement. This is thought to cause premature loosening. Illustration B shows an MRI consistent with a pseudotumor. This is thought to be a reaction to metal ions with development of metal–protein complexes (haptens) that stimulate an immune reaction. Current indications for revision in a MOM hip include component loosening, progressive osteolysis, large effusion or pseudotumor, and unremitting pain.
OrthoCash 2020
-
-
-
A 71 year-old-male who underwent a primary total knee replacement in 1990 presents with right knee pain and instability for the past several months. Current images are shown in Figure A and Figure B. Which of the following is the most appropriate treatment at this time?
-
Revision of tibial component only
-
Management with a knee immobilizer for 3 months
-
Revision of tibial component with LCL reconstruction
-
Revision of tibial and femoral components with stems and/or augments
-
Revision of tibial and femoral components without stems and/or augments Corrent answer: 4
Figures A and B shows osteolysis around both tibial and femoral components with a significant varus deformity. Surgical management should consist of revision of both components using a constrained prosthesis with stems and/or augments.
Revision knee arthroplasty in the setting of two-component osteolysis presents a challenging problem for both the patient and the surgeon. Surgical management aims to restore the joint line while maintaining stability. In the setting of ligamentous laxity, increasing the constraint of the prosthesis is often necessary. In the setting of massive osteolysis, the use of stems and augments are often necessary to restore the joint line and maintain stability of the components within the medullary canals.
Morgan et al. provide a review of the use of constraint in total knee arthroplasty. Constraint should be considered in terms of both sagittal instability (ACL/PCL) and coronal instability (MCL/LCL), with only a hinged component providing complete stability in both planes.
Illustration A and B show the post-operative photographs after revision to a constrained non-hinged prosthesis with stems and augments was performed.
Incorrect Answers:
Answer 1: The femoral component osteolysis will also need to be addressed during the operation.
Answer 2: In this situation, conservative management should only be considered if the patient is medically unfit and low demand.
Answer 3: The femoral component osteolysis will also need to be addressed during the operation. With the use of a constrained device, reconstruction of the collateral ligaments will add little to varus/valgus stability.
Answer 5: Stems and augments will need to be used to restore stability and maintain the joint line in this situation.
OrthoCash 2020
-
-
The medial and lateral joint surfaces have different tibiofemoral geometry. How does this affect the kinematics of normal knee movement from full extension into flexion?
-
Tibia will externally rotate
-
Distal femur will pivot about a medial axis of the knee
-
Distal femur will translate anteriorly on the tibia
-
Distal femur will pivot about a lateral axis of the knee
-
No effect
Corrent answer: 2
The difference in tibiofemoral geometry between the medial and lateral knee causes the distal femur to pivot about a medial axis as the knee moves from full extension to flexion.
During flexion and extension, the normal knee will pivot about an axis that is offset medially through the tibia. The biomechanical mechanism behind this motion is based on two schools of thought. One thought is that the lateral femoral condyle has a larger radius of curvature relative to the medial femoral condyle. The effect of larger radius means that the lateral femoral condyle has greater net rollback compared to the medial condyle from knee extension to knee flexion. The second school of thought is there is markedly different topography of the lateral tibial plateau compared to the medial plateau (convex vs concave). This causes a rollback greater in the lateral condyle compared to the medial condyle.
Pellicci et al describe the kinematics of knee motion in total knee arthroplasty. During flexion of the knee, the lateral femoral condyle “rolls back” on the tibia while the medial femoral condyle remains relatively stationary in its anterior-posterior position. The kinematic description of the stationary tibiofemoral contact point medially and rollback laterally is termed "medial pivot".
Howell et al. reviewed 155 varus knees and forty-four valgus knees to assess the medial and lateral femoral condyle radii in osteoarthritis. They reported the difference of the medial and lateral radii to be 0.1-0.2 mm, which was considered clinically unimportant when aligning a total knee prosthesis.
Illustration A shows a schematic of a "big-tire vs small-tire" analogy. This shows that the sphere with the larger radius will experience a greater net rollback, which will produce a pivoting motion. Illustration B shows the
posterior translation the femur with progressive flexion, termed femoral rollback.
Incorrect Answers
Answer 1: As the leg moves from extension into flexion, there is external rotation of femur (or internal rotation of tibia).
Answer 3: Distal femur will translate posteriorly on the tibia during knee flexion.
Answer 4, 5: Distal femur will pivot about a medial axis as the knee moves from full extension to flexion.
OrthoCash 2020
-
-
A 55-year-old female with a long history of diabetes mellitus presents for evaluation of chronic knee pain. A current radiograph is shown in Figure A. Which of the following knee prostheses, shown in Figures B through E, would be most appropriate in the initial treatment of this patient?
-
Figure B
-
Figure C
-
Figure D
-
Figure E
-
Figure F
Corrent answer: 1
The radiograph shows findings consistent with neuropathic arthropathy, which can occur in the setting of chronic diabetes mellitus. The main problem with these patients after total knee arthroplasty is persistant instability that occurs secondary to ligamentous laxity. Therefore, a constrained prosthesis of some type (semi-contrained condylar prosthesis or hinge prosthesis) is indicated in the surgical treatment of these patients.
Parvizi et al reviewed 40 neuropathic joints treated with total knee arthroplasty. They reported that ligamentous instability necessitated the use of long stem components in 27 knees and rotating hinge prostheses in five knees. They concluded that careful ligamentous balancing, and appropriate selection of constrained prostheses particularly are important in these patients.
Kim et al describe the results of total knee arthroplasty undertaken for severe, neurosyphilitic Charcot arthropathy in nineteen knees. A cemented condylar
constrained knee prosthesis was implanted in all but two knees. The authors reported a 16% aseptic loosening rate, which required salvage by arthrodesis. They concluded that although Charcot arthropathy is not an absolute contraindication to total knee replacement, there is a high incidence of serious complications.
Morgan et al present a Level 5 review on the different types of arthroplasty and their levels of constraint.
Illustration A shows a clinical photograph of a high post semi-constrained total knee prosthesis, which is another type of contrained prosthesis besides a hinge, which may also be acceptable.
Incorrect Answers:
2-A cruciate retaining knee arthroplasty requires intact ligaments and is not indicated in the treatment of neuropathic arthropathy.
3-An arthrodesis is generally not necessary as the initial treatment option. 4-A posterior stabilized knee arthroplasty does not provide varus/valgus
stability, and requires intact ligaments. It is not indicated in the treatment of neuropathic arthropathy.
5-This patient has tricompartmental arthritis, which is not treated with a unicompartmental knee arthroplasty.
OrthoCash 2020
-
-
Patella baja is a known problem commonly encountered intraoperatively when converting which of the following patients to a total knee arthroplasty.
-
Previous medial compartment unicompartmental arthroplasty
-
Previous medial proximal tibial opening wedge osteotomy
-
Previous lateral distal femoral closing wedge osteotomy
-
Previous patello-femoral unicompartmental arthroplasty
-
Patella baja is not associated with any of the presented options Corrent answer: 2
Patella baja is often encountered during conversion from a high tibial osteotomy (proximal medial opening or lateral closing) to a total knee arthroplasty (TKA).
High tibial osteotomy (HTO) has been well described as an effective procedure for treatment of medial compartmental osteoarthritis of knee, especially in young and active individuals. Potential causes of patella baja when converting a previous HTO to TKA include scarring from the previous surgery and shortening of the patellar tendon.
Meding et al. reported on the results of 39 bilateral TKAs at an average of 8.7 years after unilateral HTO. They concluded that despite preoperative alignment differences, the results of TKA in knees with and without a previous HTO are not significantly different.
Wright et al. assessed the patellar height and patellar ligament length pre- and postoperatively in 28 patients who underwent a medial opening wedge proximal tibial osteotomy for varus gonarthrosis. They found that 64% of patients met the radiographic criteria for patella baja post-operatively. They concluded that the high incidence of patella baja following medial opening wedge proximal tibial osteotomy may have deleterious effects on patellofemoral biomechanics or may complicate subsequent total knee arthroplasty.
Incorrect Answers:
Answer 1 & 4: Conversion from uni-compartmental arthroplasties (either tibiofemoral or patello-femoral) has not been associated with patella baja.
Answer 3: Conversion of a previous lateral distal femoral osteotomy often results in patella alta.
Answer 5: Patella baja is commonly encountered when converting a HTO (either medial opening wedge or lateral closing wedge) to a TKA.
OrthoCash 2020
-
-
A 25-year-old healthy, active male undergoes lateral closing wedge high tibial osteotomy. Which of the following complaints is most commonly associated with this procedure?
-
Joint laxity
-
Infection
-
Anterior knee pain
-
Quadricep weakness
-
Limb lengthening
Corrent answer: 3
The most common complaint associated with lateral closing wedge high tibial osteotomy is anterior knee pain.
Lateral closing wedge high tibial osteotomies are commonly associated with anterior knee pain due to the high incidence of patella baja post-operatively. Patella baja is characterized by the lowering of the patella relative to its normal position, which is typically measured using the Insall-Salvati ratio of <
0.8 (Normal ratio = 0.8 - 1.2). The most common symptoms associated with patella baja include anterior impingement, knee pain, and knee stiffness.
Scuderi at al. evaluated the effect of proximal tibial osteotomy on patellar height in 66 patients. They found that patellar height decreased by 89%, as measured by the Insall-Salvati index, and 76.3%, as measured by the Blackburne-Peel index, post-operatively.
Wright et al. reviewed the complications associated lateral closing wedge and medial opening wedge high tibial osteotomy. They reported a 64% incidence of patella baja following these high tibial osteotomy techniques, with associated complaints of anterior knee pain.
Illustration A shows an X-Ray of patella baja after closed wedge high tibial osteotomy. The Insall-Salvati ratio is determined by measuring the ratio of patella tendon length (TL) to the length of the patella bone (PL) with the knee flexed at 30 deg.
Incorrect Answers:
Answer 1: Knee stiffness, not laxity, is more common after high tibial
osteotomy.
Answer 2: The incidence of deep infection is roughly 0 - 4%.
Answer 4: Quadriceps weakness typically exists prior to high tibial osteotomy. Answer 5: Leg shortening is more likely associated with lateral closing wedge high tibial osteotomy.
OrthoCash 2020
-
-
A 88-year-old female fell onto her right hip sustaining the fracture shown in Figure A. Past medical history is significant for mild dementia and moderate coronary artery disease. At baseline, she ambulates with a walker. There are concerns about her ability to maintain weight-bearing precautions following surgery. Which of the following is most appropriate for management of the femoral side?
-
Revision total hip replacement with a proximally coated femoral stem
-
Open reduction, internal fixation with plate and cerclage wires
-
Proximal femoral replacement with megaprosthesis
-
Impaction bone grafting
-
Cortical strut allograft with cerclage wiring Corrent answer: 3
The radiograph shows a Vancouver B3 periprosthetic fracture with poor proximal femoral bone stock. Given her age, co-morbidities, and dementia, the appropriate management of her condition would be a proximal femoral replacement with megaprosthesis. This can allow for immediate weightbearing in the post-operative period.
Vancouver B3 periprosthetic fractures are fractures around or just below the tip of a loose stem with poor proximal femoral bone stock. Options for management of this fracture include a fully coated stem, a fluted tapered stem, a proximal femoral replacement with megaprosthesis, allograft-prosthesis composite, and impaction bone grafting. In elderly patients with comorbidities and an inability to maintain the strict weight-bearing precautions that impaction bone grafting and allograft prosthetic replacements require, proximal femoral replacement with a megaprosthesis is the best option.
Duncan et al. were the originators of the Vancouver classification system for
periprosthetic fractures. Type A fractures are peritrochanteric, type B fracture are around the stem tip, and type C fractures are well below the stem tip.
Parvizi et al. review the indications for proximal femoral replacements with megaprostheses. They conclude it is a reasonable option for elderly patients with massive proximal femoral bone loss. The most frequent complications are aseptic loosening and dislocation.
Klein et al. identified 23 patients who underwent proximal femoral replacement for a Vancouver type-B3 periprosthetic fracture. At a follow-up of 3 years, 22 of 23 patients were walking with minimal pain. The most frequent complications were persistent drainage (2), dislocation (2), refracture (1) and acetabular cage failure (1).
Figure A shows a Vancouver B3 periprosthetic fracture with loose stem and poor proximal bone stock. Illustration A shows an example of a proximal femoral replacement. Illustration B shows a radiograph of a proximal femoral replacement used for a failed total hip replacement with massive bone loss. Illustration C shows the Vancouver classification (A, B1, B2, B3, C)
Incorrect Answers:
Answer 1: A proximally coated femoral stem would not obtain sufficient purchase with the poor quality proximal bone in this situation.
Answer 2: Given the degree of proximal femoral bone loss and loose stem, open reduction and internal fixation with a plate is not the best option.
Answer 4: Impaction bone grafting is an option for Vancouver B3 fractures, but are typically reserved for younger patients who have bone stock that needs to be restored.
Answer 5: Given the degree of proximal femoral bone loss and loose stem, cortical strut allograft with cerclage wiring is not the best option.
OrthoCash 2020
-
-
A minimal-incision technique with an incision no more than 10 centimeters has which of the following advantages compared to a standard incision for a total hip replacement?
-
lower post-operative visual analogue pain score
-
less transfusion requirement
-
shorter length of stay
-
better cosmetic result
-
less pain medication requirement Corrent answer: 4
Ogonda et al randomized patients to standard (16cm) versus MIS incision (<10 cm). They found no significant difference with respect to postoperative hematocrit, blood transfusion requirements, pain scores, or analgesic use.
There were also no differences in early walking ability or length of hospital stay and no differences in component alignment.
OrthoCash 2020
-
-
What is an advantage of utilizing a 36-mm instead of a 28-mm femoral head in the setting of a revision total hip arthroplasty?
-
Compensating for abductor deficiency
-
Decreasing volumetric wear
-
Decreasing trunion stress
-
Delaying neck-socket impingement
-
Compensating for vertical cup placement Corrent answer: 4
Increasing femoral head size delays neck-socket impingement, enhancing stability by increasing the excursion distance prior to dislocation.
The optimal bearing in total hip arthroplasty (THA) should allow for the best stability and function while preserving implant longevity. Greater motion, stability, and patient satisfaction have been correlated with larger femoral head sizes. Stability is associated with an increased displacement distance in larger diameter heads prior to a dislocation event (increased jump distance) combined with a greater impingement-free range of motion. Although larger femoral head sizes are associated with greater volumetric wear and trunion stress (and possible taper corrosion), newer bearings such as ceramic-on-ceramic may help mitigate these factors.
Kung et al. examined the effect of femoral head size and abductor integrity on dislocation events after revision THA. They separated 230 patients who underwent revision THA into 4 groups: (1) intact abductors mechanism + 28-mm femoral head, (2) absent abductor mechanism + 28-mm femoral head,
(3) intact abductor mechanism + 36-mm femoral head, and (4) absent abductor mechanism + 36-mm femoral head. They found that in patients with intact abductor mechanisms, the 36-mm femoral head was associated with a lower dislocation rate with 6-month minimum follow-up; femoral head size did not reduce dislocation events in patients with deficient abductor mechanisms.
Burroughs et al. performed an in-vitro study evaluating the range of motion and stability in THA with 28-44-mm femoral head sizes. They found that
femoral heads >32-mm provided for greater ROM and decreased component impingement. For these reasons, the authors conclude that large femoral heads may be beneficial in revision THA.
Illustration 1 demonstrates the increased jump distance when utilizing a larger diameter femoral head. Illustration 2 demonstrates a greater impingement-free range of motion with a larger femoral head.
Incorrect Answers:
Answer 1: Increasing femoral head size will not compensate for abductor deficiency.
Answer 2: Increasing femoral head size will increase volumetric wear. Answer 3: Increasing femoral head size will increase trunion stress.
Answer 5: Increasing femoral head size will not compensate for a vertical acetabular cup.
OrthoCash 2020
-
-
A 65-year-old male is now 6 weeks status post an uncomplicated total knee arthroplasty. Figures A and B represent his x-rays at today's visit. His primary complaint is knee stiffness. His current passive range of motion is 0-80 degrees, compared to 120 degrees
preoperatively, and he has failed to improve with physical therapy. He is otherwise afebrile, has no incisional erythema or pain, and CRP is within normal limits. Manipulation under anesthesia (MUA) will provide the largest improvement in flexion if performed before which of the following:
-
Before 6 weeks
-
Before 12 weeks
-
Before 26 weeks
-
Before 34 weeks
-
Outcomes after MUA are equivalent if performed within 1 year of surgery Corrent answer: 2
Manipulation under anesthesia (MUA) yields the greatest gain in flexion and improvement in overall range of motion if performed within 12 weeks postoperatively.
Post-operative knee stiffness (flexion < 90 degrees) occurs in 1.3%-12% of patients who undergo TKA. The preoperative risk factors for stiffness include decreased preoperative range of motion, age, diabetes mellitus, and socioeconomic status. Surgical factors can also contribute, including
overstuffing the patellofemoral joint, tight flexion/extension gaps, and excessive tightening of the extensor mechanism. If left untreated, loss of flexion negatively impacts functional outcomes, patient satisfaction, and ability to perform activities of daily living. MUA is considered the initial management for patients with flexion less than 90 degrees.
Issa et al. studied the optimal timing of MUA. They found that patients undergoing MUA before 12 weeks had a significantly higher mean gain in flexion (36.5° versus 17°), higher final range of motion (119° versus 95°), and higher Knee Society objective (89 versus 84 points) and function scores (88 versus 83 points) than those who had late manipulation under anesthesia (after 12 weeks). There was no statistically significant difference between MUA before 6 weeks vs MUA before 12 weeks.
Desai et al. similarly demonstrated that maximum flexion gains were achieved when MUA was performed between 12-14 weeks post-operatively. They also demonstrated that there is no benefit to multiple manipulations, with no observed flexion gain after the initial intervention.
Incorrect Answers:
Answer 1: When compared to patients undergoing MUA at 12 weeks, patients undergoing MUA at 6 weeks had no statistically significant benefit.
Answers 3, 4, 5: Maximum flexion gains are achieved between 12 and 14 weeks post-operatively.
OrthoCash 2020
-
-
A 51-year-old male presents with worsening left hip pain over the past 8 months. The patient reports hip surgery 5 years prior as seen in Figures A and B. His CRP level is within normal limits, ESR is at the upper limit of normal, and automated cell count following a hip aspiration yields a WBC of 15,000/µL (rr, 4500-11000µL) and 85% PMNs. Metal ion testing reveals elevated serum cobalt and chromium levels. What is the next best step?
-
Manual cell count of synovial fluid
-
2-stage revision arthroplasty
-
Acetabular cup revision with bone grafting
-
Femoral revision with cerclage wire placement
-
Hip arthrodesis
Corrent answer: 1
The patient has had a metal-on-metal (MoM) hip resurfacing with osteolysis. It is likely his WBC count is falsely elevated secondary to metal debris and corrosion. The next best step is a manual cell count to evaluate for metal debris, clots, fragmented cells, or other defects preventing accurate automated cell count.
Diagnosis of infection in the setting of MoM bearing surface can be difficult given very similar presentations. Typical workup includes CRP, ESR, synovial fluid WBC, and differential. Metal debris and corrosion reactions can confound the synovial fluid analysis leading to falsely elevated WBC counts in cases of aseptic failure. Prosthetic joint infection (PJI) and metallosis can occur concurrently so aggressive diagnostic testing is important for surgical decision making. Manual cell count of synovial fluid samples can identify metal debris and avoid false positives from automated cell testing. Supplementary tests, such as MARS MRI, metal ion levels, manual cell count, and repeat aspiration, can help guide appropriate management in these scenarios.
Yi et al. investigated ESR, CRP, synovial WBC and differential in diagnosing PJI in the setting of MoM hips or non-MoM hips undergoing revision for corrosion
or full thickness wear. They found that synovial fluid WBC count can be confounded by inaccurate automated cell counts secondary to foreign material and degenerating cells. They conclude that diagnosis of PJI in the setting of failed MoM bearings or corrosion is difficult due to falsely positive synovial fluid WBC.
Carrothers et al. evaluated the prevalence of complications of hip resurfacing arthroplasty. They found the most common complication was fracture of the femoral neck, followed by loosening of the acetabular component, femoral head collapse, loosening of a femoral component, infection, aseptic lymphocyte-dominated vasculitis-associated lesion (ALVAL), loosening of both components, and malposition of the acetabular component. They conclude that knowledge of complications following hip resurfacing is important to help select patients and counsel them on risks prior to surgery.
Figures A and B show a metal-on-metal hip resurfacing with supra-acetabular osteolysis.
Incorrect Answers:
Answer 2: Two-stage revision arthroplasty would be indicated if the aspiration and lab values were consistent with infection.
Answer 3 and 4: Acetabular cup revision or femoral revision in isolation only would not address the problem of metallosis.
Answer 5: Hip arthrodesis would not be indicated in this young patient with metallosis.
OrthoCash 2020
-
-
An 87-year-old female presents with longstanding knee pain. The structures identified in Figure A are formed through the pathologic activation of endochrondral ossification. The pathway involved in this process involves which of the following signaling molecules?
-
Indian hedgehog (Ihh)
-
Peroxisome proliferator-activated receptor gamma (PPARG)
-
Receptor activator of nuclear factor kappa-ligand (RANKL)
-
Osteoprotegerin (OPG)
-
Sclerostin
Corrent answer: 1
Osteoarthritis and formation of osteophytes are felt to be mediated by the differentiation of quiescent chondrocytes through the Indian hedgehog signaling pathway.
Indian hedgehog (Ihh) is an important mediator of chondrocyte and osteoblast differentiation in prenatal endochondral bone formation. Adult articular cartilage without osteoarthritis does not have active Ihh signaling. However, Ihh and its downstream signaling proteins have been found to be unregulated in osteoarthritic joints. Ihh may play an important role in activating endochondral ossification leading to some of the clinical features of osteoarthritis (osteophytes and subchondral cysts). Modulation of Ihh has therapeutic potential in the treatment of osteoarthritis.
Maeda et al. studied mice with Ihh genes ablated from postnatal chondrocytes.
They found a loss of columnar structure, premature vascular invasion, and formation of ectopic hypertrophic chondrocytes in the growth plate. They concluded that Ihh is essential for maintained trabecular bone, skeletal growth, and articular cartilage.
Wang et al. reviewed the molecular mechanisms associated with cartilage degeneration in osteoarthritis. The study found that upregulation of the Ihh signaling molecules leads to the development of osteoarthritis, similar to that found in injury-induced controls. Additionally, they found that inhibiting the Ihh pathway reduces the severity of injury-induced osteoarthritis in mouse models.
Figure A shows a knee with varus deformity and severe medial sided arthritis with femoral and tibial sided osteophytes(red arrows).
Incorrect Answers:
Answer 2: PPARG is a regulator of mesenchymal progenitor cells that induces adipocyte formation.
Answer 3: RANKL stimulates bone resorption through the interaction of RANK receptors on osteoclasts.
Answer 4: OPG inhibits both osteoclast activation and differentiation by acting as a decoy receptor for RANK-L.
Answer 5: Sclerostin inhibits the WNT pathway leading to decreased bone mass.
OrthoCash 2020
-
-
Figure 1 is the AP pelvis radiograph of a 55-year-old male with known hip dysplasia and chronic right hip pain. He has no prior surgical history and is otherwise healthy. Your initial plan is to reconstruct his hip center at the site of his true acetabulum. Your preoperative template demonstrates that your acetabular component will have 40° of abduction, 15° of anteversion. However, there will be 25% uncoverage at the superolateral margin. What is the most appropriate next step to ensure adequate cup fixation?
-
Increase the abduction angle to 60 degrees for better coverage
-
Medialize the acetabular component beyond the medial wall for improved coverage
-
Elevate the hip center in search of better bone stock
-
Accept 25% uncoverage and proceed with total hip arthroplasty as templated
-
Use cement augments to improve superolateral coverage of the acetabular component
Corrent answer: 4
The most appropriate next step is to proceed with total hip arthroplasty (THA) as templated. Less than 30% of uncoverage is acceptable and has not been associated with increased rates of aseptic loosening.
Dysplasia of the hip broadly refers to abnormal development of the hip that leads to poor acetabular coverage of the femoral head. The characteristic pelvic deformities of dysplasia include a retroverted acetabulum with bone loss at the superolateral margin. As a result, there is often insufficient bone stock to provide complete coverage of the acetabular component when placed at the true hip center. Insufficient coverage (<60-70%) can lead to a lack of initial stability and early failure. If the pre-operative template suggests extreme superolateral bone loss, the surgeon must then consider alternative methods of achieving satisfactory fixation, including downsizing the acetabular component size, medializing or elevating the hip center, or use of trabecular metal augments or cement augmentation for better coverage.
Haddad et al. discuss pre-operative considerations when performing primary total hip arthroplasty on dysplastic hips. The literature review suggests that
less than 30% uncoverage does not increase the risk of aseptic loosening.
Paavilainen et al. studied the short-term outcomes on 100 cementless total hip replacements in severely dysplastic hips. They describe the importance of exposing the proximal aspect of the pubic and ischial bones since the pelvic wall is usually hypoplastic, and the use of augmentation when the superolateral rim was deficient. Overall, their outcomes with cementless total hip replacements were equivalent to cemented prostheses.
Figure A is an AP pelvis radiograph demonstrating a dysplastic right hip. Illustration A is a T1-weighted coronal MRI of a hip. Hip dysplasia can be assessed using the center-edge angle, the angle formed between a vertical line through the center of the femoral head and a line connecting the center of the femoral head with the lateral edge of the acetabulum. Less than 20° is one marker of hip dysplasia. Illustration B demonstrates the concept of joint reactive forces. Joint reactive force is minimized when the moment arm of body weight and abductor tensioning are balanced. Anatomic changes that reduce abductor tension such as elevating the hip center, decreasing femoral head size, and increased valgus neck angle increase the joint reactive force.
Incorrect Answers:
Answer 1: Increasing the abduction angle to 60 degrees would reduce coverage of the femoral head and decrease joint stability, as well as leading to increased wear rate.
Answer 2: Though medialization of the acetabular component will decrease the joint reactive forces and provide increased coverage, there is no need to penetrate through the medial wall. Coverage is sufficient so violating the medial wall would not be indicated.
Answer 3: Elevation of the hip center will alter natural joint mechanics and reduce abductor tensioning leading to increased joint reactive forces. Bone quality also decreases as you move into the ileum further.
Answer 5: There is no need to use cement augmentation to increase coverage as your template suggests this patient will have adequate coverage.
OrthoCash 2020
-
-
A 65-year-old man presents to your clinic with chronic, progressive knee pain. Figure A is an x-ray of his right knee. He would like to pursue non-surgical treatment options. The AAOS clinical practice guidelines on the treatment of symptomatic knee arthritis support which of the following with "strong evidence"?
-
NSAIDs; tramadol
-
Weight loss; arthroscopic debridement
-
Weight loss; intra-articular steroid injections
-
Valgus-offloading brace; glucosamine chondroitin injections
-
Tramadol; acupuncture
Corrent answer: 1
The AAOS clinical practice guidelines (CPG) summary "strongly" recommends tramadol or NSAIDs for the treatment of symptomatic knee osteoarthritis.
Symptomatic knee osteoarthritis is widespread, with an incidence of 240 people per 100,000. Symptoms are often progressive, though addressing the modifiable risk factors of muscle weakness, large BMI, and repetitive loading can help with pain control. Definitive management is total knee arthroplasty, however, non-operative interventions are often successful in delaying surgery, particularly in younger patients.
The AAOS Clinical Practice Guideline Summary performed a systematic review of the available literature to propose evidence-based guidelines for the management of symptomatic knee osteoarthritis. The current guidelines show strong support for the engagement in physical activity (such as guided physical therapy), non-steroidal anti-inflammatory drugs, and Tramadol for the management of symptomatic osteoarthritis. The Clinical Guidelines provided “strong” evidence against acupuncture, glucosamine and chondroitin, hyaluronic acid, and arthroscopy with lavage or debridement.
Fishman et al. performed a double-blind, randomized study of 552 patients studying the efficacy of Tramadol vs placebo in the management of arthritic knee pain. Tramadol demonstrated a statistically significant improvement in knee pain over placebo as reported by the Patient Global Rating of Pain Relief Scale.
Incorrect Answers:
Answer 2: The AAOS CPG supports weight loss with "moderate" evidence and "strongly" recommends against arthroscopic debridement in symptomatic arthritic knees.
Answer 3: The AAOS CPG supports weight loss with "moderate" evidence and intra-articular steroid injections with "inconclusive" evidence.
Answer 4: The AAOS CPG supports valgus off-loading bracing with "inconclusive" evidence and "strongly" recommends against the use of glucosamine chondroitin injections
Answer 5: The AAOS CPG "strongly" recommends Tramadol though "strongly" recommends against the use of acupuncture.
OrthoCash 2020
-
-
An 82-year-old healthy male presents to the ED with right leg pain and inability to bear weight after a fall from standing. He has a history
of revision right total knee arthroplasty performed 5 years ago and was doing well until his fall this morning. On exam, he is able to actively extend his knee, though limited by pain, and is neurovascularly intact. Figures A-B are radiographs of his distal femur. What is the most appropriate treatment?
-
Retrograde intramedullary nail
-
Revision total knee arthroplasty with a stemmed femoral component
-
Nonoperative management with application of a long leg cast
-
Open reduction and internal fixation with a lateral plate
-
Antegrade intramedullary nail Corrent answer: 4
Open reduction and internal fixation with a lateral plate is the most appropriate treatment for this supracondylar periprosthetic femur fracture above a well-fixed total knee arthroplasty (TKA).
Supracondylar periprosthetic femur fractures are defined as fractures within 15cm of the joint line or within 5cm of the proximal end of the implant in the case of a stemmed component. Risk factors include rheumatoid arthritis, neurologic disorders, steroid use, anterior cortical notching of the femoral
component and revision TKA. Nonoperative management usually requires long-term immobilization, which can result in significant loss of knee range of motion (ROM). Therefore, surgical treatment is often preferred. An acceptable outcome is > 90° of knee ROM, fracture shortening < 2cm, varus/valgus malalignment < 5°, and flexion/extension malalignment < 10°.
Su et al. performed a literature review of the management of periprosthetic femur fractures above a TKA. First, it is crucial to determine the stability of the femoral component. An unstable femoral component requires either revision TKA or distal femoral replacement, depending on the quality of the distal femoral bone stock. A fracture with a stable femoral component can be well fixed with a buttress plate or retrograde intramedullary nail. In a series of 12 patients undergoing fracture fixation with lateral plate, all patients healed and returned to pre-fracture ambulatory status.
Zehntner et al. studied the surgical outcomes of buttress plating in 6 supracondylar femur fractures above TKA. There was no nonunion, loss of fixation or infection. Knee ROM averaged 97°, and all patients were ambulating postoperatively.
Figures A and B represent AP and lateral radiographs of a supracondylar periprosthetic femur fracture above a stemmed TKA. Illustrations C and D represent AP and lateral radiographs of a supracondylar periprosthetic femur fracture above a stemmed TKA status post repair with lateral plate.
Incorrect Answers:
Answer 1: Retrograde intramedullary nailing provides suitable fixation for distal supracondylar periprosthetic femur fractures. However, this is a stemmed femoral prosthesis and is not amenable to retrograde nailing.
Answer 2: The femoral component is well fixed and therefore, there is no need for revision.
Answer 3: Nonoperative management with prolonged immobilization results in a significant decrease in knee ROM and function.
Answer 5: Antegrade intramedullary nailing may be used in proximal or midshaft femur fractures above a TKA. Antegrade nailing would not be the best option for this distal femur fracture.
OrthoCash 2020
-
-
A 60-year-old woman presents for follow-up two weeks after right total knee arthroplasty. She complains of significant anterior knee pain after fall from standing onto a flexed knee. On physical exam, her passive range of motion is 0-120 degrees and she is stable to varus and valgus stress. She is able to achieve full extension with active range of motion, though she is experiencing severe pain. Lateral radiograph of the knee is provided in Figure A. What is the most appropriate next step?
-
Non-operative management with long leg cast
-
Closed treatment with immediate active range of motion
-
Removal of patellar component with early active range of motion
-
Open reduction and internal fixation of patella with wire or screw fixation
-
Extensor mechanism repair with Achilles allograft and revision of the patella Corrent answer: 1
The appropriate treatment for a transverse periprosthetic patella fracture with an intact extensor mechanism is closed treatment and immobilization in a long leg cast.
Periprosthetic patella fracture after total knee arthroplasty is rare (incidence of 0.68%-5.2%) and is overwhelmingly secondary to a traumatic mechanism.
Risk factors include excessive patella resection, use of cementless components, and elevated BMI. Broadly, the treatment algorithm for periprosthetic patella fracture depends on the stability of the implant and the involvement of the extensor mechanism. Ortiquerra and Berry classified periprosthetic patella fractures as intact extensor mechanism with well a fixed implant (Type I), extensor mechanism disruption with either a loose or stable implant (Type II), or intact extensor mechanism with a loose implant (Type III). Generally, fractures associated with component loosening or extensor mechanism injuries (Type II & Type III) require revision surgery and often yield poor results. Type I fractures are successfully managed non-operatively.
Nam et al. reviewed the management of extensor mechanism complications in
total knee arthroplasty. They report excellent results in Type I peri-prosthetic patella fractures managed non-operatively, with only 1 failure in 37 patients. They propose a treatment algorithm based upon fracture pattern (transverse vs vertical), component fixation, and extensor mechanism integrity. Type I injuries with transverse fractures should be immobilized in long leg casts, whereas Type I injuries with vertical fractures are more stable and can benefit from early active range of motion.
Cottino el al. similarly review the outcomes of peri-prosthetic patella fractures. They report a 50% complication rate and a 20-40% re-operation rate for type II and type III injuries, even with anatomic reductions.
Figure A is a lateral radiograph of the knee that demonstrates a transverse peri-prosthetic patella fracture without component loosening. There is no obvious extensor mechanism injury and the prompt indicates extension is intact. Illustration A represents the peri-prosthetic patella fracture treatment algorithm as proposed by Nam et al. Illustration B represents the Ortiquerra and Berry peri-prosthetic patella fracture classification.
Incorrect Answers:
Answer 2: Early active range of motion is not appropriate with transverse fracture patterns as this might propagate the fracture and compromise the extensor mechanism.
Answer 3: Removal of the patella component is not appropriate in this patient as the component is well fixed.
Answers 4 + 5: Type I fractures are often successfully managed nonoperatively. Surgical intervention is not appropriate for this patient.
OrthoCash 2020
-
-
A 68-year-old healthy male with a history of a right total hip arthroplasty 8 years prior presents with one year of right hip pain and several months of inability to bear weight. An AP pelvis radiograph is shown in Figure A. ESR is 8 mm/hr and CRP is 1 mg/L. What is the best treatment option?
-
Cemented acetabular cup with morselized bone graft
-
Two stage revision with antibiotic spacer
-
Cup-cage construct with or without morselized bone graft
-
Definitive removal of hardware/Girdlestone procedure
-
Anterior and posterior column plates with hemispherical uncemented cup Corrent answer: 3
Cup-cage constructs have demonstrated excellent outcomes in patients with pelvic discontinuity and Gross type V acetabular defects.
When treating chronic pelvic discontinuities, four general principles need to be addressed: 1) restoration of the acetabulum by reconnecting the ischium to the ilium, 2) optimizing contact of bleeding bone to component surfaces with ingrowth potential, 3) grafting osseous deficiencies, 4) obtaining a mechanically stable reconstruction to protect components until ingrowth is achieved. There are several different classifications of acetabular bone loss, though the Gross classification is unique in that it not only grades the degree of bone loss but also provides reconstructive options that may be considered (Illustration A & B). Due to the segmental bone loss associated with Gross
Types IV and V defects, cup-cage constructs are used to bridge segmental defects or areas of discontinuity.
DeBoer et al. studied the long-term outcomes of 28 patients with pelvic discontinuity treated with custom triflange acetabular prostheses. They reported no instances of broken screws, implant migration, or component revision at 10 years. Complications included one partial sciatic nerve palsy and five patients with dislocation.
Makinen et al. reviewed the role of cages in revision arthroplasty. In Gross Type V defects, cages provide mechanical stability to protect the acetabular cup until ingrowth is achieved. They report two case series following cup-cage reconstructions for pelvic discontinuity, demonstrating 88% survival at 44.6 months and 87.2% survival at 82 months. The survivorship of cage only constructs was 49.9% at seven years.
Figure A demonstrates a failed right total hip arthroplasty with associated chronic pelvic discontinuity. Illustration A demonstrates the Gross classification for acetabular bone defects. Illustration B demonstrates the suggested reconstructive options for defects classified according to the Gross classification system.
Incorrect Answers:
Answer 1: Cemented acetabular components with bone graft can be used to treat Gross type I defects. This construct does not provide sufficient stabilization for pelvic discontinuity.
Answer 2: Infection is unlikely with normal ESR and CRP values, therefore two-stage revision with an antibiotic spacer is not indicated.
Answer 4: Removal of hardware/Girdlestone procedure will not maximize function for this young patient with alternative surgical options.
Answer 5: Anterior and posterior column plates with hemispherical uncemented cups can be used in cases of acute pelvic discontinuity with minimal bone loss, not in cases of chronic discontinuity that require a cup-cage construct to bridge the defect.
OrthoCash 2020
-
-
A 60-year-old male presents with significant left knee pain and end-stage osteoarthritis. He failed non-operative management and is requesting a total knee arthroplasty (TKA). His past medical history is significant for a left distal femur fracture that occurred when he was struck by a car 30 years prior. A standing, full-length radiograph of his left lower extremity is shown in Figure A. His femoral coronal plane deformity measures 28 degrees. When proceeding with a TKA, what must be done to address this patient's coronal deformity?
-
Soft tissue balancing and intra-articular bone cuts
-
Distal femoral medial closing-wedge osteotomy
-
Distal femoral lateral closing-wedge osteotomy
-
High tibial osteotomy
-
Hinged TKA
Corrent answer: 3
Coronal plane deformities of the femur >20 degrees require an extra-articular femoral osteotomy to achieve proper mechanical alignment when performing a TKA.
Coronal and sagittal plane deformities of the femur less than 20 degrees can usually be addressed with intra-articular bone cuts and soft tissue balancing. Attempting to correct deformities greater than this without an extra-articular osteotomy can compromise ligamentous stability. This highlights the importance of careful pre-operative templating/planning and obtaining full-length, standing radiographs when clinically warranted. Although correcting severe deformities with staged or concomitant extra-articular osteotomies can be challenging, they are often successful when properly executed.
Lonner et al. provided a retrospective case series and review article addressing
severe extra-articular deformities with simultaneous femoral osteotomy and TKA in patients with osteoarthritis. In their 10-patient series, they were successful in restoring coronal alignment within two degrees of anatomic in all patients. They suggest securing the femoral osteotomy site with a plate or locked intramedullary nail, depending on the osteotomy site.
Rajgopal et al. presented a case series of TKA in 36 knees in the setting of extra-articular deformities. In this series, they treated all patients with intraarticular bone resection and soft-tissue balancing to address their deformities. Femoral-coronal, sagittal, and tibial-coronal deformities successfully treated included 11-18 degrees, 0-15 degrees, and 12-24 degrees, respectively.
Figure A is a standing, full-length radiograph of the left lower extremity demonstrating a significant coronal plane deformity resulting from a previous distal femur fracture malunion. Illustration A is a left knee radiograph demonstrating an extra-articular distal femoral osteotomy with placement of a fluted, press-fit stem. Illustration B is the same patient at 3-year follow-up demonstrating a fully healed osteotomy and maintained hardware alignment.
Incorrect Answers:
Answer 1: Coronal plane deformities with >20 degrees of coronal plane deformity require an extra-articular osteotomy.
Answer 2: A distal femoral medial closing wedge osteotomy would exacerbate this patient's varus deformity.
Answer 4: It would be inappropriate to address this patient's femoral deformity with a tibial osteotomy.
Answer 5: A hinged TKA is not indicated in this patient, and, prior to any TKA, this patient's femoral coronal plane deformity must be addressed in a 1 or 2-stage procedure.
OrthoCash 2020
-
-
A 76-year-old female presents with right hip pain 6 years after total hip arthroplasty. She denies constitutional symptoms at this time. On physical examination, her incision is well healed. Current radiographs are shown in Figures A and B. In addition to a complete blood count (CBC), C-reactive protein (CRP), and erythrocyte sedimentation rate (ESR), an alpha-defensin immunoassay is ordered. What does the alpha-defensin immunoassay test for?
-
The presence of an intra-articular, antimicrobial peptide
-
The presence of an antimicrobial peptide within serum
-
The presence of an intra-articular, pro-inflammatory marker
-
The presence of an intra-articular, pro-inflammatory cytokine
-
The presence of intra-articular leukocytosis Corrent answer: 1
A synovial alpha-defensin immunoassay tests for the presence of an intraarticular, antimicrobial peptide.
When there is suspicion of a periprosthetic infection, laboratory workup should
include CRP and ESR. If suspicion remains after laboratory examination, an aspiration of the joint should be performed for cell count and culture. A fairly recent test developed to aid in the diagnosis of PJI is the synovial alpha-defensin immunoassay. Defensins are antimicrobial peptides that are active against many bacteria, fungi, and enveloped viruses. Alpha-defensin is an antimicrobial peptide which is abundant in neutrophils and macrophages. It is present in the natural local tissue response to infection. In the setting periprosthetic joint infection (PJI) the levels of intra-articular alpha-defensin increase substantially and may reach levels that can be detected by an immunoassay. Alpha-defensin may be more prone to false positive results in adverse local tissue reaction caused by a metal-on-metal arthroplasty.
Bonanzinga et al. performed a prospective study to determine the reliability of the alpha-defensin immunoassay for diagnosing PJI. They aspirated hips and knees in all patients presenting with pain in both primary and revision arthroplasties. They found the sensitivity of the alpha-defensin immunoassay was 97%, the specificity was 97%, the positive predictive value was 88%, and the negative predictive value was 99% (95% CI, 96%–99%). They conclude that alpha-defensin appears to be a reliable test.
Shinsky et al. performed a study to evaluate the utility of commonly available tests for determining the presence of PJI in patients undergoing revision total hip arthroplasty. 235 consecutive total hip arthroplasties undergoing revision were evaluated. They found that no hip in a patient with a preoperative ESR of
<30 mm/hr and a CRP of <10 mg/dL was infected. They also conclude that a synovial fluid cell count of >3000 white blood cells/mL was the most predictive perioperative testing modality when ESR and CRP were elevated as well.
Figures A and B are AP and lateral radiographs of the right hip, respectively, demonstrating a region of periarticular erosions around the proximal femoral stem.
Incorrect Answers:
Answer 2: The alpha-defensin immunoassay is performed on synovial fluid, not serum.
Answer 3: this describes synovial CRP, another marker of inflammation that can be useful in PJI testing panels.
Answer 4: This describes the test for intra-articular interleukin-6 (IL-6) which has shown early promise in identifying PJI.
Answer 5: This describes the cell-count which is used in the diagnosis of PJI.
OrthoCash 2020
-
-
A 72-year-old male presents to your clinic with a 6-month history of left groin pain without preceding trauma. He previously underwent an uneventful left total hip arthroplasty 15-years prior at an outside institution. He has been ambulating with a cane for the last month due to pain. A left hip radiograph is obtained and presented in Figure A. Less than 4 centimeters of intact diaphyseal bone remains distal to the isthmus. An infection work-up is negative. Of the choices provided, what is the most appropriate management option at this time?
-
Continued observation
-
Placement of a femoral cortical strut allograft and cerclage wires
-
Femoral revision with impaction grafting and a cementless femoral stem
-
Femoral revision with a metaphyseal-engaging tapered stem
-
Femoral revision with a modular fluted tapered stem Corrent answer: 5
This patient has a Paprosky type-IIIB femoral defect (less than 4cm diaphyseal bone available for distal fixation) that can be revised with a modular fluted tapered stem
Femoral revision with a modular fluted tapered stem is the preferred method
in the treatment of Paprosky type-IIIB femoral defects. Proximal femoral replacement, allograft prosthetic composite, resection arthroplasty, and impaction grafting may also be considered in the treatment of Paprosky type-IIIB and IV femoral defects. Impaction grafting involves creating a neomedullary canal by impacting cancellous bone chips into the femoral canal followed by cementation of the final femoral component.
Valle et al. present a review article describing the Paprosky classification of femoral defects as well as treatment options. They differentiate a type-IIIA and IIIB defect as having >4 and <4 centimeters of intact diaphyseal bone available for distal fixation, respectively.
Hartman et al. describe management strategies for femoral fixation in the setting of revision total hip arthroplasty. They review classification systems of femoral bone loss and provide an overview of various treatment options. The authors feel that type-II and IIIA defects can be effectively treated with cylindrical, extensively porous-coated implants, whereas IIIB defects can usually be treated with a modular fluted tapered stem.
Figure A show a total hip arthroplasty with less than 4cm of diaphyseal bone available for distal fixation with extensive metaphyseal and diaphyseal bone loss. Illustration A shows a diagram of the Paprosky classification for femoral bone loss.
Incorrect Answers:
Answer 1: Observation would not improve this patient’s symptoms and would likely lead to worsening of the patient’s femoral defect.
Answer 2: Cortical strut allograft and cerclage wires would not address this patient’s loose stem and extensive intramedullary bone loss.
Answer 3: The femoral stem should be cemented when impaction grafting is employed.
Answer 4: While a diaphyseal engaging tapered stem may work, a metaphyseal-engaging tapered stem would not be effective in a Type IIIb femoral defect with extensive metaphyseal bone loss.
OrthoCash 2020
-
-
An 85-year-old woman sustains a ground level fall. Her THA was done 25 years ago. She was previously ambulatory but with a significant limp. With regard to the femur specifically, what is the Vancouver classification and preferred treatment option?
-
Vancouver C, revision to proximal femoral replacement
-
Vancouver B1, ORIF with impaction grafting
-
Vancouver AG, ORIF
-
Vancouver B3, revision to proximal femoral replacement
-
Vancouver B2, ORIF with femoral strut allograft augmentation Corrent answer: 4
The periprosthetic femoral fracture should be classified as Vancouver B3 given the location around the femoral stem with very poor proximal femoral bone stock, thus making reconstruction of this bone unrealistic. The best treatment options for a B3 fracture would be revision of the stem to a proximal femoral replacement or proximal femoral allograft composite.
The Vancouver periprosthetic classification system is one of the most useful classifications in orthopaedics as it can reliably guide treatment. Vancouver B1 or C type fractures confer a well-fixed stem and so ORIF would be the
treatment of choice. B2 and B3 fractures signify a loose femoral stem and so revision is necessary while B3 fractures have very poor proximal bone quality making any reliance on metaphyseal proximal bone for fixation or reconstruction futile.
Klein et al. retrospectively reviewed 21 patients (mean age 79) with Vancouver B3 fractures treated with a proximal femoral replacement. At most recent follow-up, 20/21 patients were ambulatory with no or minimal pain.
They note a relatively high complication rate (2 recurrent dislocators, 1 repeat fracture, 1 acetabular complication, 2 infections treated with I&D). However, given the circumstances and morbidity of non-operative management, they still recommend a proximal femoral replacement for this complicated patient group.
Savvidou et al. offer a good review of proximal femoral replacement both for periprosthetic fracture and also revision THA with significant proximal bone loss. They emphasize the complications such as dislocation and aseptic loosening and offer recommendations for minimizing them such as a large femoral head, soft tissue repair, and preservation/repair of abductors.
Figure A demonstrates a cemented total hip arthroplasty with significant osteolytic changes about both the femoral and acetabular components with multiple fractures around the grossly loose femoral stem. Taken from Klein et. al.
Illustration A demonstrates an example of a proximal femoral replacement. Taken from Savvidou et al.
Incorrect Answers:
Answer 1: Vancouver C fractures are distal to the tip of the stem and are treated with ORIF.
Answer 2: Vancouver B1 fractures are around the stem but the stem remains well-fixed and are treated with ORIF. This stem is loose.
Answer 3: Vancouver AG fractures are of the greater trochanter. This is not the fracture pattern associated here.
Answer 5: Vancouver B2 fractures are best treated with revision as mentioned above. Adding a strut allograft and ORIF would not address the stem loosening.
OrthoCash 2020
-
-
Which of the following liner types have been associated with early acetabular component loosening?
-
Figure A
-
Figure B
-
Figure C
-
Figure D
-
Figures B and C Corrent answer: 1
Figure A is an illustration of a lateralized or offset acetabular liner which has been shown to have a higher rate of loosening with primary and revision total hip arthroplasty (THA).
Extended offset polyethylene liners allow restoration of soft tissue tension by adding 4 to 10 mm of additional offset to the acetabular side. They translate the center of hip rotation laterally which will increase offset in the horizontal plane and add a few millimeters of additional limb lengthening in the vertical plane. The lateral translation of the center of rotation has been found to increase joint reaction forces and polyethylene wear. Additionally, this results
in an eccentric loading pattern and leads to increased torsional forces at the liner-shell interface and the bone-implant interface. This may cause motion at the interface resulting in failure of ingrowth and early loosening.
Glori performed a study to determine if torsion on an offset acetabular component may increase the risk of fixation failure. He found that a 70 kg person walking normally on a well-positioned 10-mm offset component will produce more torsion compared to a 4-mm offset component. Vertical cup placement was also found to increase torsion. He concludes that these torsional moments are comparable to moments shown to cause failure of the initial interference fit of cementless acetabular components in vitro. He suggests that after using an offset cementless acetabular component, one should initially limit weight bearing to minimize the risk of failure.
Archibeck et. al reviewed 1919 patients with primary THA and 346 with revision THA to evaluate acetabular component loosening. A 7-mm offset acetabular liner was used in 120 of the primary and 100 of the revision THAs. The aseptic loosening rate in the primary THA group was 0.12% in the standard offset and 4.2% in the extended offset groups at a minimum of 2 years follow-up. The aseptic loosening rate in the revision group was 1.7% in the standard and 7% in the extended offset groups at a mean of 4 years follow-up. They conclude that offset acetabular liners have a high failure rate in primary and revision THA.
Figure A is an illustration depicting an offset acetabular component. Figure B is an illustration depicting an oblique acetabular component or a face-changing liner. Figure C is an illustration depicting an elevated rim acetabular component or a lipped liner. Figure D is an illustration depicting a standard offset component.
Incorrect Answers:
Answers 2, 3, 4, & 5: Only the offset (lateralized) acetabular liner has been shown to lead to early implant loosening.
OrthoCash 2020
-
-
During a revision hip surgery, which of the following muscles labeled in Figure 1 can be transferred in the setting of an abductor deficiency?
1. A
2. B
3. C
4. D
5. E
Corrent answer: 1
The gluteus maximus (Label A) can be successfully transferred in the setting of abductor deficiency.
Injury to the abductors can occur during surgical approaches to the hip. The direct lateral approach (Hardinge), which involves splitting the gluteus medius and vastus lateralis, is most commonly associated with a post-operative Trendelenburg gait. Abductor and soft tissue deficiency, especially in the setting of a hip revision surgery, can result in an increased risk of dislocation. One solution to restoring abductor strength in the setting of an irreparable abductor injury is a gluteus maximus transfer to the greater trochanter.
Whiteside described a gluteus maxiumus and tensor fascia lata transfer for complete, irreparable avulsions of the hip abductors. Along with the tensor fascia lata, the anterior aspect of the gluteus maximus is freed and transferred to the greater trochanter so that the fibers are similarly oriented to the native abductor musculature. This technique showed promising results in a small series of patients.
Whiteside also described a similar technique to restore abductor function at the hip by transferring the anterior half of the gluteus maximus to the greater trochanter, suturing it beneath the vastus lateralis. In this paper, he also described a separate posterior flap transfer under the maximus to make up for minimius and capsular deficiency. He had good results in an eleven patient
series with this technique, restoring abductor function in nine patients.
Figure A is an axial MRI of the pelvis. Illustration A is a labeled axial MRI of the pelvis through a similar cross-section. Illustration B shows an intra-operative photograph of the anterior gluteus maximus flap prepared for transfer through a tunnel in the greater trochanter (asterisk).
Incorrect Answers:
Answer 2: B corresponds to the gluteus medius m. Answer 3: C corresponds to the iliacus m.
Answer 4: D corresponds to the psoas m.
Answer 5: E corresponds to the gluteus minimus m.
OrthoCash 2020
-
An 85-year-old female presents to your clinic following a left total hip arthroplasty done 20 years ago. Current radiographs of her left hip
are seen in Figures A and B. Which of the following best describes the process responsible for the radiographic findings seen?
-
Lymphocyte activation and infiltration due to metal debris
-
Macrophage activation following reaction to particulate debris
-
Wear of the femoral head-neck interface
-
Age related decrease in bone mass
-
Removal of bone stresses resulting in decreased bone density Corrent answer: 2
Osteolysis is caused by particulate debris activation of macrophages with subsequent bone resorption seen as expansile lytic lesions on radiographs.
Supra-acetabular osteolysis can occur secondary to particular debris most commonly generated from ultra-high molecular weight polyethyelne (UHMWPE) debris. Prior to the development of highly cross-linked polyethylene liners eccentric wear or backside wear of polyethylene liners lead to the production of high volumes of polyethylene particle debris. Particulate debris led to macrophage activation and the production of osteolytic lesions behind acetabular cups or around acetabular screws. This process often occurs more than 10 years out from the index surgery and can go on to cause fracture or loosening of components due to loss of prior bone ingrowth. Osteolysis rates have significantly decreased since the introduction of highly cross-linked polyethylene liners which have much lower wear rates.
Clohisy et al. in 2004 retrospectively reviewed 439 patients to investigate the reason for revision hip surgery. They found 55% of revisions were for aseptic loosening, 14% for instability, and 13% for osteolysis around a well-fixed implant. They concluded aseptic loosening was the most common reason for revisions surgery and osteolysis around a well-fixed component was a common reason for late revisions. (Of note this study was performed in 2004, reviewing outcomes of the older generation UHMWPE.)
Schmalzried et al. review wear in total hip and knee arthroplasty. They state osteolysis around uncemented acetabular components tends to migrate away from the implant interface into the cancellous bone of the pelvis. These expansile lytic lesions may be asymptomatic until pelvis fracture occurs. If noted, they recommend at least annual radiographic examination to monitor the progression of osteolysis.
Figures A and B show an AP and iliac oblique view, respectively, of supra-acetabular osteolysis characterized by a lytic lesion behind the acetabular cup and eccentric wear of the polyethylene liner.
Incorrect Answers:
Answer 1: Metal-on-metal implants can lead to an aseptic lymphocyte-
dominated vasculitis-associated lesion (ALVAL) with surrounding soft tissue damage or pseudotumor.
Answer 3: Trunnionsis or wear of the femoral head-neck interface can lead to failure of total hip arthroplasty due to metal ion related complications or due to gross trunnion failure.
Answer 4: Osteoporosis is characterized by a global age-related bone mass secondary to an imbalance between osteoblast and osteoclast activity.
Answer 5: Stress shielding can be seen when implants alter the typical forces seen by bone leading to local loss of bone density.
OrthoCash 2020
-
-
An 84-year-old woman, previously ambulatory with an walker, presents after a mechanical fall. X-rays are shown in Figures A. Assuming she is medically cleared for surgery, which treatment offers the best chance at the patient being ambulatory 1 year postoperatively?
-
Open reduction and internal fixation with lateral locking plate
-
Open reduction and internal fixation with medial and lateral plating
-
Retrograde intramedullary nail
-
Conversion to distal femoral replacement
-
Definitive treatment with external fixator Corrent answer: 4
In elderly, osteopenic patients with distal femoral periprosthetic fractures without sufficient bone for internal fixation, revision to a distal femoral replacement is the preferred treatment option and offer a higher chance at being ambulatory 1 year postoperatively.
For older individuals with comminuted distal femoral periprosthetic fractures, distal femoral replacement (DFR) is an effective treatment option. It allows for immediate weight bearing unlike ORIF. There is some controversy regarding DFR use in more proximal fractures that can be fixed with a plate or a nail or a combination of both. However, in fractures as distal as the one in this case, there is often insufficient distal bone stock to obtain fixation and so DFR is the best option. In converting to a distal femoral replacement, the tibial component often needs to be revised as well to accept the hinge mechanism. A rotating hinge is a part of all distal femoral replacements by definition as the collaterals will be incompetent after the distal femoral resection.
Hart et al. retrospectively reviewed 33 distal femur periprosthetic fractures in patients over 70 years old and compared 10 patients treated with DFR vs 23 treated with ORIF. Mean time to union with ORIF was 24 weeks with an 18% nonunion rate. Additionally, all DFR patients were ambulatory at 1 year compared to 76% of ORIF patients.
Chen et al. compared outcomes of primary DFR to patients who needed conversion to DFR as a secondary procedure after failure of an ORIF. There were significantly more complications in the group requiring later conversion. They argue for strong consideration of DFR in elderly patients and those at high risk for nonunion.
Figures A demonstrate an AP and lateral xray of cemented posterior stabilized TKA with a very distal femoral periprosthetic fracture. There is minimal residual distal bone attached to the femoral component.
Illustration A shows an example of a distal femoral replacement performed for a periprosthetic fracture.
Incorrect answers:
Answers 1, 2, and 3: While fixation may be possible, the quality and amount of residual bone would make this difficult. Additionally, in an elderly patient around a prosthesis, distal femoral replacement provides immediate weight bearing and a better chance at remaining ambulatory at 1 year postoperatively.
Answer 5: Rarely should definitive treatment of a periprosthetic fracture be performed with external fixation if other options are available given the theoretical risk of seeding to the prosthetic joint.
OrthoCash 2020
-
-
When performing a total hip arthroplasty (THA), which of the following combinations of implant and approach have the highest risk for intraoperative femur fracture?
-
Broaching for the implant shown in Figure A through an anterior approach in a 45-year-old male
-
Broaching for the implant shown in Figure A through a lateral approach in a 67-year-old female
-
Broaching for the implant shown in Figure B through an anterior approach in a 45-year-old male
-
Final implant insertion of the implant shown in Figure A through a lateral approach in a 67-year-old female
-
Final implant insertion of the implant shown in Figure B through a lateral approach in a 67-year-old female
Corrent answer: 5
Intraoperative fractures of the femur during THA are the highest during final implant insertion of an uncemented stem (Figure B) through a lateral approach in females older than 65.
There are certain risk factors that make intraoperative fractures with THA more likely. During THA performed through a lateral approach, the medial gluteal muscle may direct broaches and final implant positions in a medial and anterior direction. This may induce stress forces to the medial cortex, predisposing to calcar fracture. In general, uncemented stems subject the
femur to higher forces both during broaching and final implant placement which may also be a risk factor for perioperative fracture.
Abdel et al. performed a retrospective study to define risk factors, nature, chronology, and treatment strategies for periprosthetic femoral fractures in primary THA. They found an incidence of 1.7%. The incidence of femoral fracture in uncemented stems was 7.7% compared to 2.1% for a cemented stem. The probability of a postoperative fracture within 30 days after an uncemented stem was ten times higher than a cemented stem. They conclude that intra-operative fractures occur 14 times more often with uncemented stems compared to cemented stems.
Abdel et al. performed a study to define the risk factors, characteristics, and chronology of fractures in revision THA. They found an incidence of 12%.
Fractures were three times more common with uncemented stems compared to cemented stems. The incidence of femoral fracture varied by uncemented stem type: fully-coated (20%), proximally-coated (19%), and modular fluted tapered (16%), respectively. Most fractures occurred during final insertion of the femoral component in patients over 65 and were treated with cerclage wires, as nearly 70% were minimally displaced diaphyseal fractures.
Miettinen et al. performed a retrospective study to determine the incidence and risk factors for intraoperative calcar fractures during THA. They found an incidence of 3.7%. They found that the lateral Hardinge approach was a risk factor for intraoperative calcar fracture. Radiological analysis showed that in the calcar fracture group, there were more deviated femoral anatomies and proximal femur bone cortices were thinner. They conclude that to avoid intraoperative fractures, attention should be paid when cementless stems are used with deviant-shaped proximal femurs and with thin cortices.
Figure A is a highly polished, cemented stem. Figure B is a proximally porous-coated, uncemented stem.
Incorrect Answers:
Answers 1, 2, 3, & 4: Perioperative femoral fractures are most likely during final implant insertion through a lateral approach in a female over 65.
OrthoCash 2020
-
-
An 80-year-old female presents to your clinic with a complaint of pain and instability of her right total knee arthroplasty. On examination there is a well healed midline incision and a positive anterior drawer. She is unable to perform a straight leg raise.
Infectious work up is negative. A non-weight bearing radiograph with slight valgus stress is shown in Figure A. In addition to addressing her extensor mechanism, which of the following is the best option for revision of her total knee arthroplasty?
-
Polyethylene liner upsize
-
Reconstruction of the MCL
-
Femoral component revision
-
Revision to semi-constrained high-post implant
-
Revision to hinged-knee prosthesis Corrent answer: 5
Revision to hinged-knee prosthesis is indicated given significant multi-planar instability and a deficient extensor mechanism in an elderly patient.
In the revision setting, component selection should be directed toward the least constrained component that will adequately treat the patient. In multi-planar instability, hinged-knee prostheses are indicated to help supplement for incompetent collateral ligaments and avoid possible worsened deformity and dislocation in less constrained component options.
Pellegrini et al. discuss indications for hinged-knee prostheses including massive bone loss, severe ligamentous instability, flexion instability, revision of a previous hinged prosthesis, and complex fractures in elderly patients.
They cite sources confirming improvement in patient symptoms with hinged-knee prostheses but at the cost of high complication rates (32%). They
conclude a hinged-knee prosthesis can be used in complex primary or revision arthroplasty cases but with caution given the high complication rates.
Dennis et al. review revision total knee arthroplasty including component selection. They similarly state that hinged-knee prostheses should be used only when adequate stability cannot be achieved with less constrained systems.
Figure A shows a non-weight bearing AP of a right knee with significant valgus instability suggestive of MCL deficiency. Illustration A shows an example of a hinged-knee prosthesis.
Incorrect Answers:
Answer 1: Isolated polyethylene upsizing in the setting of global instability and MCL insufficiency would not allow for adequate stability and would leave residual instability of the knee.
Answer 2: Reconstruction of the MCL in the setting of an elderly patient with a total knee arthroplasty leads to extremely high failure rates and would leave the patient with significant residual instability.
Answer 3: Isolated femoral component revision would not address the instability issues, and in order to create a hinged-knee system, the tibia would need to be revised as well.
Answer 4: A semi-constrained implant would not be sufficient constraint to address multi-planar instability.
OrthoCash 2020
-
-
After implanting trials during a primary cruciate retaining total knee arthroplasty, the surgeon decides to recut the proximal tibia with increased slope. What intra-operative examination findings in the operative knee would lead to this decision?
-
Tight in extension, balanced in flexion
-
Loose in extension and flexion
-
Balanced in extension, loose in flexion
-
Balanced in extension, tight in flexion
-
Loose in extension, balanced in flexion Corrent answer: 4
Recutting the proximal tibia with increased slope would help to balance a knee that is tighter in flexion than extension.
The goals of sagittal plane balancing during primary total knee arthroplasty are to create a stable knee with equal flexion and extension gaps. The flexion gap is controlled by the posterior cut of the femur (and resultant implant size), the tibial cut, and the posterior cruciate ligament (PCL) and other surrounding soft tissue structures. Cutting the tibia with increased slope preferentially opens the flexion gap as a result of femoral rollback. In cruciate-retaining designs, recessing the PCL can also preferentially open up the flexion gap.
Finally, decreasing the anterior-posterior size of the femoral implant by recutting to a smaller size would also preferentially open the flexion gap.
Mihalko et al. reviewed soft-tissue balancing during total knee arthroplasty in the varus knee. They noted that intra-operative techniques used to balance the knee include femoral component rotation, osteophyte resection, soft-tissue release, and bone resection. The authors conclude that balancing the flexion and extension gap is critical to long-term success and patient satisfaction.
Manson et al. reviewed sagittal plane balancing during total knee arthroplasty. They explained the differences between anterior and posterior referencing systems and the basics of gap balancing techniques. The authors provide an overview of the strengths and weaknesses of various system types and provide troubleshooting advice regarding intraoperative sagittal plane balancing.
A measurement of the posterior tibial slope angle (PTSA) is shown in Illustration A.
Incorrect Answers:
Answer 1: Balancing the knee, in this case, would require resecting more distal
femoral bone or releasing the posterior capsule.
Answer 2: Balancing the knee, in this case, would require using a thicker polyethylene insert or adding medial and lateral augments to the tibial tray. Answer 3: Balancing the knee, in this case, would require increasing the size of the femoral component (AP dimension) with subsequent augmenting the posterior femur or taking slope out of the tibial cut.
Answer 5: Balancing the knee, in this case, would require augmenting the distal femur.
OrthoCash 2020
-
-
A 64-year-old male is evaluated for a painful, swollen right knee 3 weeks after undergoing a right primary total knee arthroplasty (TKA). As part of his workup, a right knee aspiration is performed and the synovial fluid white blood cell count returns at 10,500 cells/microliter. How should this value be interpreted in the context of evaluating for a prosthetic joint infection (PJI)?
-
It is above the cutoff of ~3,000 cells/microliter in the perioperative period; a PJI is likely present
-
It is below the cutoff of ~30,000 cells/microliter in the perioperative period; a PJI is unlikely
-
It is above the cutoff of ~10,000 cells/microliter in the perioperative period; a PJI is likely present
-
It is below the cutoff of ~50,000 cells/microliter in the perioperative period; a PJI is unlikely
-
It is not possible to interpret the synovial WBC count within 6 weeks of a primary TKA
Corrent answer: 2
A synovial fluid WBC count of 27,800 cells/microliter (approximately 30,000) within 6 weeks after primary TKA has been shown to predict prosthetic joint infection.
Prosthetic joint infection (PJI) is a relatively rare but devastating complication in total joint arthroplasty. Despite technological advances, diagnosis of PJI remains difficult. This is especially true in the acute perioperative period when post-surgical hemarthrosis and inflammation can confound the interpretation of the synovial fluid WBC count. Given these physiologic changes, a synovial fluid WBC of approximately 30,000 cells/microliter has been established as the cutoff for infection in the first 6 weeks following primary TKA.
Bedair et al. evaluated the diagnostic performance of the synovial fluid WBC count within 6 weeks of a primary TKA. They found that a cutoff value of 27,800 cells/microliter had a positive predictive value of 94% and a negative predictive value of 98% for the diagnosis of a PJI in the first 6 weeks following a primary TKA. The authors concluded that the standard cutoff value of 3,000 cells/microliter used at their institution would have led to unnecessary operations.
Mason et al. sought to determine the value of the synovial WBC count that would most accurately predict infection in a total knee arthroplasty. They found that aspirates with a synovial fluid WBC count higher than 2,500 cells/microliter and 60% PMNs were highly suggestive of infection, although the time from the original surgery to the aspiration was not mentioned. The authors concluded that the synovial fluid WBC count and differential analysis is a relevant indicator for the presence or absence of infection in prosthetic joints.
Incorrect Answers:
Answers 1 and 3: The cutoff for synovial fluid WBC count within 6 weeks of a primary TKA is approximately 30,000
Answer 4: While a PJI is unlikely, the cutoff for synovial fluid WBC count within 6 weeks of a primary TKA is approximately 30,000, not 50,000
Answer 5: The synovial fluid WBC count can be interpreted within 6 weeks of a primary TKA
OrthoCash 2020
-
-
You are performing a primary total knee arthroplasty and decide to use a design with a highly congruent medial compartment to try to replicate native knee kinematics. As the knee goes from flexion to extension, where is the center of pivot and how does the femur rotate and translate?
-
Lateral pivot, femur externally rotates and anteriorly translates
-
Medial pivot, femur externally rotates and posteriorly translates
-
Lateral pivot, femur internally rotates and anteriorly translates
-
Medial pivot, femur internally rotates and anteriorly translates
-
Medial pivot, femur internally rotates and posteriorly translates Corrent answer: 4
In terminal extension, the femur internally rotates relative to the tibia and the lateral femoral condyle translates anteriorly.
Highly congruent liners (HCL) offer a highly-conforming articular geometry via a tibial polyethylene to control kinematics and confer stability. In the native knee, the medial femoral condyle (MFC) and the lateral femoral condyle (LFC) do not move symmetrically. Rather, the larger MFC remains fairly immobile and acts as a pivot center while the smaller LFC glides posteriorly during knee flexion causing femoral external rotation and femoral rollback. HCLs use high articular congruence medially to keep the medial side positioned centrally. The medial side concavity is spherical and holds the MFC in place while it rotates in flexion. The lateral side is more of a slot than a dish, allowing the LFC to glide posteriorly in flexion. Finally, the anterior lip of the liner is raised to prevent anterior translation of the femur during flexion.
Atzori et al. describe the medial pivot knee in total knee arthroplasty. They note that, when a PS or CR polyethylene liner is used for TKA, excellent clinical results are generally obtained but knee kinematics are altered from native.
This can result in a paradoxical anterior slide of the femur in flexion, especially in a PCL incompetent CR knee, resulting in inferior quadricep function and reduced flexion.
Fang et al. report on a computational model comparing the medial pivot
designs with or without the posterior cruciate ligament (PCL). They conclude that either the PCL or post-cam mechanism is necessary for medial pivot prostheses to regain normal kinematics after total knee arthroplasty. The morphology of medial tibial insert was also shown to produce a small but noticeable effect on knee kinematics.
Illustration A is a diagram depicting how a highly congruent liner allows for medial pivot and translation through the lateral femoral condyle.
Incorrect Answers
Answers 1-3 and 5: These answers incorrectly state the normal kinematics of the knee towards terminal extension. Towards terminal extension, the femur internally rotates relative to the tibia and the lateral femoral condyle translates anteriorly. When the knee goes from extension to flexion, the femur externally rotates relative to the tibia and undergoes posterior rollback (lateral femoral condyle contact pressures translate posteriorly).
OrthoCash 2020
-
-
In a revision total knee arthroplasty, you are planning on using the implant shown in figure A. Which of the following is a relative contraindication to its use?
-
Plan on using a hinged component
-
There is a large uncontained defect in the tibial metaphysis
-
Plan on using a stemmed component
-
Patient has rheumatoid arthritis
-
There is a capacious tibial diaphysis with thin cortices Corrent answer: 2
The implant shown is a stepped porous-coated metaphyseal sleeve for the proximal tibia. They require circumferential containment for stability and are therefore contraindicated for the management of uncontained defects.
Current options for revisions performed for metaphyseal cavitary defects include cementation, structural allograft, and metaphyseal-filling trabecular forms such as the metaphyseal sleeve or trabecular metal cones. There has been a recent growing interest in obtaining metaphyseal fixation to improve overall implant stability and longevity. Stepped porous-coated metaphyseal sleeves have been shown to have promising mid-to-long term data and are efficient and simple to use. However, they are expensive, can be difficult to remove, and are less effective for uncontained defects. Trabecular metal cones have promising short-to-mid term data and are expensive and difficult to remove but, unlike stepped porous-coated metaphyseal sleeves, can be used to treat severe uncontained defects in the metaphysis.
Haidukewych et al. review current concepts of metaphyseal fixation in revision total knee arthroplasty. They note that the use of a long-stem cemented tibial
revision component is not appropriate for an uncontained defect within the metaphysis, as the design stability relies upon a circumferential fit. They advocate for the use of tantalum trabecular metal cones, as they allow for adaption to the defect for a custom fit and are compatible with multiple implant types.
Bauman et al. report on the limitations of structural allograft in revision TKA. They reviewed 79 knees that underwent revision utilizing structural allograft. They noted a 22.8% failure rate with this technique. They suggest a selective utilization of structural allograft and encourage alternative methods of fixation in patients with large metaphyseal defects.
Figure A is an example of a stepped porous-coated metaphyseal sleeve for the proximal tibia. Illustration A shows examples of trabecular metal cones.
Illustration B is a radiograph of a stepped porous-coated metaphyseal sleeve with a tibial stem. Illustration C is a radiograph of a proximal tibia with a trabecular metal cone.
Incorrect Answers:
Answers 1, 3, 4, and 5: These are not contraindications to metaphyseal sleeve. The use of a sleeve with an osteopenic diaphyseal canal can actually aid as a guide for bone resection as intramedullary guides in these scenarios are often unreliable.
OrthoCash 2020
-
-
Figure A shows a 38-year-old male 2 week after left hip surgery. Two months later he presents with the x-rays shown in Figure B. Which of the following risk factors likely led to this type of failure?
-
Infection
-
Gender
-
Varus positioning
-
Sclerotic bone
-
Age
Corrent answer: 3
The patient has a femoral neck fracture following hip resurfacing placed in varus position.
Hip resurfacing is a controversial procedure with best outcomes seen in young males with good bone stock. The two most common modes of failure following hip resurfacing are femoral neck fracture and aseptic loosening. Risk factors for femoral neck fracture include osteonecrosis of the hip, femoral neck notching, osteoporotic bone, large amounts of femoral head bone loss, femoral
neck impingement, female sex, and varus positioning of the femoral component. Awareness of those risk factors that are surgeon dependent is important to decrease rates of failure. Following a periprosthetic femoral neck fracture conversion to total hip arthroplasty is recommended.
Gabriel et al. present a case of femoral stem fracture following hip resurfacing in a 15-year-old male with a large segment of femoral head osteonecrosis with collapse. They conclude that in patients with avascular necrosis of the majority of the femoral head, surgeons should consider using an implant with a substantial femoral neck.
Amstutz et al. present their results of a series of 600 metal on metal hip resurfacing arthroplasties. They found 5 (0.83%) femoral neck fractures all of which had some form of structural and/or technical risk factor present. They conclude that understanding the factors that contribute to femoral neck fractures may limit their prevalence.
Figure A shows a left hip resurfacing in varus alignment. Figure B shows a subsequent radiograph with periprosthetic femoral neck fracture. Illustration A shows a conversion to a total hip arthroplasty following periprosthetic femoral neck fracture.
Incorrect Answers:
Answer 1: There are no signs of infection based on this vignette.
Answer 2: Female, not male gender is thought to be associated with a higher risk of femoral neck fractures following hip resurfacing.
Answer 4: Osteoporotic bone is a risk factor for fracture following hip resurfacing, not sclerotic bone.
Answer 5: Older age may be associated with higher risk of femoral neck fracture.
OrthoCash 2020
-
-
A 75-year-old female patient underwent a primary total hip arthroplasty 20 years ago for osteoarthritis. She did well for many years, but for the last year has had progressively worsening groin pain. Figure A is a recent radiograph. ESR is 12 mm/hr (ref 0-25) and CRP is 4 mg/L (ref 0-10). Which of the following studies is the next most appropriate step in evaluation?
-
Technitium-99 bone scan
-
Joint aspiration
-
Serum Co and Cr levels
-
CT scan
-
Dynamic fluoroscopy
Corrent answer: 4
This patient has osteolysis from conventional polyethylene wear and what are likely stable implants. Assuming an infection has been ruled out, Judet oblique view radiographs or a CT scan should be completed as part of the preoperative workup.
In a painful hip arthroplasty with osteolysis, the treatment algorithm is dictated by the stability of the implants. The primary treatment for poly wear and osteolysis with well fixed components is polyethylene liner exchange and conversion to a highly crosslinked poly liner when applicable. Additionally, it is important to characterize the osteolytic defects pre-operatively in order to prepare for defects that may require bone grafting or revision implants. Judet oblique view radiographs or a CT scan can provide this important information. Bone grafting may be performed into the osteolytic lesions but is not required. If the acetabular component is well fixed and in good position, revision should be done with great caution as the size of the remaining defect is often underappreciated on imaging and can make subsequent reconstruction very challenging. Cementation of a new poly liner can also be an option should compatible liners not be available or if the acetabular component locking mechanism has been damaged.
Zimlick et al. demonstrate via a cadaver study that a simple AP pelvis can often underappreciate the size and extent of osteolytic defects especially of the posterior column. Judet view oblique radiographs or a CT scan is important to fully appreciate the amount of bone loss.
Walmsley et al. review the treatment of these types of osteolytic poly wear cases with isolated head and liner exchanges along with important pre- and intra-op considerations.
Figure A is an AP pelvis that demonstrates a metal on poly uncemented total hip arthroplasty with eccentric poly wear and osteolysis around the acetabular component.
Incorrect Answers
Answer 1: A bone scan may be helpful in equivocal cases of component loosening. However, in this case revision is necessary and intra-operative testing of component stability is the gold-standard. Always be prepared to revise all components in any revision operative setting when necessary.
Answer 2: Normal ESR and CRP values make the likelihood of infection low. Answer 3: Serum Co and Cr levels are useful in cases of metal on metal articulations or when there is concern for trunionosis. In this case, the eccentric poly wear and osteolytic defects in a hard bearing on poly articulation make this failure mechanism incredibly unlikely.
Answer 5: Dynamic fluoroscopy would be more useful in instability cases, but is generally not useful in the workup of a painful THA.
OrthoCash 2020
-
-
A 60 year-old man with a history of atrial fibrillation and a mechanical aortic valve replacement undergoes total hip athroplasty via a posterior approach. Post-operatively, at the recommendation of his cardiology team, he is restarted on warfarin. Four days after discharge, the patient returns to the emergency department with a 10% drop in hematocrit (now 20%) and a new complete sciatic nerve palsy. His INR is 6 and his dressing is saturated. What is the best course of action?
-
Reversal of INR, application of incisional wound vac, clinical monitoring of nerve palsy
-
Transfusion to raise the hematocrit to 30%, sequential neurovascular examinations
-
Reversal of INR, urgent surgical irrigation and debridement
-
Hold warfarin and transition to enoxaparin, application of incisional wound vac, sequential neurovascular examinations
-
Reversal of INR, placement of a temporary vena cava filter, exploration of the sciatic nerve
Corrent answer: 3
This patient has developed new wound drainage and a nerve palsy that is likely secondary to a wound hematoma after becoming supratherapeutic on warfarin. The patient's INR should be reversed (Vitamin K +/- FFP) and decompression of the hematoma should be performed in an expedited manner in order to minimize injury to the sciatic nerve.
Therapeutic anticoagulation, whether for mechanical heart valves, atrial fibrillation, or DVT/PE, can carry risks in the post-operative setting such as wound drainage and hematoma formation. The risks and benefits of prophylactic anticoagulation must be weighed against the postoperative risk of bleeding complications for individual patients.
MacDougall et al. performed a retrospective review examining the risks of therapeutic anticoagulation in the post-arthroplasty setting. Treatment with warfarin was found to significantly increase the risk of wound drainage/hematoma formation, as well as the risk of deep and superficial wound infections postoperatively.
Austin et al. published a case report of a patient with a postoperative sciatic nerve palsy due to anticoagulation that was treated with urgent decompression.
Incorrect Answers:
Answers 1 and 2: The new nerve palsy points to a hematoma and requires decompression in order to avoid permanent nerve injury.
Answer 4: Transition to enoxaparin does not address the hematoma and should not be initiated while the patient's INR is supratherapeutic.
Answer 5: An IVC filter is not appropriate for this patient, given that he was placed on anticoagulation for a mechanical valve and AFib -- neither of which is affected by an IVC filter.
OrthoCash 2020
-
-
A 73-year-old man who underwent a cementless total hip arthroplasty 8 years ago was in a high-speed motor vehicle accident and sustained the injury seen in Figure A. Appropriate management includes which of the following?
-
Revision of the femoral component to a long, diaphyseal engaging stem
-
Open reduction internal fixation
-
Revision of the acetabular component and revision of the femoral component to a long, diaphyseal engaging stem
-
Revision of the femoral component to a cemented long stemmed prosthesis
-
Proximal femoral replacement Corrent answer: 2
Open reduction internal fixation is the most appropriate management given the well-fixed stem and adequate bone stock.
Periprosthetic fractures around a total hip arthroplasty are nearly always treated operatively. Operative treatment is directed by the location of the fracture, stability of the implant, and remaining bone stock. When the fracture occurs around or distal to a well-fixed implant with good bone stock the treatment of choice is to fix the fracture. Fracture fixation can be achieved with a long laterally based plate and may need hook plates, locking screws, and/or cerclage cables.
Masri et al. reviewed periprosthetic fractures with regard to diagnosis and management. They state femoral periprosthetic fracture treatment is based on location, implant stability, and bone stock. They also state that this allows for reproducible assessment of these factors and directs the formation of reliable treatment plans.
Ricci et al. describe a surgical technique of indirect reduction of periprosthetic fractures around a stable implant. They reviewed 50 consecutive patients treated with an indirect reduction technique following a periprosthetic fracture around a stable implant. They found all fractures healed at an average of 12 weeks and of the 41 who returned for follow up 30 returned to their baseline ambulatory status. They conclude that indirect reduction with a single lateral locking plate for stable periprosthetic fractures leads to satisfactory outcomes.
Figure A shows a radiograph of the left hip and femur showing a periprosthetic hip fracture around a well-fixed stem. Illustration A shows postoperative radiographs following open reduction and internal fixation of the left periprosthetic fracture. Illustration B shows a depiction of the Vancouver classification of periprosthetic hip fractures.
Incorrect Answers:
Answer 1: The femoral component is well fixed and revision to a long stem would therefore not be necessary.
Answer 3: There is no fracture around the acetabular component so revision of the acetabular component is not necessary.
Answer 4: Revision of the femoral component to a cemented stem is not the
most appropriate treatment option in this fracture.
Answer 5: There is adequate bone stock to repair the fracture so a proximal femoral replacement would unnecessarily remove functional bone.
OrthoCash 2020
-
-
According to the 2013 American Academy of Orthopaedic Surgeons' Guidelines for the treatment of symptomatic osteoarthritis of the knee, which of the following treatments was recommended and strongly supported?
-
Acupuncture
-
Hyaluronic acid injections
-
NSAIDs or tramadol
-
Glucosamine and/or chondroitin
-
Arthroscopic lavage and/or debridement Corrent answer: 3
The only recommendation that was supported of the above mentioned answer choices was the use of NSAIDs or tramadol for patients with symptomatic
osteoarthritis of the knee.
The 2013 AAOS Guidelines for the treatment of symptomatic arthritis discuss both conservative and non-conservative forms of therapy and critiques the literature supporting each modality. Amongst the strong or moderately strong recommendations were weight loss, low impact physical activity, and NSAIDs and/or tramadol for the treatment of pain. While tramadol is a u-opioid receptor agonist, NSAIDs block the cyclooxygenase enzyme. Numerous modalities were not supported including the use of acupuncture, needle lavage, hyaluronic acid, glucosamine and chondroitin, arthroscopic lavage, lateral wedge insoles, and braces. Of note, they were unable to recommend for or against steroid injections, medial unloader braces, tylenol, opioids (not tramadol), pain patches, growth factor or PRP injections, and arthroscopic partial meniscectomy in patients with osteoarthritis of the knee with a meniscal tear.
Zhang et al. critically appraise the quality of the guidelines and review current research evidence for the management of osteoarthritis. They find that the overall quality of the existing guidelines is sub-optimal, and consensus recommendations are not always supported by the best available evidence.
Zhang et al., in part II of their study, present 25 recommendations for the treatment of osteoarthritis. They cover pharmacologic, non-pharmacologic and surgical modalities including joint replacements, arthroscopy, fusion, and osteotomy. Their strongest recommendation is that all patients with hip and knee OA should be given information access and education about the objectives of treatment and the importance of changes in lifestyle, exercise, pacing of activities, weight reduction, and other measures to unload the damaged joints.
Illustration A shows the mechanism of action of different anti-inflammatory medications.
Incorrect Answers:
Answers 1, 2, 4, 5: These were not supported in the 2013 AAOS Guidelines.
OrthoCash 2020
-
-
A 68-year-old man presents to your office with complaints of increasing left hip pain, especially when he first starts walking. He underwent right total hip arthroplasty (THA) approximately 10 years ago. He is afebrile and his incision appears well-healed with no evidence of infection. His current radiograph is shown in Figure A. Which of the following statements is true?
-
The extent of osteolysis can be accurately determined from his radiographs
-
Urine N-telopeptide levels would be expected to be decreased in this patient
-
TNF-alpha activity would be expected to be decreased in this patient
-
RANK activity would be expected to be increased in this patient
-
CRP and ESR levels would be expected to be increased in this patient
Corrent answer: 4
The patient's radiograph demonstrates evidence of osteolysis, which is mediated by macrophages that indirectly increase RANK activity.
Osteolysis represents a histiocytic response to wear debris. Specifically, particulate debris formation leads to macrophage activation. Activated macrophages release cytokines such as TNF-alpha, which indirectly promotes osteolysis by increasing RANK (a receptor found on osteoclast-precursor cells). Indications for revision THA secondary to osteolysis include pain, with conservative treatment reserved for those who are asymptomatic. Although osteolysis may be detected on radiographs, the extent of osteolysis is often underestimated, so CT is the preferred method of imaging.
Holt et al. performed a literature review on the biology of aseptic osteolysis. They reported that the most important mediators of osteolysis were TNF-alpha and IL-1. These mediators produce a pro-osteoclastogenic effect in response to implant-derived wear particles.
Bitar et al. published a review focusing on the biological response to prosthetic debris. They reported that RANKL and osteoprotegerin (OPG) have been shown to play a major role in the initiation and progression of osteolytic lesions. More specifically, RANKL is an osteoblast receptor which activates osteoclasts by binding Receptor Activator of NF-κB (RANK). The RANK pathway is the chief regulator of bone turnover (osteolysis) whereas osteoprotegerin is the antagonist of this pathway, and thus the RANK/RANKL/OPG pathway is considered crucial for the occurrence of osteolysis.
Figure A is a radiograph of the left hip demonstrating eccentric poly wear (this is generating the debris) and osteolysis around the cement mantle surrounding the femoral component.
Incorrect Answers:
Answer 1: Radiographs typically underestimate the extent of osteolysis. Answer 2: In the setting of osteolysis, urine N-telopeptide levels would be expected to be elevated as they are a marker of bone turnover.
Answer 3: TNF-alpha activity would also be expected to increase.
Answer 5: CRP and ESR levels would be expected to be normal in a patient with aseptic osteolysis.
OrthoCash 2020
-
-
When investigating a periprosthetic hip or knee joint infection, which marker is most sensitive and specific for infection?
-
Alpha-defensin
-
Intraoperative frozen section
-
Serum interleukin-6 (IL-6)
-
C-reactive protein (CRP)
-
Leukocyte esterase (LE) colorimetric strip test Corrent answer: 1
Synovial alpha-defensin has been shown to be 100% sensitive and 98% specific for periprosthetic joint infections. The other answers can aid in the diagnosis of a periprosthetic joint infection (PJI), however, they do not have higher sensitivity and specificity as alpha-defensin.
The 2018 definition is the most established predictive clinical tool for the diagnosis of PJI. While WBC, ESR, and CRP are included in the criteria, these are all nonspecific markers of inflammation. Inflammatory markers such as CRP, while highly sensitive, can be affected by renal clearance. The LE strip test is more specific (97-100%) but with only a cited 69-81% sensitivity.
Alpha defensin is an antimicrobial peptide produced by neutrophils in synovial fluid, with levels greater than 5.2mg/L being cited as 100% sensitive AND specific for infection.
Wyatt et al. performed a systematic review investigating the efficacy of alpha defensin and LE strip test as biomarkers for PJI. The authors found that alpha-defensin had a pooled diagnostic sensitivity and specificity of 100% and 96%, respectively, while LE was only 81% sensitive albeit 97% specific. They concluded that alpha defensin was of greater specificity.
Deirmengian et al. compared alpha-defensin to LE as markers of PJI, as diagnosed by MSIS criteria. They found that alpha-defensin was 100% sensitive and specific at diagnosis, while LE was 69% sensitive and 100% specific. They also found that 17% of LE test strips could not be interpreted due to blood interference. The authors concluded that alpha defensin was a superior marker for infection.
Incorrect Answers:
Answer 2: The intraoperative frozen section has a sensitivity of 85% and specificity of 90-95%.
Answer 3: Serum IL-6 has a sensitivity of 100% and a specificity of 95%. Answer 4: CRP has demonstrated a sensitivity of 88-96% and specificity of 70-77% for PJI.
Answer 5: Multiple studies have compared synovial alpha-defensin to LE colorimetric strip testing and have shown alpha defensin to be more sensitive and specific for PJI.
OrthoCash 2020
-
-
A 67-year-old active male with a prior total knee arthroplasty presents to clinic with inability to extend his knee after falling while exiting a bus 4 months ago. On examination, he has full passive range of motion but can only extend to 85°. His radiograph is depicted in Figure A. What is the most effective long-term treatment for this patient to regain function?
-
Primary quadriceps tendon repair with transosseous tunnels
-
Primary quadriceps tendon repair with suture anchors
-
Primary patellar tendon repair
-
Extensor mechanism reconstruction
-
Revision total knee arthroplasty with rotating hinge device Corrent answer: 4
This patient's history and examination is consistent with a chronic quadriceps rupture after total knee arthroplasty (TKA). The most effective long-term treatment to regain function in this active patient would be extensor
mechanism reconstruction.
Extensor mechanism insufficiency is a devastating complication following TKA. Chronically-disrupted extensor mechanism after TKA may include quadricep tendon and patellar tendon ruptures and patella fractures. In chronic cases, primary repair with or without local tissue augmentation have had disappointing results and been associated with high failure rates, whereas the use of an extensor mechanism allograft has been shown to more effectively restore active extension in a substantial percentage of patients. More recently, extensor mechanism reconstruction using Marlex mesh is becoming more popular and has had some success in the literature.
Bates et al. published a review article on extensor mechanism disruption after TKA. They discuss nonsurgical management including of the use of walking aids and knee braces for the inactive patient, as well as surgical options including primary repair and reconstructive techniques using allograft, autograft, and synthetic material. They recommended allograft reconstruction, as it has the potential to improve extensor mechanism function, augment host tissue, maintain range of motion, and decrease dependence on walking aids.
Browne et al. performed a study to investigate the results of a novel surgical technique of using synthetic mesh for extensor mechanism reconstruction.
They reported that 70% of patients demonstrated an extensor lag of no more than 10°, maintained functional flexion, and had no loss of extension at final follow-up. They concluded that the use of synthetic mesh to reconstruct a disrupted extensor mechanism is a straightforward surgical procedure that was successful and durable in the majority of patients in their study.
Abdel et al. published a case series on 77 patients who underwent extensor mechanism reconstruction with the Marlex mesh technique. They reported a 2-year survivorship of 95% and 86% for patients with and without prior extensor mechanism reconstruction using Marlex mesh, respectively. They also reported improved patient Knee Society Scores and improved postoperative extensor function. They concluded that reconstruction of the extensor mechanism with use of Marlex mesh is a viable option in patients with catastrophic disruption after TKA.
Figure A is a radiograph demonstrating patella baja after a TKA, indicating a quadriceps tendon rupture.
Incorrect Answers:
Answer 1: Primary quadriceps tendon repair using transosseous tunnels in the setting of a chronic extensor mechanism disruption after TKA is fraught with
high failure rates including re-rupture.
Answer 2: Primary quadriceps tendon repair using suture anchors in the setting of a chronic extensor mechanism disruption after TKA is fraught with high failure rates including re-rupture.
Answer 3: This patient's history and examination is consistent with a chronic quadriceps tendon rupture, not a patellar tendon rupture.
Answer 5: Without a functioning extensor mechanism, revision total knee with a hinged device will fail.
OrthoCash 2020
-
-
A 65-year-old male presents to your office for evaluation of chronic debilitating left hip pain over the last 5 years. He requires a shoe lift to ambulate. Radiographs are shown in figure A. The patient undergoes surgical treatment with a left THA, and his intra-op radiographs reveal equal leg lengths. The sciatic nerve was well visualized and protected during the procedure. On the first postoperative day, the patient is noted to have weakness in ankle dorsiflexion with paresthesias over the dorsum of the foot. What intraoperative technique could have prevented this complication?
-
Hip extension and knee flexion during exposure
-
Subtrochanteric femoral shortening osteotomy
-
Smith-peterson anterior approach
-
Increasing femoral offset
-
Upsizing the femoral component Corrent answer: 2
The patient has a Crowe type III dysplastic left hip with chronic dislocation and significant shortening of the extremity. A THA resulting in equal leg lengths would place the patient at risk for sciatic neuropraxia; therefore this procedure should be accompanied by either a concomitant femoral shortening osteotomy or significant downsizing of the components to accommodate for the increase in limb length.
Sciatic nerve palsy is a significant complication associated with THA. The peroneal subdivision of the nerve is the most commonly affected (80%). There are several etiologies of sciatic nerve palsy including direct trauma during exposure (generally associated with a posterior approach), stretch causing by an acute significant increase in leg length, hematoma causing direct nerve compression, and heat generated during cementation. There are also several risk factors for the development of a sciatic nerve palsy including hip dysplasia, revision surgery, female gender, post-traumatic osteoarthritis and as previously mentioned, limb lengthening. Overall, this is a devastation complication, as only 35-40% of patients will go on to recover full strength after injury.
Masonis et al. reviewed the results of 21 primary THAs performed in Crowe grade III or IV hip dysplasia combined with a subtrochanteric shortening osteotomy. At an average follow up of 5.8 years, it was noted that 91% of the osteotomies had healed while 2 developed non-unions requiring revision. No patients reported neurological deficiencies. Furthermore, the Harris hip score improved from 32.5 to 73.6. The authors concluded that the subtrochanteric shortening osteotomy is a safe and predictable method of restoring the anatomic hip center in high developmental hip dislocation with the caveat that overall complications are still higher than for patients with routine THA for degenerative arthritis.
Krych et al. performed a retrospective review of 28 hips in 24 patients with Crowe IV hip dysplasia who underwent cementless THA with a simultaneous subtrochanteric shortening osteotomy. At an average 4.8-year follow-up, the mean Harris hip score increased from 43 points preoperatively to 89 points (p
< 0.01). It was noted that 12 of the hips (43%) had an early or late complication or reoperation (2 due to osteotomy site non-union and 4 due to instability). There were no sciatic neurapraxic injuries in the cohort. Authors concluded that cementless total hip arthroplasty combined with a subtrochanteric femoral shortening osteotomy in select patients provided
excellent results but with complications rates higher than those THAs performed for primary DJD.
Figure A: AP pelvis radiograph of a patient with severe left hip dysplasia with 15% proximal migration and 75% femoral head subluxation consistent with Crowe stage III.
Illustration A: The Crowe classification detailing the proximal displacement and femoral head subluxation in dysplastic hips.
Incorrect answers:
Answer 1: While hip extension and knee flexion during exposure decrease sciatic nerve tension and is a great technique to minimize sciatic nerve injury during exposure, it would not affect the final outcome given this patient's left leg was significantly lengthened during the procedure.
Answer 3: An anterior approach has decreased the risk of sciatic nerve injury (and increased risk of femoral nerve injury) compared to the posterior approach but ultimately this patient's sciatic nerve palsy is due to nerve stretching to accommodate the LLD without femoral shortening.
Answer 4: Adjusting femoral offset does not affect limb length and adjusting it would not decrease the risk of a sciatic nerve injury.
Answer 5: Downsizing, NOT upsizing, the femoral component would cause shortening of the overall limb length and potentially help avoid this complication. Upsizing the femoral component may also change offset but will not lead to a decrease in limb length as is needed in this case.
OrthoCash 2020
-
-
A 67-year-old male golfer presents with right hip pain of three years duration which has persisted despite daily oral anti-inflammatory medications and physical therapy. He wishes to compete in a recreational golf tournament in three months. Which treatment modality would offer the most reliable functional improvement in this patient?
-
Corticosteroid injection
-
Hyaluronic acid injection
-
Hip resurfacing
-
Cemented hip hemiarthroplasty
-
Cementless total hip arthroplasty Corrent answer: 5
This active 67-year-old patient presents with end-stage osteoarthritis of the hip. Of the options given, total hip arthroplasty (THA) would provide the most predictable symptomatic relief and return to function in 3 months.
Current supported conservative therapies for osteoarthritis of the hip includes NSAIDs, tramadol, weight loss, activity modification, physical therapy, and corticosteroid injections. Once a patient has exhausted conservative management, surgical options include THA, hip hemiarthroplasty, and hip resurfacing, though the latter is more controversial and would not be indicated in this patient. THA remains the preferred treatment for patients over the age of 50 with advanced bipolar degenerative changes and has shown to provide the most predictable pain relief and superior functional outcomes.
Corticosteroids have been shown to offer more limited and transient symptomatic relief. Furthermore, several studies have demosntrated an increased risk of periprosthetic joint infection (PJI) when THA is performed within 3 months of an ipsilateral hip intraarticular injection. As a result, a corticosteroid injection given at this visit would preclude surgery for at least 3 months.
Stea et al. performed a multinational study to examine the fixation method used in hip replacements with age in context. They reported that in patients
over 75 years old, cementless fixation had a significantly higher revision risk than hybrid fixation (cemented femur, cementless acetabulum). They concluded that hybrid fixation should be utilized in THA for patients over the age of 75 years.
Werner et al. published a study to evaluate the association of a preoperative intraarticular hip injection prior to THA on postoperative PJI incidence. They reported a significantly greater risk of PJI at 3 months and at 6 months in patients who underwent ipsilateral hip injection 3 months prior to a THA compared to controls. The authors did not observe this association when the injection was performed greater than 3 months prior to THA.
Figure A is an AP pelvic radiograph depicting end-stage osteoarthritis of the right hip.
Incorrect Answers:
Answer 1: While a corticosteroid injection may be a reasonable option, it would not provide as predictable or lasting symptomatic relief as a THA. Answer 2: The AAOS issues a strong recommendation against hyaluronic acid injections in the 2017 AAOS Clinical Practice Guidelines.
Answer 3: The indications for hip resurfacing are presently limited to younger (<50 years) active patients and to those with severe proximal femoral deformity that may make THA difficult.
Answer 4: Cemented hemiarthroplasty would not be a reasonable option for this patient given the severe degenerative changes observed in the acetabulum.
OrthoCash 2020
-
-
Which comorbidity is most significantly associated with the early surgical wound complications requiring surgical interventions following primary total knee arthroplasty?
-
Type 2 diabetes mellitus
-
Previous arthroscopic knee surgery
-
Post-operative aspirin DVT prophylaxis
-
Body mass index of 31 kg/m^2
-
Tourniquet use for 70 minutes
Corrent answer: 1
Increased wound complication rates have been associated with diabetes mellitus.
Post-operative wound complications following total knee arthroplasty can be potentially devastating to the patient and come at increased care-related costs. Multiple systemic risk factors have been associated, including diabetes mellitus, renal disease, steroid use, and inflammatory arthritis. Additionally, previous open knee surgery can predispose the patient to deleterious outcomes, especially when previous incisions are not incorporated into the approach. Prior to surgery, modifiable risk factors must be addressed to ensure primary wound-healing including smoking cessation, nutritional optimization, and discontinuing any potentially harmful immunomodulating medications.
Jones reviewed wound healing complications in total joint arthroplasty patients. The author described local wound risk factors consisting of traumatic arthritis, multiple previous incisions, and poor vascular perfusion and systemic risk factors of diabetes mellitus, rheumatoid arthritis, renal disease, liver disease, steroid use, tobacco use, and nutritional compromised state. The author recommended optimizing modifiable risk factors, such as smoking cessation and nutritional status, and the use of the most lateral of previous incisions.
Galat et al. performed a retrospective study of 17,784 primary knee replacements of which 59 patients with early wound complications that required operative intervention. They reported that patients requiring early surgical intervention had major subsequent surgery and deep infection rates of 5.3% and 7.0%, respectively, which was statistically significant compared to controls. Further, a diagnosis of diabetes mellitus was more strongly associated with patients requiring surgical treatment of early wound complications than inflammatory arthritis, smoking, peripheral vascular disease, steroid use, body mass index >30 kg/m^2, and previous open knee surgery. The authors concluded that patients requiring early surgical intervention for wound complications are at significant risk for developing further complications and they stressed the importance of obtaining primary wound-healing after the index procedure.
Incorrect answers:
Answer 2: Open knee surgery, as opposed to arthroscopic knee surgery, has been reported as a risk factor for wound complications, especially when a more medial based incision is used during the arthroplasty procedure. This can result in necrosis of the lateral region of the wound, which may sometimes require a rotational gastrocnemius flap.
Answer 3: Deep vein thrombosis prophylaxis with aspirin is associated with fewer wound complications compared to low-molecular-weight heparin and novel oral anticoagulants.
Answer 4: Morbid obesity (BMI >40 kg/m^2) has been associated with increased infection rates and wound complications.
Answer 5: The use of a tourniquet and the duration have not been correlated with postoperative wound complications.
OrthoCash 2020
-
-
When performing a cruciate-retaining (CR) total knee arthroplasty (TKA), all of the following are steps taken to recreate anatomic tension of the posterior cruciate ligament (PCL) EXCEPT:
-
Matching the anteroposterior dimension of the femoral component to the native femur
-
Ensuring full extension can be obtained through soft tissue release alone following only the most conservative distal femoral resection
-
Establishing anatomic tibial and femoral component rotation
-
Increasing the native posterior tibial slope to improve flexion and accommodate the prosthesis
-
Resecting as much tibia from the less affected side as will be replaced by the tibial component
Corrent answer: 4
All of the above are steps that are taken to recreate anatomic PCL tension in CR TKA with the exception of increasing tibial slope. Increasing the native posterior tibial slope may improve flexion but results in a looser flexion gap and as a result, an under-tensioned PCL which may contribute to flexion instability.
PCL integrity and appropriate tension are essential in CR TKA to ensure stability and restoration of the proper kinematics of the knee. Over-tensioning of the PCL may lead to increased loading and subsequent wear, while under-tensioning of the PCL can contribute to instability. Appropriate tension is best achieved by anatomic sizing of the femoral component, maintaining optimal implant rotation without excessive internal or external rotation, preserving the native joint line through conservative tibial and femoral cuts in which the bone removed is only that which will be replaced by the implants, and finally, soft tissue balancing. Soft tissue release should be performed in place of additional distal femoral resection to address a tight extension gap so as to avoid altering the kinematics of the knee by raising the joint line.
Kuriyama et al. evaluated the impact of adjustments in posterior tibial slope and femoral size on posterior cruciate ligament tension in a computer simulation of PCL-retaining TKA. The authors found that downsizing the
femoral component by 2mm decreased PCL tension by 47%, and that a 2-degree increment in posterior tibial slope similarly decreased PCL tension by 41%. The authors additionally showed that internal rotation of the femoral component increased PCL tension by 11% while external rotation of the tibial component increased PCL tension by 18%. They concluded that precise component sizing, orientation, and placement and paramount to restoring native PCL tension and knee kinematics.
Verra et al. performed a systematic review of PCL retention versus sacrifice in TKA for osteoarthritis. The authors found that there were no clinically relevant differences in range of motion, pain relief, functional or radiographic outcomes between implant designs that preserved versus those which sacrificed the PCL. There was a statistically significant difference in range of motion (2.4 degrees) and functional Knee Society Score (2.3 points) favoring PCL sacrifice, though these were deemed negligible.
Incorrect Answers:
Answer 1: The femoral component size corresponds to the anteroposterior dimension. Re-approximating the true anteroposterior femoral dimensions would best restore native PCL tension. Oversizing the femoral component would over-tension the PCL, and undersizing the component would have the inverse effect.
Answer 2: Only as much distal femur should be resected as will be replaced by the femoral component. Following conservative distal femoral resection, full extension should be attained by soft tissue release and balancing. Resection of additional distal femur will result in a loose extension gap
Answer 3: Both tibial and femoral component rotation have been found to alter the native PCL tension, most significantly with internal rotation of the femoral component and external rotation of the tibial component.
Answer 5: Resecting only as much tibia from the less affected side as will be replaced by the tibial component will ensure that the joint line is maintained and neither elevated or lowered. Changes to the joint line significantly affect the kinematics of the knee and thereby result in non-anatomic tension on the PCL.
OrthoCash 2020
-
-
Which of the following received a strong recommendation against it by the American Academy of Orthopaedic Surgeons (AAOS) in the most recently released (2nd) edition of the Clinical Practice Guideline on the Treatment of Osteoarthritis of the Knee?
-
Arthroscopic debridement in patients with a primary diagnosis of osteoarthritis of the knee
-
Arthroscopic partial meniscectomy in patients with a primary diagnosis of osteoarthritis of the knee with a torn meniscus
-
Valgus-producing high tibial osteotomy in patients with symptomatic medial compartment osteoarthritis of the knee
-
Free-floating interpositional devices in patients with symptomatic medial compartment osteoarthritis of the knee
-
None were issued as strong recommendations due to a lack of high strength evidence for or against
Corrent answer: 1
In the most recently released 2nd edition of the Clinical Practice Guideline on the Treatment of Osteoarthritis of the Knee, the AAOS issued the only STRONG recommendation AGAINST arthroscopic debridement in patients with a primary diagnosis of osteoarthritis of the knee, though none of the patients had a primary diagnosis of meniscal tear, loose body, or other mechanical arrangement.
Arthroscopic debridement in the setting of knee arthritis has long been controversial. While early studies suggested some benefit, higher quality comparative investigations have shown no significant improvement in pain or functional outcomes at any time point over continued conservative management. There is however some limited evidence to support partial meniscectomy for acute symptomatic meniscal tears in the presence of arthritis.
Moseley et al. performed a single-blinded randomized controlled trial directly comparing arthroscopic debridement, arthroscopic lavage, and placebo sham surgery of the knee for patients with a primary diagnosis of osteoarthritis of the knee. The authors found that neither the debridement nor lavage group experienced any difference in pain or function as compared to the placebo group at any time point within the first two years of surgery. They concluded that there was no discernable benefit.
Laupattarakasem et al. performed a systematic review and meta-analysis of three randomized controlled trials comparing arthroscopic debridement to either placebo or conservative interventions in patients with primary or secondary arthritis of the knees. They reported that arthroscopic debridement was not significantly different from lavage, but demonstrated inferior functional outcomes at two weeks as compared to placebo and there was no significant difference at two years. One study found that debridement was associated with less knee pain than lavage at 5 years.
Incorrect Answers:
Answer 2: The AAOS issued an INCONCLUSIVE statement, neither recommending for nor against arthroscopic partial meniscectomy in patients with a primary diagnosis of osteoarthritis of the knee with a torn meniscus, due to low-quality studies demonstrating possible benefit in patients with osteoarthritis and symptoms of an acute meniscal tear.
Answer 3: The AAOS issued a LIMITED recommendation FOR performing a valgus-producing high tibial osteotomy in patients with symptomatic medial compartment osteoarthritis of the knee due to a preponderance of low-strength case series reporting improved VAS.
Answer 4: The AAOS issued a CONSENSUS statement recommending AGAINST the use of free-floating interpositional devices due to the absence of reliable evidence and high revision rates, increased pain, and associated risks.
Answer 5: The AAOS issued a STRONG recommendation AGAINST arthroscopic debridement in patients with a primary diagnosis of osteoarthritis of the knee.
OrthoCash 2020
-
-
A 56-year-old male undergoes a total hip arthroplasty (THA). Which of the following would increase the patient's risk for dislocation or instability?
-
Acetabular cup anteversion of 16 degrees
-
Acetabular cup abduction of 45 degrees
-
High femoral offset
-
Male sex
-
Reduced femoral head to neck ratio Corrent answer: 5
There are many different factors that can lead to instability or increase the risk of dislocation of a total hip. A decreased or reduced femoral head to neck ratio is one of those risks.
Revision Total Hip Arthroplasty (THA) can be indicated for osteolysis, loosening, infection, mal-alignment, polyethylene wear, fracture, implant failure, or instability. Risk factors for instability include prior hip surgery, female, age greater than 70, posterior approach, component mal-alignment, neuromuscular disease, alcohol or drug abuse, decreased femoral offset and decreased femoral head to neck ratio. Isolated or single dislocations are typically treated with closed reduction and immobilization. Revision arthroplasty is indicated in the setting of recurrent instability usually following two or more dislocations or in the setting of component malposition.
Kluess et al. used a finite element model to predict the stability of four different femoral head sizes in variable implant positions. They found that larger femoral heads had a greater range of motion, higher resting moments, and decreased contact pressures at the egress site of the liner. They concluded that an optimal implant position and a larger femoral head can reduce dislocation rates.
Shoji et al. evaluated the CT data of 101 patients who had undergone THA using a modular implant. They found that increased offset led to increased range of motion (ROM) before bony and component contact during flexion and internal rotation, but decreased during external rotation. Additionally, they noted increasing leg length led to increased ROM in external rotation before bony and component contact. They concluded that using an appropriate long offset with a low shaft angle increases ROM in flexion and internal rotation and a high neck-shaft angle increased ROM in external rotation.
Mertl et al. looked at 145 THA patients with recurrent instability that were revised to dual mobility cups with a mean followup of 7.7 years. They noted 7 hips had dislocation of the large articulation with 2 recurrences, 2 dislocations of the small articulation requiring revision, 2 loose acetabular cups, 6 acetabular cups with possible loosening, 29 hips had acetabular or femoral osteolysis, and an 8-year survival rate of 92.6%. They concluded that dual mobility cups are an option for recurrent instability with low risk of recurrence and mechanical complications.
Illustration A is a cartoon depicting the relationship between femoral head and neck to ultimate range of motion and stability.
Video demonstrates the difference between single and dual mobility acetabular cups in reference to stability.
Incorrect Answers:
Answer 1: Normal acetabular cup anteversion is 15 degrees. Greater than 20 degrees can increase the risk of anterior dislocation.
Answer 2: Normal acetabular cup abduction is approximately 35-45 degrees. Cup abduction greater than 60 degrees can increase the risk of instability.
Answer 3: Increased femoral offset tightens the abductors and would add to stability.
Answer 4. Male sex has not been associated with an increased risk of instability.
OrthoCash 2020
-
-
A 65-year-old woman undergoes an uncomplicated right total hip arthroplasty as seen in Figure A. Preoperatively her right leg length was 2mm shorter than the left side. On the evening of surgery she has full motor strength and intact sensation. On the morning of postoperative day two she develops an inability to dorsiflex her ankle. There is no wound drainage and her anticoagulation levels are within normal limits. What is the next best step in treatment?
-
Maintain head of the bed flat with non-weight bearing restrictions
-
Obtain an EMG
-
Radiograph of the knee
-
Place the hip in flexion and the knee in extension
-
Place the hip in extension and the knee in flexion Corrent answer: 5
This patient has developed a sciatic nerve palsy of unknown origin. The hip should be placed in extension and the knee flexed to relax the nerve while reversible causes of the palsy are investigated.
Sciatic nerve palsy is the most common nerve palsy following total hip arthroplasty. Symptoms may present immediately post-op suggesting iatrogenic injury (retractor placement, cement heat, injury from cup screw placement, cerclage cabling, direct injury, indirect through excessive lengthening, "double crush" in the setting of lumbar spondylosis) or may evolve after surgery. The immediate treatment of a new nerve palsy is to limit tension on the nerve by placing the hip in extension and knee in flexion. The patient may then undergo workup for the cause of new nerve palsy.
Expeditious axial imaging may be obtained to evaluate for hematoma. Based on clinical judgment the patient may be taken for decompression of hematoma versus reversal of other causes of nerve compression.
Farrell et al. reviewed a total joint database for nerve palsies following total hip replacement. They found a preoperative diagnosis of posttraumatic arthritis or hip dysplasia, a posterior approach, lengthening of the limb, and the use of an uncemented femoral implant increased the odds ratio of a postop nerve palsy. They concluded that, although nerve palsy is uncommon, complete or incomplete palsies did not fully resolve in their series.
DeHart et al. reviewed nerve injuries and postoperative management. The authors stated that the sciatic nerve is the most commonly injured nerve, with up to 70% of cases having subclinical electrodiagnostic changes. The cause of this is multifactorial, but the vast majority of studies reviewed reported complete spontaneous recovery by 6-12 months. The authors recommend observation of the nerve deficit with ankle-foot-orthosis and follow-up EMG to determine the level of the injury.
Figure A shows an uncomplicated right total hip arthroplasty with equal leg lengths.
Incorrect Answers:
Answer 1: Restricting the patient to head of the bed flat and non-weight bearing would not relieve tension on the sciatic nerve.
Answer 2: An EMG may be indicated eventually, but the next best step is to relieve tension on the sciatic nerve.
Answer 3: An X-ray if the knee would likely be of little value as the nerve pathology likely resides around the hip joint.
Answer 4: Hip flexion and knee extension positioning would put more tension on the sciatic nerve.
OrthoCash 2020
-
-
A 56-year-old man presents with chronic anterior knee pain and the radiographs shown in Figure A. He undergoes the procedure depicted in Figure B. Regarding his prosthesis, which of the following statements is most accurate?
-
Patellofemoral arthroplasty has superior functional outcomes when compared to either medial or lateral unicompartmental arthroplasty
-
If disease progression to the medial compartment occurs, the addition of a medial UKA offers more predictable clinical outcomes than conversion to a
total knee arthroplasty (TKA)
-
The most common long-term mode of failure is progression of osteoarthritis to involve the other compartments
-
Patellar instability is the most common reason for long-term revision to TKA
-
Aseptic loosening is the most common short-term complication necessitating revision to TKA
Corrent answer: 3
This patient underwent an onlay-design unicompartmental patellofemoral arthroplasty (PFA) for isolated patellofemoral arthritis. The most common long-term mode of failure is progression of osteoarthritis in the tibiofemoral compartments.
The ideal candidate for PFA has isolated, non-inflammatory patellofemoral compartment arthritis. While the original PFA had high complication rates from patellar instability secondary to inlay-style trochlear prosthesis design, the more recent onlay-design trochlear prostheses in PFA replaces the entire anterior trochlear surface and has significantly decreased patellar instability. In the long-term, even when patella tracking is satisfactory after PFA, the primary mode of failure is progressive tibiofemoral arthritis, irrespective of the type of trochlear prosthesis used.
Lonner et al. published a paper evaluating the clinical outcome of PFA. They reported that there has been a disparity in the early and mid-term failures that occur after PFA as a result of patellar instability and maltracking, depending on whether an inlay-style or onlay-style component is used, with significant rates of early failure from patellar instability with the inlay design. They concluded that decreasing patellar instability risk with the contemporary onlay-design PFA has improved long-term, leaving progressive tibiofemoral arthritis as the primary failure mechanism beyond 10 to 15 years.
Kooijman et al. conducted a retrospective study to assess the long-term results of PFA. They reported excellent or good results in 86% of cases at mean follow-up of 17 years, a 3.6% rate of tibiofemoral arthritis and a 2% rate of loosening. Thus they concluded that PFA is a reliable treatment option in middle-aged patients with isolated patellofemoral osteoarthritis.
Figure A is an AP, lateral, and sunrise view of the right knee depicting isolated patellofemoral osteoarthritis. Figure B depicts the onlay-design PFA, which replaces the entire anterior trochlear surface. Illustration A depicts an example of an inlay-design PFA, which attempted to position the prosthesis flush with the surrounding trochlear articular cartilage.
Incorrect Answers:
Answer 1: There are no current studies demonstrating that PFA results in superior outcomes as compared to either medial or lateral unicompartmental arthroplasty.
Answer 2: If disease progression to the medial compartment occurs, conversion to a TKA offers the most reliable clinical outcomes as compared to the addition of a medial UKA.
Answer 4: While the most common short-term mode of failure for the inlay-design PFA was patellar instability, this has decreased substantially with the contemporary onlay-design. Progression of arthritis, and NOT patellofemoral instability, is the most common reason for long-term revision.
Answer 5: Aseptic loosening is not a short-term complication and is much less often a reason for revision than progression of tibiofemoral arthritis.
OrthoCash 2020
-
-
A 62-year-old man undergoes total hip arthroplasty 6 months ago. He is now presenting with groin pain, which is exacerbated when he is going up the stairs, but not down the stairs. He has pain and significant weakness with resisted hip flexion. He is afebrile and laboratory markers are unrevealing for infection. His recent radiographs are depicted in Figures A and B. Which of the structures depicted in the representative image in Figure C (labeled 1-5) is responsible for his pain?
1. 1
2. 2
-
3
-
4
-
5
Corrent answer: 1
This patient underwent a total hip arthroplasty (THA) and is now experiencing groin pain, which is exacerbated with hip flexion (i.e. ascending stairs). This is classic for iliopsoas tendonitis. The iliopsoas tendon is labeled "1" in Figure C.
Iliopsoas tendinitis after THA is a rare cause of groin pain that has been associated with activities requiring flexion. The symptoms may include groin pain with an associated snapping sensation and/or palpable mass. This condition may be associated with acetabular component malposition, where the anterior edge of the acetabulum protrudes beyond the anterior wall. While nonoperative management including rest, NSAIDs, and stretching has been recommended for the treatment of this condition, a lidocaine injection to the iliopsoas tendon may be both diagnostic and therapeutic. Recalcitrant cases may require iliopsoas tenotomy and/or acetabular component revision in cases of component malposition.
Lachiewicz et al. reviewed anterior iliopsoas impingement and tendonitis after THA. They reported that the diagnosis of iliopsoas tendonitis is confirmed by imaging studies (either cross-table lateral radiograph, computed tomography, magnetic resonance, or ultrasonography) in combination with a confirmatory lidocaine diagnostic injection. They concluded that although a trial of nonoperative treatment is feasible, it is not likely to provide permanent relief of symptoms and often requires the release of the iliopsoas tendon, alone or in combination with acetabular component revision, especially in the setting of an anterior overhanging component.
O'Sullivan et al. investigated iliopsoas tendonitis as a complication after THA. They reported that of iliopsoas tenotomy from the lesser trochanter resulted in symptomatic relief in all but one patient who required reposition of the acetabular component. They concluded that iliopsoas tendonitis should be considered in the differential for all patients who present with groin pain after THA, and recommended tenotomy as a viable option in recalcitrant cases.
Figures A and B depict radiographs after total hip arthroplasty with evidence of anterior overhang of the acetabular component on the cross-table lateral radiograph. Figure C is a T2 axial MRI representative image of the hip at the level of the femoral head (1=iliopsoas tendon; 2=rectus femoris tendon; 3=femoral vein; 4=femoral artery; 5=anterior wall of the acetabulum).
Incorrect Answers:
Answer 2: The structure labeled 2 is the rectus femoris tendon. Answer 3: The structure labeled 3 is the femoral artery.
Answer 4: The structure labeled 4 is the femoral vein.
Answer 5: The structure labeled 5 is the anterior wall of the acetabulum.
OrthoCash 2020
-
-
A 67-year-old woman presents with chronic groin pain 5 months after undergoing total hip arthroplasty. A representative native MRI (Figure A) shows inflammation in the structure marked with the red arrow. What is the most likely cause of inflammation of this structure in this patient?
-
Excessive femoral offset
-
Decreased limb length compared to the contralateral side
-
Increased limb length compared to the contralateral side
-
Anterior overhang of the acetabular component
-
Excessive anteversion of the acetabular component Corrent answer: 4
The structure marked with the red arrow in Figure A is the iliopsoas tendon. Iliopsoas tendonitis after total hip arthroplasty (THA) would most likely be caused by anterior overhang of the acetabular component.
Iliopsoas tendonitis after THA is a rare cause of groin pain that has been associated with activities requiring hip flexion. This diagnosis may be associated with acetabular prosthesis malposition, where the anterior edge of the acetabulum protrudes beyond the anterior wall. Nonoperative management includes rest, NSAIDs, and stretching as first line. A lidocaine injection to the iliopsoas tendon may be both diagnostic and therapeutic.
Recalcitrant cases may require iliopsoas tenotomy and/or acetabular component revision in cases of component malposition.
Lachiewicz et al. reviewed iliopsoas impingement after THA. They reported that the diagnosis can be confirmed by imaging studies, including a cross-table
lateral radiograph, computed tomography, magnetic resonance imaging, and ultrasonography, in combination with a diagnostic and often therapeutic iliopsoas tendon injection. They concluded that treatment often consists of iliopsoas tendon release in isolation or in combination with acetabular revision for an anterior overhanging component.
O'Sullivan et al. investigated a series of patients with iliopsoas tendonitis after THA. They reported that release of the iliopsoas tendon from the lesser trochanter resulted in adequate relief in all except one patient who required revision of the acetabular component. They emphasized the importance of this diagnosis in the differential diagnoses of all patients who present with groin pain after THA, and recommended surgical release of the iliopsoas tendon as an excellent option in these patients.
Figure A is an axial MRI of a native pelvis with the iliopsoas tendon marked by the red arrow. Illustration A is a CT demonstrating anterior acetabular overhang.
Incorrect Answers:
Answer 1: Increased femoral offset can lead to trochanteric bursitis, not iliopsoas tendonitis.
Answers 2 and 3: Limb length discrepancy is not a significant risk factor for iliopsoas tendonitis.
Answer 5: Excessive acetabular component retroversion (not anteversion) results in anterior overhang, which can lead to iliopsoas tendonitis.
OrthoCash 2020
-
-
A 34-year-old female presents with chronic insidiously progressive left hip pain. The physical exam is significant for 15 degrees of internal rotation with the hip in 90 degrees of flexion and a positive flexion-internal rotation impingement sign. Radiographs show a Tonnis angle of 15 degrees and a lateral center-edge angle of 15 degrees. MRI is significant for an anterosuperior labral tear. Which of the following is true of patients who undergo hip arthroscopy in the presence of this pathology as compared to those without?
-
Decelerated progression of arthritis
-
Decreased femoral head subluxation
-
Decreased risk of reoperation
-
Greater failure rate
-
Greater functional improvement Corrent answer: 4
The patient in this vignette has symptomatic adult hip dysplasia. Patients with acetabular dysplasia who undergo arthroscopic procedures of the hip have been found to have significantly greater rates of surgical failure as compared to patients undergoing arthroscopic procedures without hip dysplasia.
Adult hip dysplasia has been found to be present in roughly 13% of patients undergoing hip arthroscopy and is most often borderline or mild. Arthroscopic management may be considered in borderline or mild dysplasia. If arthroscopic management is undertaken, labral repair and capsular plication have been shown to improve functional outcomes. However hip arthroscopy in patients with acetabular dysplasia has been shown to be associated with accelerated progression of arthritis, hip migration, comparably inferior functional outcomes as well as increased reoperation and surgical failure. As a result, a periacetabular osteotomy should be considered in patients with moderate to severe acetabular dysplasia and maintained acetabular cartilage (joint space).
Byrd et al. evaluated the results of operative hip arthroscopy in patients with hip dysplasia. At an average 27-month follow-up, the authors found that functional outcomes were comparable to those of the general population, though two-thirds were classified only as “borderline” dysplasia. They concluded that the response to treatment correlated more with the hip pathology than the presence of dysplasia.
Parvizi et al. reviewed the outcomes following arthroscopic management of labral tears in a population of patients with hip dysplasia. The authors found that arthroscopic management failed to relieve pain in just over 70% of
patients, while accelerated arthritis and progressive migration of the femoral head was observed in 41% and 38% of patients, respectively. Additionally, nearly one-half (47%) of patients required reoperation including PAO, revision arthroscopy, or THA. The authors cautioned that patients with acetabular dysplasia may not benefit from arthroscopic management of labral pathology.
Matsuda et al. performed a comparative case series evaluating the presence of hip dysplasia in patients undergoing hip arthroscopy. The authors found that the majority of patients with dysplasia had borderline or mild severity, and reported an increased incidence of hypertrophic labrum in this population.
They conclude that the most common procedures performed in the population were labral repair, femoroplasty, and capsular closure.
Larson et al. compared the outcomes following arthroscopic management of intra-articular pathology in patients with hip dysplasia versus femoroacetabular impingement (FAI). They showed that patients exhibited significantly less improvement in functional outcomes and a significantly greater failure rate (32% versus 11%, respectively). They concluded that arthroscopic management of patients with mild to moderate acetabular dysplasia should be undertaken with caution and that labral repair and capsular plication should be performed.
Incorrect answers:
Answer 1: Accelerated arthritis is observed in nearly one-third of patients with acetabular dysplasia who undergo arthroscopic management of labral tear.
Answer 2: Progressive femoral head subluxation and migration has been found to be significantly increased in patients with acetabular dysplasia who undergo arthroscopic management of labral tear.
Answer 3: Reoperation has been reported in nearly one-half of patients with acetabular dysplasia who undergo arthroscopic hip procedures.
Answer 5: Improvement in functional outcomes has been shown to be at best comparable to patients with FAI in the presence of borderline or mild dysplasia and significantly worse in the presence of moderate to severe dysplasia.
OrthoCash 2020
-
-
A 45-year-old female is referred to you for evaluation of her right hip. Five years ago, she was involved in a motor vehicle collision and sustained a femoral head fracture that was treated with a hip surface replacement arthroplasty. Her post-operative course was remarkable and she didn't have any issues until 4 months ago when she noticed subtle hip and groin pain. She is otherwise healthy and wants to begin training for a marathon but is concerned about her hip due to
television ads about metal-on-metal devices. Her radiographs show a well-placed implant and are otherwise unremarkable. What test would best evaluate for the most concerning complication from this implant?
-
CT including the pelvis, hip, and femur
-
Whole body triple-phase bone scan
-
MRI with metal artifact reduction sequence
-
Serum cobalt and chromium levels
-
Serum cobalt, chromium, molybdenum, and nickel levels Corrent answer: 3
This patient has a painful metal-on-metal hip, which requires a metal artifact reduction sequence (MARS) MRI to evaluate for pseudotumor formation and soft tissue destruction.
Metal on metal hip arthroplasties have become a burden for both surgeons and patients. As with traditional arthroplasty wear which causes osteolysis, reaction to metal-on-metal debris also causes soft tissue destruction. In the setting of a painful metal-on-metal hip arthroplasty, infection and fracture should always be ruled out, followed by MRI to evaluate for soft tissue fluid collections or masses termed pseudotumors. MARS MRI best correlates with prognosis in the setting of metal debris. Serum metal ion levels can be followed to monitor the amount of metal debris but have little correlation with fluid collection or soft tissue destruction.
Kwon discussed use of MARS MRI in MoM hip arthroplasty. The MARS suppresses implant metal artifact to better define the soft tissues.
Characteristic signs of MoM reaction are cysts and masses (collectively termed pseudotumors), tendon avulsions, and bone edema. New MRI protocols (i.e.
MAVRIC and SEMAC) have been developed since the adoption of MARS, but they compromise image quality and are difficult to use in pre-operative planning.
Kwon et al. formed a consensus statement on the diagnostic approach to the patient with concern for metal debris from modular taper wear. They warn against relying on one clinical test when investigating the painful hip and emphasize the use of MARS MRI for detecting soft tissue lesions.
Jacobs et al. reviewed the utility of serum metal ion levels monitoring. They note these levels can remain high up to 3 years following surgery, and there was little variation in total level amount between MoM THAs and resurfacing arthroplasties. Additionally, altered renal function can falsely elevate metal ion levels due to poor excretion.
Illustrations A and B are T2 MARS MRIs showing different appearances of pseudotumors, with the first being completely cystic while the second has a heterogeneous appearance.
Incorrect Answers:
Answer 1: CT is best used in the setting of periprosthetic bone loss. Answer 2: Bone scans are non-specific for many processes, including pseudotumor formation.
Answers 4 and 5: Metal ion levels may be used in surveillance for metal wear but aren't useful in the setting of a painful prosthesis.
OrthoCash 2020
-
-
Which of the following is an absolute indication for constrained nonhinged total knee arthroplasty (TKA)?
-
Conversion TKA from knee arthrodesis
-
Hyperextension instability
-
Repairable intra-operative MCL injury
-
Revision of medial unicondylar knee arthroplasty (UKA) to TKA for aseptic loosening
-
Varus deformity >20 degrees with LCL incompetency Corrent answer: 5
Severe coronal deformity with collateral ligament incompetency is an indication for a constrained nonhinged prosthesis.
While more often indicated in the revision setting, varus-valgus or condylar constrained total prostheses may be necessary in the complex primary knee. Indications for use of a constrained nonhinged prosthesis include severe varus/valgus deformity with MCL/LCL incompetency, severe bone loss, inability to balance flexion-extension gaps due to severe flexion contracture, persistent varus-valgus laxity despite adequate releases, neuropathic arthropathy, and post-polio sequelae. Intra-operative MCL injury may be addressed with primary repair and retention of an unconstrained prosthesis or conversion to a constrained design if irreparable.
Natoli et al. evaluated the outcomes following TKA in 8 knees with severe tibia vara (average meta-diaphyseal angle >20 degrees). The authors noted significant posteromedial tibial defects and the need for extensive medial release in most cases. Though only two knees required constrained prosthesis at the index procedure, two knees required revision to rotating hinge prosthesis by the time of final follow-up, leading the authors to conclude that constrained prosthesis should be considered in the initial procedure.
Kernkamp performed a systematic review evaluating the outcomes following conversion of knee arthrodesis to TKA. The authors found that most knees (66% of those for which implant was reported) were successfully converted to standard posterior stabilized implants. The majority of cases (65%) were associated with a complicated post-operative course, often secondary to wound complications, with 11% necessitating revision. Despite this, the authors concluded that conversion to TKA was associated with improved functional outcomes scores and reliable clinical results in lieu of increased post-operative complications.
Incorrect answers:
Answer 1: While varus-valgus constrained or even hinged prosthesis may be necessary depending on the degree of bony resection and technique for knee arthrodesis, posterior stabilized prosthesis may often be used if an adequate
soft tissue envelope is preserved.
Answer 2: Hyperextension instability necessitates a constrained hinged prosthesis.
Answer 3: Intra-operative MCL injury may be treated with MCL repair and retention of a non-constrained prostheses. If the MCL is irreparable, a constrained unhinged prosthesis is required.
Answer 4: Though revision of a medial unicondylar knee arthroplasty often necessitates a stemmed tibial prosthesis with a medial augment, a non-constrained design may be sufficient at achieving adequate stability. Varus-valgus constraint may be required if there is significant medial bone loss with MCL insufficiency.
OrthoCash 2020
-
-
A 75-year-old female presents with chronic severe and progressively worsening right knee pain. Her symptoms have been present for nearly a decade, and she has failed an appropriate course of conservative management but remains persistently symptomatic. Radiographs are shown in Figures A through C. The patient finally elects to have surgery and receives a posterior-stabilized (PS) total knee arthroplasty (TKA). The surgeon prefers an intramedullary femoral guide, posterior referencing, and an intra-medullary tibial guide. The surgeon should pay special attention to which of the following due to the nature of the deformity and preferred surgical technique?
-
Angulation of the femoral component
-
Exposure of the patella and posterolateral corner
-
Flexion of the femoral component
-
Rotation of the femoral component
-
Rotation of the tibial component
Corrent answer: 4
Valgus deformities of the knee often involve hypoplasia of the lateral femoral condyle. As a result, exclusive reliance on a posterior referencing guide may result in excessive internal rotation of the femoral component.
TKA in the valgus knee presents a unique set of challenges with regard to establishing optimal component alignment and positioning as well as a balanced, stable joint. Whereas varus deformities are often tibial-driven, valgus deformities typically result from hypoplasia of the lateral femoral condyle, though this may progress to eventual wear of the posterolateral tibial plateau. Furthermore, due to the nature of the deformity, the lateral capsuloligamentous structures are typically contracted, while the medial structures are attenuated and sometimes deficient. As a result, the extent of soft tissue release and attenuation may necessitate increased constraint.
Whiteside et al. evaluated the utility of the anteroposterior axis of the distal femur compared to the posterior condylar axis in establishing the rotational alignment of the distal femoral component in valgus knees. The authors found that posterior referencing resulted in significant internal rotation of the femoral component, resulting in a high rate of lateral patellar instability. The knees which referenced the anteroposterior axis had a significantly lower risk of patellar maltracking.
Favorito et al. present a comprehensive review of the challenges associated with performing TKA in the valgus knee. The authors emphasize that the rotation of the femoral component must be checked with the epicondylar axis and Whiteside’s line to avoid over-resection of the posterolateral femoral condyle and internal rotation of the femoral component. This would in turn lead to asymmetry of the flexion gap as well as patellar maltracking. They conclude that careful attention to this is essential to producing a stable and balanced joint.
Figures:
Figure A is a standing hip-to-ankle alignment film demonstrated a valgus deformity.
Figure B is an AP radiograph of a knee with valgus osteoarthritis.
Figure C is a lateral radiograph of the same knee showing no excessive wear of the posterolateral tibia.
Incorrect answers:
Answer 1: The resection of the lateral aspect of the distal femur will be more limited due to the existing femoral valgus deformity. However, the risk of placing the femoral component in excessive valgus is not increased with
proper use of the intramedullary guide.
Answer 2: In a valgus knee, exposure of the patella and the posterolateral corner is often facilitated by the deformity.
Answer 3: Flexion of the femoral component is determined by the entry point and trajectory of the intramedullary guide. The degree of flexion of the femoral component is not specifically influenced by the valgus deformity.
Answer 5: Rotation of the tibial component based off the tibial tubercle is generally not altered significantly in a valgus knee.
OrthoCash 2020
-
-
A 67 year-old female presents for evaluation of her right knee. She reports that is had become gradually more painful over the past 3 years and injections and therapy provide no relief. She would like to proceed with total knee arthroplasty. Utilizing posterior femoral condylar referencing in this patient increases which intraoperative error if not accounted for?
-
External rotation of the femoral component
-
External rotation of the tibial component
-
Internal rotation of the femoral component
-
External rotation of the tibial component
-
Postoperative patellar fracture Corrent answer: 3
This patient demonstrates a valgus knee with advanced osteoarthritis and a hypoplastic lateral femoral condyle. Posterior referencing places this patient at an increased risk of internal rotation of the femoral component.
There are three reference axis that may be used for the femoral prosthesis. These are the anteroposterior axis, transepicondylar axis, and the posterior condylar axis. The posterior condylar axis is an average of 3 degrees of internal rotation compared to the transepicondylar axis. Typically, 3 degrees of external rotation is set within the guide to compensate for this disparity.
However, more than 3 degrees of internal rotation is often observed in patients with a hypoplastic lateral condyle. If this is not accounted for when utilizing posterior referencing, excessive internal rotation of the femoral component will result.
Whiteside et al. evaluated utilization of the anteroposterior axis of the distal femur, rather than the transepicondylar or posterior femoral condylar axis, to establish rotational alignment of the femoral component in valgus knees. They found that in 107 valgus knees using the anteroposterior axis for rotational alignment of the femoral component, patellar tracking problems that required realignment were significantly reduced. They concluded that this referencing technique is useful and helps to prevent patellar instability.
Favorito et al. reviewed the valgus knee and its unique set of problems that must be addressed during total knee arthroplasty. They report that both bone and soft-tissue deformities complicate restoration of proper alignment, positioning of components, and attainment of joint stability. They concluded that lateral femoral condyle deficiency needs to be addressed and that understanding the specific pathologic anatomy associated with the valgus knee is a prerequisite to selecting the proper surgical method to optimize component position and restore soft-tissue balance.
Figure A is the AP radiograph of the right knee demonstrating a valgus knee with advanced osteoarthritis and a hypoplastic lateral femoral condyle
Incorrect Answers:
Answers 1,2,4: Failure to recognize a hypoplastic femoral condyle increases the error of placing the femoral component in too much internal rotation Answer 5: Patient is at increased risk of patellar instability with a valgus knee
OrthoCash 2020
-
-
Figure A is the radiograph of a 61-year-old female who is undergoing revision TKA. To aid in exposure, you perform a tibial tubercle osteotomy. What factor on her pre-operative radiograph could have predicted the need for this osteotomy?
-
Well-fixed tibial component
-
Patella baja
-
Neutral tibial slope
-
Cruciate retaining design
-
Bone mineral density Corrent answer: 2
This patient has significant patella baja on her preoperative radiographs, which can make exposure much more difficult. Tibial tubercle osteotomies can be performed to safely mobilize the extensor mechanism to improve exposure.
Gaining adequate surgical exposure in the setting of revision TKA is one of the most important aspects of the surgery. Several techniques for achieving this
exist including quadriceps snip, quadriceps V-Y turndown and the tibial tubercle osteotomy (TTO). In patients with patella baja, well fixed tibial stems, or significant scaring on the anteromedial tibia, the TTO is the most beneficial. TTO consists of making an 8-10cm osteotomy along the medial tibial tubercle and hinging the osteotomized bone on the lateral periosteum to safely mobilize the extensor mechanism. Additionally, proximal fixation of the osteotomy fragment can be performed during repair helping to reduce patellar baja and quadriceps tension.
Ries et al reviewed the outcomes of their patients who underwent tibial tubercle osteotomies. In 7/30 knees the tubercle was recessed 1-2cm proximally to assist in treating patellar baja. They report 29/30 osteotomies healing primarily without difficulty, and conclude that TTOs are a safe and effective way to improve exposure during TKA.
Della Valle et al review the different surgical exposures for revision TKA. They report that a V-Y quadricepsplasty or a TTO can be used to obtain increased exposure in particularly difficult knees, including those with patellar baja. They conclude that the TTO has stronger healing potential and is their preferred extensile exposure.
Whiteside et al outline their experience with using a tibial tubercle osteotomy for difficult to expose TKA. They report that following TTO, adequate exposure was achieved in all patients and no further release of the quadriceps mechanism was needed. They conclude that this is an effective technique for gaining exposure in both primary and revision TKA.
Figure A is the lateral radiograph of a patient with a well fixed TKA demonstrating patellar baja.
Incorrect Answers:
Answer 1: TTOs are commonly used to assist in removing long stemmed tibial implants, sleeves or cones, however, they are not typically necessary for the removal of well-fixed standard tibial baseplates.
Answer 3,4: Tibial slope and standard CR design would not predict the need for a TTO preoperatively.
Answer 5: While low bone mineral density may be a contraindication to TTO it would not predict whether or not one is needed.
OrthoCash 2020
-
-
Figure A is the AP radiograph of a 42-year-old male construction worker presenting with medial-sided knee pain. He reports a history of subtotal medial meniscectomy 12 years ago. He has failed conservative management including physical therapy and injections. He had initial success with the use of a medial unloader brace, however it no longer provides relief. The patient has a BMI of 24. On examination, he has knee range of motion from 8°-130° of flexion, negative Lachman, and has a varus-thrust during ambulation. Which of the following is a contraindication to a valgus-producing high tibial osteotomy in this patient?
-
Age
-
BMI
-
Lack of full knee extension
-
Medial compartment arthritis
-
Varus thrust with ambulation Corrent answer: 5
This patient has postmeniscectomy medial compartment arthritis. A varus-thrust during gait is a contraindication to perform a valgus-producing high tibial osteotomy (VPHTO).
A VPHTO is appropriate in this patient given his young age, high-functional
occupation, physical examination, radiographic findings, and initial response to medial unloader bracing. Technical considerations for a VPHTO include a medial opening-wedge, a lateral closing-wedge osteotomy, or a focal dome osteotomy. A lateral closing wedge osteotomy has faster rehabilitation and weight bearing. A medial opening-wedge osteotomy maintains posterior slope while avoiding the peroneal nerve and proximal tibiofibular joint, but should be avoided in patients who smoke because of concern for nonunion. A varus thrust gait has been shown to be predictive of medial OA progression and produces joint laxity laterally which causes the entire force of the knee to be transmitted to the medial compartment, placing the medial compartment at greater risk for breakdown.
Sprenger et al. retrospectively reviewed the long-term results of 76 medial compartment gonarthrosis treated with a VPHTO. They found that survival at ten years was 90% when the radiographic valgus angle at one year was between 8° and 16° with arthroplasty as the end point. They conclude that there is a role for tibial osteotomy, as an alternative to total knee arthroplasty, in patients who are less than sixty years old.
Rossi et al. reviewed the use high tibial osteotomy. They report that medial compartment overload following meniscectomy is an indication for VPHTO. They conclude that its use is commonly accepted to correct alignment and achieving durable results.
Figure A is the weight-bearing AP radiograph of the right knee demonstrating isolated medial compartment arthritis.
Incorrect Answers:
Answer 1: An age of 42 years is an indication for VPHTO. A VPHTO serves as an alternative to total knee arthroplasty in patients who are less than sixty years old
Answer 2: This patient has a normal BMI which is not a contraindication to a VPHTO
Answer 3: Limitations in knee range of motion may be a contraindication to a VPHTO. Generally, a flexion contracture >15° or knee flexion <90° are contraindications to a VPTHO
Answer 4: Isolated medial compartment arthritis in a young, active patient is an indication for a VPHTO
OrthoCash 2020
-
-
Figure A is the AP radiograph of a 38-year-old male firefighter presenting with medial-sided knee pain. Past surgical history includes a subtotal medial meniscectomy 10 years ago. He has failed conservative management and the decision is made to proceed with a valgus-producing high tibial osteotomy. He is a pack-a-day smoker. He has a BMI of 30. On examination, he has knee range of motion from 10°-128° of flexion, and negative Lachman. Which of the following would favor the use of a lateral closing wedge valgus-producing high tibial osteotomy compared to a medial opening wedge valgus-producing high tibial osteotomy in this patient?
-
Age
-
Amount of medial compartment arthritis
-
BMI
-
History of subtotal medial meniscectomy
-
Smoking history
Corrent answer: 5
This patient has postmeniscectomy medial compartment arthritis. A medial opening-wedge valgus-producing high tibial osteotomy (VPHTO) is contraindicated in a smoker because of concern for nonunion.
A VPHTO is an appropriate treatment modality in this patient given his young
age, high-functional occupation, physical examination, and radiographic findings. Technical considerations for a VPHTO include a medial opening-wedge and a lateral closing-wedge osteotomy. A lateral closing wedge osteotomy is the most commonly performed technique. A medial opening-wedge osteotomy maintains posterior slope, avoids the peroneal nerve and proximal tibiofibular joint, but should be avoided in patients who smoke because of concern for nonunion. Therefore, a current smoking history influences the technique utilized during a VPHTO.
W-Dahl et al. reviewed the influence of smoking on 200 patients (34 smokers and 166 nonsmokers) who underwent a hemicallotasis opening wedge osteotomy in the proximal tibia for deformities of the knee. They report that 50% of the smokers and 20% of the nonsmokers developed complications.
Smokers required an average of 16 days more in external fixation and delayed healing and pseudoarthrosis were more common in smokers . They concluded that the risk ratio for smokers to develop complications was 2.5, as compared to nonsmokers.
Sprenger et al. retrospectively reviewed the long-term results of 76 medial compartment gonarthrosis treated with a VPHTO. They found that survival at ten years was 90% when the radiographic valgus angle at one year was between 8° and 16° with arthroplasty as the end point. They conclude that there is a role for tibial osteotomy, as an alternative to total knee arthroplasty, in patients who are less than sixty years old.
Figure A is the weight-bearing AP radiograph of the knee demonstrating isolated medial compartment arthritis.
Incorrect Answers:
Answer 1: His age of 38 years is an indication for HTO. It does not influence the technique utilized for a VPHTO.
Answer 2: Isolated medial compartment arthritis in a young, active patient is an indication for a VPHTO. It may be treated with either a medial opening-wedge or a lateral closing-wedge osteotomy.
Answer 3: BMI does not influence the technique utilized for a VPHTO. Answer 4: This patient has postmeniscectomy medial compartment arthritis which is why a VPHTO is being considered in this young, active patient.
OrthoCash 2020
-
-
Which of the following most accurately lists the relative risk of periprosthetic joint infection (PJI) in total joint arthroplasty in order
from lowest to highest risk?
-
Primary hip < primary knee < revision knee < revision hip
-
Primary hip < primary knee < revision hip < revision knee
-
Primary hip < revision hip < primary knee < revision knee
-
Primary knee < revision knee < primary hip < revision hip
-
Primary knee < primary hip < revision knee < revision hip Corrent answer: 2
The risk of PJI is lower among primary joint replacements and lower for total hip arthroplasty (THA) as compared to total knee arthroplasty (TKA).
The majority of PJI occur within the first 1-2 years post-operatively. In general, the risk for PJI is lowest in uncomplicated primary joints. Most studies support the lowest risk of PJI among primary THA (0.3-1.3%), followed by primary TKA (1-3%). TKA is thought to be associated with a greater risk of PJI due to the increased mobility and more superficial nature of the knee compared to the hip. Revision joint further conveys an increased risk of PJI, more so for revision TKA (5-10%) than revision THA (3-5%). Revision for aseptic loosening is perhaps unsurprisingly associated with a much lower risk for PJI than revision for infection.
Pulido et al. reviewed the incidence of prosthetic joint infection in a series of over nine thousand primary total joints performed between 2001 and 2006. The authors found that periprosthetic joint infections developed in 0.7% of cases, attributable to 1.1% in primary total knee and 0.3% in primary total hip arthroplasty cases. Infection occurred in 65% of cases within one year of index arthroplasty. The authors found that this was increased in the setting of bilateral simultaneous arthroplasty.
Mortazavi et al. evaluated the incidence and predictors of infection following revision total knee arthroplasty. The authors noted that the risk was nearly 10-fold higher in patients undergoing revision (9%) versus primary TKA. They
additionally showed that aseptic revisions were at a substantially lower risk for PJI (5%) than revision for infection (21%).
Incorrect answers:
Answer 1: Revision TKA is associated with the greatest risk of PJI, followed by revision THA, primary TKA, and finally primary THA with the lowest risk.
Answer 3: Primary TKA is associated with a lower risk of PJI than revision TKA. Answer 4: Primary THA is associated with the lowest risk of PJI.
Answer 5: Revision TKA is associated with the greatest risk of PJI.
OrthoCash 2020
-
-
A 62-year-old woman presents to your clinic with knee pain and swelling 4 months after a primary TKA. Her initial recovery was uneventful, but she had a small pustule develop 6 weeks after surgery. An emergency room physician gave her 10 days of oral antibiotics for a "suture abscess" at that time. Since the ED visit, she notes worsening pain and persistent drainage. Figurs A represents a clinical photo from today's appointment. What is the next best step in management?
-
Admit directly from clinic for I&D and polyethylene exchange
-
Prescribe oral antibiotics and follow up in 2 weeks
-
Aspirate the patient's knee and plan for surgery
-
Prescribe home health wound care
-
Placement of a knee immobilizer and hold physical therapy for 2 weeks Corrent answer: 3
This patient likely has a prosthetic knee infection with a draining sinus. Using the MSIS criteria a sinus that communicates with the joint represents a major
infectious criteria defining PJI. Given the chronic nature of the infection, a two-stage revision is recommended. Aspiration can be performed in an attempt to isolate the organism preoperatively as well as to confirm intra-articular involvement.
All prosthetic joint infections require surgical debridement if the patient is healthy enough for surgery. Additionally, exchange of the components is indicated if the infection is chronic (> 4 weeks from surgery or from the onset of acute hematogenous spread), as chronic infections are thought to include bacterial invasion of the bone-prosthesis interface and a simple I&D and polyethylene exchange is less likely to address this or disrupt the likely biofilm presence. A two-stage revision with interval placement of an antibiotic-eluting spacer and treatment for osteomyelitis with 6 weeks of IV antibiotics is the gold standard for chronic infections in patients healthy enough to undergo multiple surgeries. One-stage exchange arthroplasty has a growing body of evidence, and in certain patient groups and organism types, has shown promising results.
Parvizi et al. reviewed the literature on diagnostic strategies for assessing periprosthetic joint infections and compared these data to the findings from a single large-volume institution. Joint fluid aspirates with high cell counts and a high percentage of neutrophils, as well as positive FDG-PET scans have high sensitivities for diagnosing periprosthetic infections. While properly performed aspirates and intraoperative cultures have near perfect specificities (0.97 and 1.0, respectively), they found a 10-14% false negative rate. They go on to describe their institutional diagnostic protocol defining numerical cutoffs based on predictive value thresholds.
Koyonos et al. performed a single-institution retrospective review and found that irrigation and debridement alone was an ineffective way to definitively treat periprosthetic joint infections for acute post-op infections (< 4 weeks from surgery, 69% failure), acute delayed infections (> 4 weeks from surgery with acute onset of symptoms, 56% failure), and chronic infections (> 4 wks from onset of symptoms, 72% failure).
Figure A shows an open area of drainage over a standard midline TKA incision. Incorrect Answers:
Answers 1: An isolated I&D would have a low chance of success given the
duration of symptoms > 4 weeks.
Answer 2: The patient's periprosthetic infection would be best treated with surgical debridement and IV antibiotics. Oral antibiotics can cloud diagnostic tests and reduce culture yields and therefore should be held preoperatively.
Answers 4 and 5: Home health wound care or placement of a knee immobilizer would not treat the underlying problem in this case.
OrthoCash 2020
-
-
Compared to historical causes of revision after total knee replacement which of the following statements is most accurate?
-
Infection is now the most frequent cause for late revision
-
Polyethylene wear is no longer the major cause for revision
-
Aseptic loosening is now the most frequent cause for early revision
-
The percentage of revisions for instability and malalignment has increased
-
Stiffness is an uncommon reason for revision procedures Corrent answer: 2
The use of highly-crosslinked ultra-high molecular weight polyethylene has decreased the revision rate for wear after total knee arthroplasty (TKA) over the past 2 decades.
The most common causes for failed primary TKA are aseptic loosening, infection, instability, periprosthetic fracture, and arthrofibrosis. Infection is considered the most common indication for early revision (< 2 years from index procedure) and aseptic loosening is the most common reason for late revision (>2 years). The use of highly crosslinked polyethylene inserts has shown to significantly reduce the complications from polyethylene (PE) wear.
Sharkey et al. analyzed the indication for revision following a primary TKA. The reasons for failure (in order of prevalence among the patients in this study) included polyethylene wear, aseptic loosening, instability, infection, arthrofibrosis, malalignment or malposition, deficient extensor mechanism, avascular necrosis in the patella, periprosthetic fracture, and isolated patellar resurfacing.
Sharkey et al. published a follow-up study that analyzed the indications for revision 10 years following their index publication. They found that revision for polyethylene wear was no longer the most common indication for revision.
They noted that infection was the most common reason for early revision and that aseptic loosening was the most common indication for late revision. They also note that the percentage of revisions for instability and malalignment has decreased.
Pitta et al. similarly analyzed failure after primary TKA. The most common reasons for revision within 2 years were infection and stiffness. Risk factors for
revision included a history of drug abuse, constrained implants, younger age, and pre-operative trauma or deformity.
Illustration A is an example of osteolysis around a TKA from polyethylene wear Incorrect Answers:
Answer 1: Infection is the most common cause for early revision (<2 years).
Answer 3: Aseptic loosening is the most common cause for late revision (>2 years).
Answer 4: The percentage of revisions for instability and malalignment has decreased.
Answer 5: Stiffness is a common reason for early revision.
OrthoCash 2020
-
-
When performing a surgical dislocation of the hip, the inferior gluteal artery should be preserved. Where can this artery reliably be found?
-
Along the border of the piriformis tendon
-
Between the pectineus and iliopsoas
-
Beneath the tensor fasciae latae
-
Underneath the gluteus medius
-
Between the quadratus femoris and upper border of the adductor magnus Corrent answer: 1
The inferior gluteal artery (IGA) may reliably be found along the border of the piriformis tendon.
In a majority of hips, the main blood supply of the femoral head is from the medial femoral circumflex artery (MFCA). However, the IGA has been found to
be the primary blood supply of the femoral head in up to 17% of patients. It should, therefore, be protected especially when treated posterior hip dislocations and acetabular fractures. The IGA is a terminal branch of the internal iliac artery. It can be found along the piriformis tendon and exits the pelvis out of the greater sciatic notch between the piriformis and coccygeus.
Kalhor et al. performed an anatomic study to define the blood supply to the femoral head. They found that the MFCA was found to be the main blood supply to the hip in a majority of hips, but that the IGA was the main blood supply in 17% of hips. They also found that the foveal artery provided no significant vascular contribution in any specimen. They concluded that, in order to reduce the risk of AVN, the IGA and MFCA should be protected during surgery.
Gautier et al. performed an anatomic study to determine the anatomy of the MFCA. They found that the course of the MFCA was constant in its extracapsular segment and that it perforates the joint capsule at the level of the superior gemellus then runs along the posterosuperior aspect of the femoral neck. They also found that the obturator externus protected the MFCA from being disrupted or stretched during surgical hip dislocation.
Illustration A is a figure demonstrating the anatomy of the gluteal region and posterior thigh.
Incorrect Answers:
Answer 2: This describes the course of the medial femoral circumflex artery. Answer 3: this describes a portion of the lateral femoral circumflex artery.
Answer 4: This describes a portion of the superior gluteal artery.
Answer 5: This describes the deep branch of the medial femoral circumflex artery.
OrthoCash 2020
-
-
Acetabular cup templating examples are shown in Figures A and
B. Moving the acetabular component from position A to position B results in which of the following?
-
Increased force requirement of the abductor musculature
-
Increases leg length
-
Increases joint reactive forces
-
No change in joint reactive forces
-
Decreases joint reactive force Corrent answer: 5
Medialization of the acetabular component leads to a decrease in joint reactive forces.
Preoperative templating in joint arthroplasty is aimed at predicting the correct sizing and placement of components. Acetabular cup placement determines the hip center of rotation and subsequently, the joint reactive forces and abductor force requirements needed to maintain a level pelvis. Medialization of the cup to the native medial wall decreases both abductor forces and joint reactive forces. Decreasing these forces leads to better clinical outcomes and reduces chances of polyethylene wear. Final leg length and offset are determined by a combination of cup and femoral stem positioning.
Schmalzried et al. discussed the medialization of the acetabular component. They found that appropriate medialization can lead to improved function and decreased polyethylene wear. They discuss matching the amount of medialization of the cup with the amount of femoral offset to keep the greater trochanter position unchanged. Finally, they discuss the relationship of the acetabular and femoral head center of rotation with respect to final leg length and offset.
Johnston et al. developed a mathematical model to estimate the effects of different alterations of component positioning in total hip arthroplasty. They found loads on the hip were lower with an acetabular cup placed as far medially, inferiorly, and anteriorly.
Figure A shows a laterally placed acetabular component while Figure B shows a component that is medialized to the medial wall of the acetabulum.
Illustration A shows a free body diagram demonstrating the distance (Db) between the hip center of rotation and the center of gravity of the body(BW). As this distance shortens the lever arm of the body decreases which decreases the force required by the abductors (AT) which ultimately decreases joint reactive force (JRF) across the hip.
Incorrect Answers:
Answer 1: Medialization of the acetabular component leads to a decrease in the force requirement of the abductors.
Answer 2: The combination of cup position and stem position determines final leg length.
Answer 3 & 4: Joint reactive forces are decreased with medialization of the acetabular component.
OrthoCash 2020
-
-
A 72-year-old female presents with the radiographs in Figure A 16 years following total hip arthroplasty. Which of the following statements is true regarding this pathological process?
-
TNF-alpha is released by macrophages
-
RANKL is released by osteoclasts
-
Osteoprotegrin (OPG) binds to RANK
-
RANK and RANKL gene transcription is decreased
-
Interleukins (Il-1, IL-6) are released by osteoclasts Corrent answer: 1
In macrophage-activated osteoclastogenesis and osteolysis, TNF-alpha is released by macrophages, leading to the activation of osteoclasts and eventual bone resorption.
Aseptic loosening after THA is the most common cause of late failure and typically is the result of osteolysis. This process begins with the generation of polyethylene wear particles, which then activates macrophages. Particles smaller than 1 micron, or submicron, are most reactive to macrophage-induced osteolysis. Once activated, macrophages produce TNF-alpha and IL-1, among other cytokines, which in turn promotes RANK-RANKL pathway signaling. RANK, receptor activator of nuclear factor kappa-beta, activation leads to transcription factors that induce gene expression for osteoclastogenesis and osteoclast activation.
Holt et al. reviewed the literature regarding cell signaling molecules involved in osteolysis. The most important mediators reported were TNF-alpha and IL-1, which many studies documented the potentiation of each other during the process. Several studies reported these molecules to be present in high concentrations in the surrounding pseudomembranes in revision arthroplasty.
Azuma et al. performed a study that involved bone marrow macrophage cells cultured with mouse tibia and femur in the presence of several inflammatory
cytokines. Bone resorption assay indicated that the presence of TNF-alpha and M-CSF were crucial to the differentiation of osteoclasts and play an important role in osteolysis.
Figure A demonstrates a frog-leg lateral of the right hip demonstrating extensive periacetabular osteolysis along with the eccentric position of the femoral head within the acetabulum. Illustration A depicts a diagram of TNF-alpha and IL-1 activation of the RANK-RANKL pathway that leads to osteoclastogenesis and bone resorption.
Incorrect Answers:
Answer 2: RANKL is produced by osteoblasts, not osteoclasts.
Answer 3: Osteoprotegerin binds to RANKL, not RANK. It is a soluble inhibitor molecule that acts as a competitive inhibitor to binding with RANK.
Answer 4: The upregulation of the NF-kappa transcription factor is the direct result of RANK-RANKL signaling and leads to gene expression that promotes osteoclastogenesis and bone resorption.
Answer 5: Interleukins that signal osteolysis are not released by osteoclasts, but rather macrophages.
OrthoCash 2020
-
-
Which of the following will decrease the Q-angle when performing a total knee arthroplasty?
-
Medialization of the femoral component
-
Medialization of the tibial component
-
Medialization of the patellar component
-
Internal rotation of the femoral component
-
Internal rotation of the tibial component Corrent answer: 3
An increased Q-angle will lead to patellofemoral instability in TKA. Of the answer choices, medialization of the patellar component will decrease the Q-
angle.
The Q-angle is the angle of the extensor mechanism at the junction of the patellar tendon and quadriceps tendon. At higher angles, there is a greater laterally directed force on the patella, which leads to lateral patellar instability. In order to enhance patellar tracking, the femoral and tibial components are best placed slightly lateral and externally rotated. Patellar components are most stable when placed in a central or medialized position as this allows the patella to track more in line with the tibial tubercle.
Lee et al. performed a retrospective study including 65 primary TKA procedures performed by a single surgeon, where meticulous attention was given to component positioning. At six years follow-up there were no revisions or signs of patellar instability in any patient. The authors concluded that proper component positioning can minimize patellofemoral instability following TKA.
Malo et al. performed a review of patellar instability in total knee arthroplasty. Placement of the tibial and femoral components medially and in internal rotation places the trochlear groove medial and causes the patella to track laterally. The authors recommended CT as the ideal study for identifying component malposition, with revision arthroplasty as the preferred treatment to address malaligned components.
Illustration A demonstrates the measure of the Q-angle. The angle between the intersection of the line connecting from the ASIS to the patella and the line from the patella and the tibial tubercle.
Incorrect Answers:
Answer 1: Medialization of the femoral component will place the trochlear groove farther away from the tibial tubercle, leading to a greater Q-angle. Slight lateralization of the femoral component results in a more stable prosthetic alignment.
Answer 2: Medialization of the tibial component leads to the tibial tubercle being placed more lateral from the trochlear groove and the patella. This will increase the Q-angle and the laterally directed forces.
Answer 4: Internal rotation of the femoral component will lead to the trochlear groove being rotated further away from the tibial tubercle and increase the Q-angle. This malpositioning commonly occurs when using a posterior condylar referencing guide with a hypoplastic lateral femoral condyle.
Answer 5: Internal rotation of the tibial component will lead to the tibial tubercle being displaced laterally leading to a greater Q-angle. Optimal placement of the tibial component places the tibial tray centered between the
intersection of the medial and middle thirds of the tibial tubercle.
OrthoCash 2020
-
-
A 79-year-old patient underwent a right hip hemiarthroplasty after sustaining a femoral neck fracture 8 weeks ago. The patient fell down in the nursing home and noticed immediate pain and inability to bear weight. Figures A and B are the radiographs of the right femur. What is the best treatment approach?
-
Bed rest for 12 weeks
-
Open reduction and internal fixation with placement of original stem
-
Open reduction and internal fixation with placement of a proximally coated stem
-
Open reduction and internal fixation with placement of a diaphyseal engaging stem
-
Proximal femoral replacement megaprosthesis Corrent answer: 4
The patient has sustained a Vancouver B2 periprosthetic femur fracture. The best treatment involves open reduction and internal fixation of the femur fracture with the placement of a revision stem.
Periprosthetic femur fractures are devastating complications of total hip arthroplasty with an expected increase in incidence with increasing patient age. Challenges with treatment involve poor bone stock due to patient age as well as osteolysis. When fixing these fractures it is important to utilize a construct that provides sufficient fixation to allow the fracture to heal as well as accommodating the implant. The Vancouver classification system is useful in the management of these injuries as it dictates treatment. Type A fractures involve the trochanters, type-B1 involves the stem with a well-fixed implant, type-B2 involves a loose stem with good bone stock, and type-B3 involve a loose stem with poor bone stock. Type-C fractures are distal to the implant.
Abdel et al. in 2014 performed a retrospective review of 44 patients with periprosthetic femur fractures, 25 were Vancouver B2 and 19 were Vancouver B3, revised with uncemented modular fluted tapered stems. At follow-up, there was a 98% healing rate with 96% of implants well fixed. The authors concluded that these implant designs provide axial and rotational stability as the fixation point bypasses the fracture.
Haidukewych et al. 2013 reviewed the treatment of periprosthetic femur fractures of the hip and knee. Based on the Vancouver classification they recommended for A-type fractures to use tension band constructs whilst addressing osteolysis. Type-B1 fractures should be addressed with ORIF, type-B2 with ORIF of the proximal femur and revision stem, and type-B3 with ORIF with fluted taper stem or a proximal femoral replacement.
Figures A and B are AP and lateral radiographs showing a right femur with a Vancouver B2 periprosthetic femur fracture. Illustration A demonstrates the postoperative radiograph with cerclage cables and a revision stem.
Incorrect answers:
Answer 1: Bed rest for 12 weeks would be a poor choice as it does not address the loose stem and fracture displacement.
Answer 2: ORIF is necessary to address the fracture, but placing the original stem would not enable a stable construct. A diaphyseal stem is preferred with these fractures.
Answer 3: ORIF is necessary for this fracture, but the use of a proximally coated stem is not ideal as it would need a press fit in the fractured region. Answer 5: A proximal femoral replacement would not be ideal as there is adequate bone stock in this patient.
OrthoCash 2020
-
-
An 81-year-old man reports a year of worsening right groin pain and swelling. A current radiograph and recent MRI are shown in Figures A and B, respectively. ESR and CRP are within normal limits. Aspiration yields 8cc of black liquid with an automated cell count of 7,000 WBC. Manual cell count demonstrates 1,500 WBC and 62% neutrophils. Serum cobalt levels are 12ppb and chromium levels at 2ppb. What is the next best step in management?
-
Greater trochanteric bursa injection
-
Revision arthroplasty with exchange of both components to long modular diaphyseal engaging stem and multihole acetabular cup
-
Revision arthroplasty with exchange of both components to long modular diaphyseal engaging stem and constrained acetabular component
-
Revision arthroplasty with antibiotic spacer placement and subsequent revision arthroplasty
-
Revision arthroplasty with conversion to ceramic head with titanium sleeve and new polyethylene liner
Corrent answer: 5
The best treatment for clinical evidence of corrosion at the head-neck junction in the setting of a large cobalt chrome femoral head is conversion to a ceramic head with titanium sleeve and new polyethylene insert.
Modular components in total hip arthroplasty allow for greater flexibility for the
surgeon; however, head-neck corrosion or trunnionosis can lead to adverse local tissue reactions (ALTR). A cobalt chrome head on a titanium taper can lead to trunnionosis over time. Crevice corrosion leads to metal ion release which can lead to a lymphocytic response and an ALTR. Infection must be ruled out with aspiration prior to revision. Metal debris may lead to a falsely elevated synovial WBC on automated cell count and therefore manual cell count should be performed when head-neck corrosion is suspected. Serum cobalt levels are often elevated 4-5 times as much as chromium levels in head-neck corrosion. Trunnionosis is thought to occur secondary to larger femoral heads, cobalt chrome heads on titanium tapers, and thinner and shorter trunnions. Treatment should consist of revision arthroplasty with debridement, liner exchange, and cleaning of the taper followed by a ceramic head with a titanium sleeve.
Plummer et al. reviewed 27 patients with head-neck corrosion in the setting of a metal on polyethylene bearing. They found cobalt levels were on average
11.2ppb and chromium levels were 2.2ppn. 23 patients were revised to titanium sleeve with ceramic heads and all had improvement of their symptoms and a decrease in their metal ion levels.
Cooper et al. reviewed 10 patients with metal on polyethylene total hip implants who underwent revision for corrosion at the femoral head-neck junction. Following revision surgery, they found improvement in mean Harris hip scores and serum ion levels at 13.0 months postoperatively. They conclude that modular femoral head-neck corrosion presents similar to traditional
metal-on-metal bearings however serum cobalt levels are often significantly elevated compared to serum chromium levels.
Figure A shows an x-ray of the right hip showing a large diameter femoral head. Figure B show an MRI of the right hip demonstrating early pseudotumor formation. Illustration A shows an example of trunnionosis following placement of a large cobalt chrome metal head.
Incorrect Answers:
Answer 1: There is no evidence of greater trochanteric bursitis and injection would be unlikely to address the patient's pain.
Answers 2 & 3: Revision of the femoral or acetabular components would be unnecessary in the setting of well-fixed stable components with the diagnosis of head-neck junction corrosion.
Answer 4: Aspiration results can be falsely elevated with automated cell count in the setting of metal corrosion due to high amounts of debris. Given the low manual cell count, there is no sign of infection in this patient.
OrthoCash 2020
-
-
A 66-year-old patient is planning to undergo a right total knee arthroplasty. Figure A demonstrates the preoperative radiograph. Placing the components in a kinematic alignment (compared to neutral mechanical alignment) would result in which of the following?
-
Increased aseptic loosening
-
Varus tibial cuts and valgus femoral cuts
-
Lower rates of patient satisfaction
-
Decreased ROM
-
Increased reoperation rate
Corrent answer: 2
Kinematic alignment total knee arthroplasty is based on component placement to recreate a patient's natural anatomy. In the case of this patient, this would involve varus tibial cuts and valgus femoral cuts.
Kinematic alignment total knee arthroplasty is based on the principle of re-establishing a patient's natural anatomy. Many patients develop constitutionally varus or valgus knee alignment, in which placement of the arthroplasty components in relative varus or valgus positions would lead to symmetric mediolateral loading of the implants. This principle is further based on the idea that placing the components in neutral alignment may align the limb in an abnormal position to the patient, which may lower patient satisfaction. For varus knees, this implies varus tibial cuts with valgus femoral cuts.
Bellemans et al. performed an observational study of 250 asymptomatic study participant to determine what percentage of the population has constitutionally varus knee alignment. The authors found that 32% of males and 17.2% of females had constitutionally varus aligned knees. Furthermore, constitutionally varus knees were associated with greater physical activity during the second decade of life, believed to be secondary to Heuter-Volkmann loading of the open physis.
Lee et al. performed a systematic review of the literature comparing neutral alignment and kinematic alignment arthroplasty. Generally, the literature supported that ROM, KSS and WOMAC scores were equivalent, if not better, in kinematically aligned knees. Further, tibial components were in more varus and femoral components in more valgus. There were no differences in reoperation rates.
Figure A demonstrates a mechanical axis view radiograph with varus alignment of bilateral knees and medial compartment osteoarthritis. Illustration A demonstrates the difference in bone cuts between neutral alignment and kinematic alignment arthroplasty.
Incorrect Answers:
Answer 1: While hypothesized to be a concern especially on the tibial side, the rates of aseptic loosening have not shown to be different between kinematic and mechanically aligned knees.
Answer 3: Nearly all outcome measures have been found to be either equivalent to or better than neutrally aligned knees.
Answer 4: Range of motion has been found to be either similar to or greater than neutrally aligned knees.
Answer 5: There has been no difference in reoperation rates between the two alignment principles.
OrthoCash 2020
-
-
A 68-year-old patient with diabetes progressively worsening left knee pain of 6 months duration. They underwent a left total knee arthroplasty 7 years ago. Figures A-B demonstrate the current radiographs. Aspiration of the left knee demonstrated 11,500 WBCs and 94% neutrophils. Aspiration cultures grew methicillin-resistant Staphylococcus aureus. What would be the best treatment approach for this patient?
-
Knee arthrodesis
-
Long-term antibiotic suppression
-
One-stage revision arthroplasty
-
Two-stage revision arthroplasty
-
Above knee amputation Corrent answer: 4
The patient has several medical comorbidities and is presenting with a chronic prosthetic joint infection with a virulent organism (MRSA). The best treatment option at this time would be a two-stage revision arthroplasty.
Chronic prosthetic joint infections occur greater than 3-6 weeks from surgery and result in biofilm formation over the prosthesis, making antibiotic treatment alone ineffective for infection eradication. Diagnosis is based on the MSIS criteria, with chronic infections being suggested with CRP greater than 10, ESR greater than 30, and synovial WBCs greater than 1,100. Two-stage revision arthroplasty is the current gold standard in the United States for treating chronic prosthetic joint infections.
Kuzyk et al. 2014 reviewed two-stage revision arthroplasty for chronic periprosthetic joint infections. The authors stated that there is no consensus on laboratory and histology criteria to confirm eradication prior to proceeding with the second stage. The authors recommend holding IV antibiotics for two weeks and repeated inflammatory markers to evaluate whether to proceed with the second stage and to perform frozen section at the time of the procedure.
Nguyen et al. 2016 reviewed one-stage revision arthroplasty for the treatment of periprosthetic joint infections. The authors reported that in select patients, one-stage revision arthroplasty can have equal if not better outcomes compared to two-stage revision with less surgical morbidity and improved functional outcomes. They concluded that one-stage revision arthroplasty can be successful in patients that are not immunocompromised, minimal medical comorbidities, known pathogen prior to surgery, non-polymicrobial, no virulent pathogen (MRSA), and with good soft tissue coverage.
Figures A and B demonstrate AP and lateral radiographs of the right with radiolucencies present around the tibial and femoral prosthesis. Illustration A demonstrates a treatment algorithm proposed by Kuzyk et al. for proceeding with the second stage of a two-stage revision. Illustration B demonstrates the Musculoskeletal Infection Society diagnostic criteria for a prosthetic joint infection. Illustration C depicts specific lab values for diagnosing a prosthetic joint infection.
Incorrect Answers:
Answer 1: A knee fusion is an option after the infection has been eradicated, however, this is unfavorable as it would leave the patient with a stiff extremity.
Answer 2: Long term antibiotic suppression is an option for patients that are unable or unwilling to tolerate multiple procedures.
Answer 3: One stage revision arthroplasty is an option in the right candidate, but this patient is infected with a virulent organism making one-stage unlikely to succeed.
Answer 5: An above knee amputation is a viable option for patients that have failed to eradicate the infection after several surgical attempts with IV antibiotics. However, this is not warranted as initial therapy.
OrthoCash 2020
-
-
A 77-year-old patient presents with progressively worsening right hip pain and limp. The patient underwent a right revision total hip arthroplasty 15 years ago and is now unable to ambulate due to the pain and feels as if the hip is unstable. The patient's radiograph is shown in Figure 1. Which of the following is the appropriate classification and best treatment approach for this patient?
-
Paprosky 2A; multihole cup with posterior column plating
-
Paprosky 2B; antiprotrusio cage with structural allograft
-
Paprosky 3A; distraction arthroplasty
-
Paprosky 3B; custom triflange cup
-
Paprosky 3B; cemented cup Corrent answer: 4
The patient is presenting with pelvic discontinuity due to severe acetabular bone loss and superomedial cup migration consistent with Paprosky 3B
acetabular deficiency and pelvic discontinuity. Revision to a custom triflange cup would be a viable treatment approach.
Pelvic discontinuity in revision total hip arthroplasty is a rare treatment challenge due to extensive bone loss from osteolysis and prior surgery. Typically, the cup migrates superomedial towards the pelvic viscera and can place neurovascular structures at greater risk. This defect is classified as type 3B in the Paprosky classification. Treatment involves restoring pelvic stability through the healing of the anterior and posterior columns as well as reconstituting hip biomechanics with custom triflange cups, posterior column plating, distraction arthroplasty, or augments with highly porous cups.
Taunton et al. performed a multicenter retrospective review of 57 patients that underwent reconstruction of pelvic discontinuity with a custom triflange cup.
The authors found that 81% of patients had a stable implant and healed discontinuity at final follow-up with implant cost being comparable to off-the-shelf options. The authors concluded that that custom triflange cup provides adequate fixation with good outcomes at a comparable cost to other fixation methods.
Jenkins et al. performed a retrospective review of 58 hips, of which 11 had pelvic discontinuity, that underwent revision with a tantalum porous cup and augments. The authors reported a high rate of radiolucency in Delee and Charnley zone III and implant failure in patients with pre-operative pelvic discontinuity that were revised with this technique. The authors recommend the use of alternative or adjunctive fixation in patients with pelvic discontinuity.
Regis et al. performed a retrospective review of 18 patients that underwent revision with antiprotrusio cage and bulk allograft for pelvic discontinuity. The authors found a 72.2% survival rate at 16.6 years with cases of failure demonstrating graft resorption and acetabular loosening. The authors suggested that bulk allografting with antiprotrusio cages provide an effective means to address pelvic discontinuity.
Figure A demonstrates an AP radiograph of the pelvis with pelvic discontinuity. Illustration A depicts the Paprosky classification system. Illustration B depicts the DeLee and Charley as well as the Gruen zones.
Incorrect Answers:
Answers 1, 2, 3: The patient has a Paprosky 3B acetabular defect with pelvic discontinuity.
Answer 5: Revision with a cemented acetabular cup would not stabilize the
pelvic discontinuity in such a manner to allow for healing of the anterior and posterior columns. Furthermore, cemented cups are associated with high loosening rates.
OrthoCash 2020
-
-
A 75-year-old male sustains a ground-level fall while ambulating at home. The patient has been optimized for surgical intervention. Both prosthetic components are deemed to be stable. How would you classify this fracture and what is the appropriate treatment plan?
-
Vancouver B1; ORIF with a lateral locking plate
-
Vancouver C; revision of femoral stem from hip component
-
Vancouver C; retrograde intramedullary nail
-
Vancouver B2; revision to long stem total knee component
-
Vancouver C; ORIF with a lateral locking plate Corrent answer: 5
This patient has a Vancouver C periprosthetic fracture about stable total hip and knee arthroplasties (an interprosthetic fracture) which can be appropriately fixed with a lateral locked plate spanning the entire femur.
The success of prosthetic surgery has led to an increase in the percentage of the population having more than one prosthetic implant. This, combined with an increase in the average life expectancy and functional requirements for the elderly, has led to a higher incidence of periprosthetic and interprosthetic fractures. Treatment must be determined and assessed according to the type of fracture, the stability of the prosthesis, the bone quality and the general condition of the patient. When the implants are stable plate fixation spanning both of the prostheses has shown favorable results. Some surgeons advocate for nail/plate combination fixation in these interprosthetic fractures in an attempt to allow early weight-bearing.
Froberg et al. reviewed 60 consecutive periprosthetic Vancouver B1 or C fractures, all fixed with plate osteosynthesis. There were a total of 8 reoperations, 4 of which were for infection and 3 for fixation failure. They conclude that locking-plate osteosynthesis of periprosthetic Vancouver type B1 and C fractures gives good results in terms of fracture union. It appears that spanning of the prosthesis to avoid stress-rising areas is important for successful treatment. Infection remains the major cause of failure.
Hoffmann et al. reviewed interprosthetic femoral fractures, defined as fractures between an ipsilateral total knee and hip arthroplasty. 27 patients were designated as having interprosthetic fractures and were treated with locked plating. They conclude that locked plating can achieve satisfactory results. Additional soft tissue damage can be prevented by submuscular plate insertion. Treatment of type B fractures resulted in significantly greater nonunion rate.
Matlovich et al. reviewed fifty-seven patients treated for supracondylar periprosthetic femur fracture with either a locking plate (n = 38) or IM nail (n
= 19). There was no statistical difference between groups in the meantime to fully weight bear, the incidence of postoperative pain, range of motion, use of gait aids, time to full radiographic union, or the overall radiographic alignment of a healed fracture. Despite this, they add caution is recommended in using IM nails for fractures below the flange where limited fixation may increase the risk of nonunion.
Figure A demonstrates a Vancouver C interprosthetic fracture with well-fixed total hip and knee components.
Illustration A is an example of another patient status-post ORIF of an interprosthetic femur fracture.
Incorrect answers:
Answers 1,2,4: This is best classified as a Vancouver C periprosthetic fracture. Answer 3: This fracture is best treated with an implant that can span both components to avoid an area of a stress riser.
OrthoCash 2020
-
-
A 61-year-old man with left hip OA presents to clinic for persistent left hip pain despite a trial of conservative therapy. The decision is made to proceed with total hip arthroplasty via a direct anterior approach. Which of the following correctly describes the superficial internervous plane of this approach?
-
Rectus femoris (femoral n.) & tensor fascia lata (superior gluteal n.)
-
Tensor fascia lata (femoral n.) & sartorius (superior gluteal n.)
-
Rectus femoris (femoral n.) & gluteus medius (superior gluteal n.)
-
Sartorius (femoral n.) & gluteus medius (superior gluteal n.)
-
Sartorius (femoral n.) & tensor fascia lata (superior gluteal n.) Corrent answer: 5
The direct anterior approach to the hip is performed using the internervous interval between the sartorius (femoral n.) and tensor fascia lata (superior gluteal n.) superficially.
Total hip arthroplasty using a direct anterior approach has become increasingly
popular, with many studies showing good long-term results. It is performed through the internervous plane between the femoral nerve and superior gluteal nerve, superficially between the sartorius and TFL, and deep between the rectus femoris and gluteus medius. Advantages of the direct anterior approach include preservation of the abductor mechanism and decreased dislocation rates compared to the posterior approach. However, this approach has a steep learning curve and its use is limited in obese patients with a large pannus. Additionally, this approach places the lateral femoral cutaneous nerve at risk and may lead to increased intraoperative fracture rates.
Bohler et al. published a review on the direct anterior approach to the hip. They report that this approach allows for direct visualization of the acetabulum and offers a complete intermuscular and internervous access to the hip joint.
They found that the approach allows for decreased muscular trauma, intraoperative blood loss, and post-operative rehabilitation.
Post et al. published a review on the indications, technique, and results of the direct anterior approach for THA. They report that the steep learning curve and complications unique to this approach (fractures and nerve damage) have been well described; however, the incidence of these complications decreases with greater surgeon experience.
Illustration A is a diagram depicting the superficial internervous plane of the direct anterior approach to the hip.
Incorrect Answers:
Answers 1, 2, 3, and 4: The direct anterior approach to the hip is performed using the internervous interval between the sartorius (femoral n.) and tensor fascia lata (superior gluteal n.) superficially.
OrthoCash 2020
-
-
A 45-year-old male presents with increasing left groin pain. He has a history of bilateral hip avascular necrosis and underwent bilateral hip resurfacing arthroplasties 3 years ago. He is a recreational runner and recently ran a 10-kilometer race several weeks ago. Figure A demonstrates an AP radiograph of his pelvis. Serum testing demonstrated a cobalt level of 10 mcg/L (reference 0.8
- 5.1 mcg/L) and chromium level of 7 mcg/L (reference 0.5 - 2.5 mcg/L). What is the likely cause of the patient's symptoms?
-
Iliopsoas tendonitis
-
Edge-loading
-
Prosthetic joint infection
-
Increased activity-related wear
-
Femoral neck stress fracture Corrent answer: 2
The patient is presenting with increased left hip pain after bilateral Birmingham Hip Resurfacing (BHR) arthroplasties and elevated ion levels consistent with metallosis. The most likely cause of metallosis in this patient is the edge-loading of the implant.
Hip resurfacing arthroplasty is a bone preserving procedure that is favorable in young male patients and utilizes metal-on-metal articulations. Metallosis is, therefore, a concerning complication of these implants and can result in pseudotumor formation and subsequent destruction of the hip abductors.
Patients presenting with a painful prosthesis should be screened with metal ion levels as well as a metal artifact reduction sequence MRI. Possible causes of metallosis include edge-loading, component malpositioning, third bodywear, impingement, and sensitivity to cobalt.
Brooks performed a retrospective review of patients undergoing BHR with regards to outcomes. The author found a 0.23% rate of metallosis in the study population, with all cases being attributed to edge-loading. The author recommended a preoperative CT scan and lateral pelvic radiographs to prevent component malposition and better identify surgical candidates.
Matharu et al. performed a prospective cohort study examining metal ion levels in patients with bilateral BHRs. The authors found that cobalt, chromium, cobalt-chromium ratio, and maximum cobalt and chromium levels to be significantly higher in patients with metallosis. They recommended using a cut-off of 5.7 mcg/L for cobalt and 5.5 mcg/L for chromium for metallosis in patients with bilateral BHRs.
Figure A demonstrates an AP radiograph of a pelvis with bilateral BHR implants and an increased inclination angle of the left acetabular component.
Incorrect Answers:
Answer 1: Iliopsoas tendonitis is a cause of hip pain in patients with hip arthroplasty; however, this would not cause the patient to have increased metal ion levels.
Answer 3: A prosthetic joint infection could cause the patient to have increasing pain in their hip but would not cause elevated metal ion levels.
Answer 4: Studies have not shown that increased physical activity leads to elevated metal ion levels.
Answer 5: Femoral neck stress fractures are a potential complication in the acute postoperative period, but this would not cause an elevation in metal ion levels.
OrthoCash 2020
-
-
An ambulatory 57-year-old man with post-polio syndrome presents for follow-up of his right knee pain. He has failed all nonoperative measures for his right knee pain. On exam, he hyperextends to 15° and flexes to 120° with global instability of the knee. He has maintained antigravity strength in the right limb. Radiographs are shown in Figures A and B. What is the best treatment option for this patient?
-
Cruciate retaining knee with ligamentous reconstruction
-
Robotic-assisted posterior stabilized total knee arthroplasty
-
Distal femoral osteotomy with total knee arthroplasty
-
Hinged total knee arthroplasty
-
Above-knee amputation
Corrent answer: 4
The best treatment for this patient with post-polio syndrome is a hinged total knee arthroplasty.
Patients affected by the 1950 poliomyelitis outbreak are now approaching an age where degenerative knee changes are impacting their quality of life. These patients often have global instability and significant hyperextension deformity and require a hinged prosthesis. Any less constrained components put the patient at risk for continued instability and early failure. Patients with maintained antigravity strength in the operative limb may have improved outcomes in the setting of post-polio syndrome.
Gan et al. reviewed 16 knee replacement in patients with post-polio syndrome and degenerative knee changes. They found an improvement in the mean of all outcomes scoring measures including the AKSS, Oxford knee scores, AKSS pain scores, and SF-36 scoring. They concluded that primary knee arthroplasty for patients with post-polio syndrome shows a good improvement in patients quality of life and decreases pain.
Giori et al. retrospectively reviewed 16 patients with a history of poliomyelitis and a history of primary total knee arthroplasty. They found four cases of recurrent instability, two of which had a preoperative hyperextension deformity of 20 degrees. They concluded that pain and knee scores improved in these patients and that recurrent instability or functional deterioration occurred more often in the most severely affected knees. They recommend consideration of hinged arthroplasty or arthrodesis in this challenging subgroup of patients.
Figures A and B are AP and lateral radiographs of the right knee showing and severe valgus-hyperextension deformity and degenerative changes in a post-polio limb. Illustrations A and B show an AP and lateral postoperative radiograph following a hinged arthroplasty.
Incorrect Answers:
Answer 1: Ligament reconstruction and a cruciate retaining arthroplasty would likely lead to early failure in a patient with post-polio syndrome.
Answer 2: Although a posterior stabilized knee would increase constraint, it would be insufficient in a globally unstable knee.
Answer 3: Although there is a valgus deformity, this would not address the problem of global instability.
Answer 5: An above-knee amputation would unnecessarily remove a salvageable limb and increase energy expenditure with ambulation.
OrthoCash 2020
-
-
When compared to a median parapatellar approach which of the following approaches may lead to higher rates of component malposition?
-
Quadriceps sparing
-
Lateral parapatellar
-
Midvastus
-
Quadriceps snip
-
V-Y turndown
Corrent answer: 1
A quadriceps-sparing approach has been found to lead to a high rate of component malpositioning.
Improvements in surgical instrumentation and techniques drove surgeons to perform total knees replacement in a less invasive manner. One such technique was the quadriceps-sparing approach which uses minimal subluxation of the patella and special side cutting instruments. This technique was thought to lead to quicker recovery due to the minimal disturbance of the extensor mechanism however, studies have shown that it may lead to statistically significant higher rates of component malposition when compared to a traditional median parapatellar approach.
Kazarian et al. reviewed the outcomes of the quadriceps-sparing (QS) approach compared to a median parapatellar(MP) approach for total knee arthroplasty. They found statistically and clinically significant disadvantages to the QS approach including femoral and mechanical axis outliers, increased surgical time, and increased tourniquet time. They concluded the QS approach does not demonstrate any clinically significant advantages and leads to higher rates of component malalignment.
Kelly et al. randomized 42 consecutive total knee patients to either median parapatellar (MP) approach or a vastus splitting (VS) approach. They found a statistically significant increase in the rate of lateral release and blood loss in the MP approach but showed no difference in functional parameters, tourniquet time, or patellar resurfacing. They conclude the VS approach is a reasonable alternative to the MP approach and may lead to lower rates of lateral releases without impairment of quadriceps function.
Liu et al. compared outcomes of the minimally invasive midvastus (MV) and
subvastus (SV) approaches compared to a traditional median parapatellar (MP) approach for total knee arthroplasty. They found the number of days needed to perform a straight leg raise was significantly longer following MP compared to SV or MV approaches. They conclude that further studies should be performed to assess the outcomes of the various minimally invasive approaches for total knee arthroplasty.
Illustration A shows four different approaches for a total knee arthroplasty including three minimally invasive approaches.
Incorrect Answers:
Answers 2, 3, 4, & 5: These approaches have not been shown to lead to component malpositioning.
OrthoCash 2020
-
-
A 66-year-old patient that underwent a right total knee arthroplasty approximately 4 years ago presents with worsening right knee pain over the last 48 hours. The patient has a history of rheumatoid arthritis and recently underwent a dental procedure a week ago. Labs were significant for CRP of 212, ESR 105, and a WBC count of 11K. Aspiration yielded a milky-looking fluid with 55K nucleated cells with 97% PMN. Radiographs are shown in Figures A and B. What is the next best step?
-
Surgical intervention after cultures finalize
-
Repeat aspiration of the knee and send for alpha-defensin
-
Begin IV antibiotics and re-evaluate in 24-48 hours
-
Proceed with surgical intervention now
-
IR guided drain placement Corrent answer: 4
The patient is presenting with an acute hematogenous prosthetic joint infection, which requires surgical treatment as soon as safely possible.
Periprosthetic joint infections (PJI) are generally managed surgically. Diagnosis is composed of a battery of findings as established by the Musculoskeletal Infection Society (MSIS), which requires the presence of one of two major criteria or four of six minor criteria (Illustration A). Acute infections can often be treated with irrigation and debridement with polyethylene exchange (IDPE), whereas chronic infections are best managed with a two-stage revision.
Buller et al. performed a retrospective study looking at variable affecting the success of IDPE treatment for PJI. The authors found that infections with MRSA or VRE, higher ESR levels, symptoms longer than 3 weeks, and previous joint infections were strong risk factors for failure of IDPE. The authors
concluded that patients presenting with these characteristics may be best treated with a two-stage revision rather than IDPE.
Figures A and B are AP and lateral radiographs of a right knee with a stable appearing total knee arthroplasty prosthesis. Illustration A is a table that depicts the 2011 MSIS criteria for diagnosing prosthetic joint infections.
Incorrect Answers:
Answer 1: Delaying surgical intervention for aspiration cultures to finalize is not recommended. Ideally, surgery should proceed as soon as safely possible. Answer 2: Repeat aspiration and sending for alpha-defensin would further delay treatment in this patient.
Answer 3: IV antibiotics alone is not sufficient treatment for PJI in patients able to tolerate surgery.
Answer 5: IR guided drain placement is not sufficient treatment for PJI in patients able to tolerate surgery.
OrthoCash 2020
-
-
A 63-year-old patient with a previous right TKA 4 years ago presents with worsening pain in the right knee. The patient reports that pain is worsened when starting physical activity, but is also present at night. Two weeks prior to presentation the patient was given a 1-week course of oral antibiotics for cellulitis affecting the right knee. Serum labs were significant for a CRP of 11 mg/L and an ESR of 35 mm/hr. Synovial fluid analysis revealed 1,000/µL nucleated cells with 85% PMNs and no evidence of crystals. Synovial cultures were negative for any bacterial or fungal growth. Synovial alpha-defensin is positive. Figures A and B are the AP and lateral
radiographs of the right knee. The patient opts to undergo a revision total knee arthroplasty. What is the best management at this point?
-
Femoral component revision
-
Tibial component revision
-
Polyethylene component revision
-
One-stage revision of both the femoral and tibial components
-
Two-stage revision of both the femoral and tibial components Corrent answer: 5
The patient is presenting with increasing knee pain consistent with either septic or aseptic loosening of the prosthesis. Serum and synovial labs are not diagnostic for an infection, but there is a positive synovial alpha-defensin suggesting the presence of a chronic prosthetic joint infection.
Prosthetic joint infections are diagnostic challenges as there is no single confirmatory test. Rather, diagnosis is composed of a conglomerate of physical and laboratory findings as laid forth by the Musculoskeletal Infection Society criteria. Diagnosis can be made by either the presence of one major criterion or four minor criteria. Synovial alpha-defensin is a new assay that tests for the presence of an antimicrobial peptide that is part of the innate immune system. Recent studies have suggested a high sensitivity and specificity of this test for prosthetic joint infections, even with prior antibiotic administration. Treatment involves two-stage revision arthroplasty with culture-specific antibiotics for at least six weeks. Reimplantation of a prosthesis is done with infection eradication is confirmed.
Shahi et al. performed a retrospective diagnostic study looking at whether prior antibiotic administration affected synovial alpha-defensin levels. The authors found that alpha-defensin was not affected by prior antibiotic administration. The authors concluded that since many patients with PJI will present with prior antibiotic administration, alpha-defensin may be an ideal diagnostic adjunctive test.
Frangiamore et al. performed a prospective cohort study on the sensitivity and specificity of alpha-defensins in diagnosing prosthetic joint infections. The authors found that alpha-defensin has a sensitivity and specificity of 100% and 98%, respectively, for diagnosing PJI in single-stage and first-stage revisions. The authors concluded that alpha-defensin has the potential as a useful adjunct in diagnosing PJI.
Figures A and B demonstrate AP and lateral radiographs of the right knee with loosening of the tibial and femoral components. Illustration A depicts the MSIS criteria for the diagnosis of PJI.
Incorrect Answers:
Answers 1-3: Revision arthroplasty without addressing the infection that is present will lead to failure. A two-stage revision is needed given the chronicity of the infection.
Answer 4: One-stage revision for prosthetic joint infections has been found to be effective in select cases, however, two-stage revision remains the gold-standard for addressing chronic infections.
OrthoCash 2020
-
-
Of the following, which has the highest strength of recommendation according to the AAOS Clinical Practice Guidelines (CPG) for Surgical Management of Osteoarthritis of the Knee?
-
Preoperative physical therapy improves pain and physical function postoperatively
-
Tourniquet use during total knee arthroplasty (TKA) decreases short-term postoperative function
-
Continuous passive motion (CPM) after TKA improves outcomes
-
Rehabilitation started on the day of TKA reduces length of hospital stay
-
Surgical navigation should be used because there is a decrease in pain and functional outcomes
Corrent answer: 4
Rehabilitation started on the day of TKA decreasing length of stay has been deemed a "strong recommendation" by the AAOS.
Postoperative management following TKA is an important aspect of achieving an optimal outcome following total knee arthroplasty. The general recommendation is that patients should work with a physical therapist on the
day of surgery or as early as possible as it will decrease pain and improve function. Early rehabilitation is felt to also decrease the length of stay.
Recommendations against cryotherapy machines and CPM are moderate and strong, respectively, as they do not appear to improve outcomes.
McGrory et al. present the AAOS CPG's for surgical management of osteoarthritis of the knee. Strong evidence supports postoperative rehabilitation started on the day of surgery, which has been shown to decrease the length of stay. Moderate evidence supports rehabilitation starting the day of surgery compared to postoperative day 1 reduced pain and improves function. Various other preoperative and intraoperative topics are reviewed for the corresponding strength of recommendation.
Incorrect Answers:
Answer 1: Preoperative physical therapy has a "limited" strength of recommendation with regard to decreased pain and improved function postoperatively.
Answer 2: Tourniquet use during TKA has a "limited" strength of recommendation with regard to decreasing postoperative short-term function. Answer 3: A strong recommendation was given to CPM after TKA not improving outcomes.
Answer 5: A strong recommendation was given to not use surgical navigation for TKA as there is no difference in outcomes or complications.
OrthoCash 2020
-
-
A 62-year-old patient that underwent a right hip resurfacing arthroplasty 3 years ago develops worsening right hip pain over the past 6 months. The pain is present at all times, including at night. The patient does not walk with a Trendelenburg gait and does not have reproducible pain on hip examination. Laboratory inflammatory markers from 1 week ago were erythrocyte sedimentation rate of 66 mm/hr (reference <20 mm/hr), C-reactive protein of 22 mg/dL (reference <2.5 mg/dL), cobalt 0.5 µg/L (reference <0.7 µg/L), and chromium of 0.4 µg/L (reference <0.4 µg/L). Figure A demonstrates an AP radiograph of the pelvis. What is the next best step in management?
-
Physical therapy
-
Routine follow-up
-
Arthrocentesis with synovial fluid analysis
-
2-stage revision
-
Metal artifact reduction sequence MRI Corrent answer: 3
The patient is presenting with a painful right hip after a metal-on-metal arthroplasty with recent elevated inflammatory markers and normal metal ion levels. The best next step in diagnostic workup would include an arthrocentesis with synovial fluid analysis.
Diagnosis of chronic prosthetic joint infections is challenging due to the requirement of a conglomerate of physical and laboratory findings for the diagnosis. The most common presenting symptom is pain in the affected joint, but there may be draining sinus tracts and systemic inflammatory signs.
Work-up should start with serum inflammatory markers, which if elevated should prompt an arthrocentesis with synovial fluid analysis. If the diagnosis is still not clear a repeat aspiration can be performed or frozen section performed in the operating room. It is important in aspiration of metal on metal joints to request manual cell counts as the metal debris can often result in faulty automated counts.
Connelly et al. performed a prospective cohort study of indications for performing metal artifact reducing sequence (MARS) MRI on patients with metal-on-metal hip resurfacing arthroplasty. They found that elevated cobalt and chromium were the strongest predictors for an adverse local tissue reaction and using 1.15 ppb of Co and 1.09 ppb for chromium as cut-offs for
performing a MARS MRI.
Yi et al. performed a retrospective study evaluating the diagnostic accuracy of serologic and synovial tests for PJI in MoM hip arthroplasty. The authors found that a high rate of inaccurate reporting of MoM aspirations, with 35% of inaccurate reports having a synovial WBC count >3000 suggesting a false positive for infection. The authors concluded that using synovial WBC >4350 and PMN >85% provided greater diagnostic sensitivity and specificity than standard MSIS criteria.
Figure A demonstrates an AP pelvis radiograph with a stable appearing right hip resurfacing arthroplasty.
Incorrect answers:
Answer 1 and 2: Elevated inflammatory markers in the presence of a painful hip arthroplasty should warrant further workup, including arthrocentesis with synovial fluid analysis. Physical therapy and routine follow-up would neglect the possibility of an infection.
Answer 4: A 2-stage revision would be ideal if the diagnosis of a prosthetic joint infection is confirmed.
Answer 5: Performing a metal artifact reduction sequence MRI of the hip to identify an aseptic lymphocyte-dominated vasculitis-associated lesion or abductor damage would be indicated if there was suspicion of metallosis with elevated metal ions.
OrthoCash 2020
-
-
Compared to a cruciate retaining knee prosthesis, an anterior stabilized prosthesis has what effect on the contact area and what effect on the stability in PCL deficient knees?
-
Decreased contact area; increased stability
-
Increased contact area; increased stability
-
No change in contact area; no change in stability
-
Decreased contact area; decreased stability
-
Increased contact area; decreased stability Corrent answer: 2
An anterior stabilized knee prosthesis is composed of highly conformed polyethylene component with a large anterior lip, which prevents anterior translation of the femur on the tibia. The high conformity of the polyethylene component increases the contact area of the implant.
In arthroplasty, several factors can affect wear characteristics and stability at the bearing surface. Articular surfaces that are more congruous decrease the contact stresses at the surface by dispersing the joint reactive forces across a greater area. There are drawbacks to this as the articular surface is less anatomic and can prevent the natural roll back kinematics in total knee arthroplasty. New prosthesis designs with a large anterior lip formed on the bearing can be used to stabilize the knee in the absence of a functional PCL without sacrificing bone stock for the box cut and potential patellar complications.
Peters et al. performed a retrospective cohort study of total knee arthroplasty outcomes between cruciate retaining prostheses in intact PCL knees and anterior stabilized bearing prostheses in PCL deficient knees. They found that anterior stabilized bearing prostheses had similar knee society scores, radiographic alignment, component loosening, and major complications but had a significantly lower number of revisions performed. They concluded that the use of anterior stabilized bearings is an effective implant to stabilize PCL deficient knees.
Brockett et al. performed a biomechanical study that examined wear characteristics of poly-ether-ether-ketone (PEEK) and carbon fiber reinforced PEEK (CFR-PEEK) and compared to ultra-high molecular weight polyethylene (UHMWPE). It was found that PEEK had worse wear characteristics compared to UHMWPE, specifically with increasing contact pressures, decreased contact surface, and increased cross-shear. However, CFR-PEEK had similar wear performance as UHMWPE, but there were increased wear characteristics with increasing shear and contact pressure. They concluded that CFR-PEEK may be a potential alternative bearing surface in arthroplasty, but further investigation is needed to determine it's role in less conforming bearing due to the increased shearing wear.
Illustration A depicts the design of a cruciate retaining total knee prosthesis and an anterior stabilized total knee prosthesis.
Incorrect Answers:
Answer 1, 3, 4 and 5: Anterior stabilized knees are more conforming than standard cruciate retaining prostheses which increases the contact area. Due to the anterior lip, it confers greater stability than a cruciate retaining prosthesis in a PCL deficient knee. All other answer choices are incorrect.
OrthoCash 2020
-
-
Resection of the posterior cruciate ligament during total knee arthroplasty simulates which of the following techniques below?
-
Excessive distal femur resection
-
Excessive distal femur augmentation
-
Excessive posterior femur resection
-
Excessive posterior femur augmentation
-
Oversized femoral component Corrent answer: 3
Posterior cruciate ligament (PCL) resection during total knee arthroplasty (TKA) results in a relative increase in the flexion gap compared to the extension gap. This effect simulates excessive posterior femur resection, which also results in an increased flexion gap.
The PCL acts as a central stabilizer to prevent posterior subluxation, allows femoral condyle roll back on the tibial plateau during flexion, and permits clearance of the tibia in high degrees of flexion to improve the mechanical efficiency of the extensor mechanism. The PCL may be preserved or resected during TKA. Biomechanical studies have demonstrated that after PCL resection, the flexion gap increases significantly compared with the extension gap. This has implications on gap balancing during posterior-stabilizing (PS) TKA, as the flexion gap must match the extension gap.
Park et al. performed a study to investigate the change in the medial-lateral gap in flexion and extension after PCL resection in severely deformed knees and its effect on bone resection, rotation, and size of the femoral component. They reported that after PCL resection, the flexion gap increased significantly compared with the extension gap. They concluded that PCL resection in PS-TKA designs necessitates an increase in the size of the femoral component to
balance the resulting gap mismatch.
Sierra et al. published an article on the surgical technique differences between cruciate-retaining (CR) and PS TKA designs. They reported that PCL resection selectively opens the flexion space approximately 2mm more than the extension space, resulting in some flexion instability. They recommended that in PS-TKA, surgeons must avoid flexion instability due to an extra large flexion space caused by PCL sacrifice and postoperative knee flexion contracture by underresection of the distal femur.
Incorrect Answers:
Answer 1: Excessive distal femur resection would result in a knee loose in extension.
Answer 2: Excessive distal femur augmentation would result in a knee tight in extension.
Answers 4 & 5: Excessive posterior femur augmentation and an oversized femoral component would result in a knee tight in flexion.
OrthoCash 2020
-
-
A 70-year-old healthy woman presents with recurrent left prosthetic hip dislocations after undergoing total hip arthroplasty 6 months ago. Workup for infection has been negative. Radiographs from her visit today are depicted in Figure A. Which of the following will most definitively prevent further dislocations?
-
Exchange polyethylene liner to a lipped acetabular liner
-
Exchange polyethylene liner to a thinner liner and increase the size of femoral head component
-
Cemented acetabular component revision
-
Uncemented acetabular component revision
-
Exchange polyethylene liner to a constrained acetabular liner Corrent answer: 4
This patient appears to have a significantly increased acetabular abduction (theta) angle, which places her at a high risk of periprosthetic dislocation. She should undergo revision of her malpositioned acetabular component to prevent further dislocation events.
Variables that help determine stability after total hip arthroplasty (THA) include component design, component position, soft tissue tension, and soft tissue function. The component position comprises of both femoral and acetabular implants. The recommendations are femoral component anteversion of 10-15 degrees, acetabular anteversion of 5-25 degrees, and acetabular abduction of 30-50 degrees. Excessive abduction may result in posterosuperior instability whereas inadequate abduction may result in impingement during flexion as well as inferior instability. Component malposition generally requires revision and cannot be compensated for by abductor strengthening or orthoses.
Dewal et al. retrospectively reviewed THA dislocations to determine the effectiveness of abduction bracing following closed reduction. They observed no significant differences in first-time dislocators or recurrent dislocators with or without the use of abduction braces. They concluded that abduction bracing following closed reduction of THA dislocation is ineffective in preventing re-dislocation.
McCarthy et al. performed a study to investigate cup position angles associated with impingement in a group of subjects during different activities. They reported that true acetabular target for impingement-avoidance is much smaller than previously believed and varies considerably between patients and that certain tasks including low-chair rise and squatting decrease the size of the target zone. As such, they recommended preoperative patient-specific planning and intraoperative execution for placement of the components.
Figure A depicts a THA construct with significantly increased acetabular abduction angle.
Incorrect Answers:
Answer 1: While lipped liners are associated with a significantly decreased rate of revision for instability, this patient has a malpositioned acetabular component that needs to be revised and could not be managed with a liner change alone.
Answer 2: Increase in femoral head size may increase relative stability but the acetabular component is malpositioned in this patient.
Answer 3: Cemented acetabular components have higher revision rates than uncemented components
Answer 5: A constrained liner in a socket in poor position will lead to early failure as the reduction of range of motion will lead to early impingement and either liner failure or cup pull out.
OrthoCash 2020
-
-
A 65-year-old woman with a history of right total hip arthroplasty presents with a fall. Her injury radiographs are depicted in Figure A. What are the fracture classification and most appropriate treatment?
-
Vancouver AG; nonoperative with partial weight bearing
-
Vancouver AG; open reduction internal fixation with trochanteric claw plate
-
Vancouver AG; femoral component revision
-
Vancouver AL; open reduction internal fixation with trochanteric cables
-
Vancouver B1; open reduction internal fixation with lateral locking plate Corrent answer: 2
This patient has a displaced (> 2cm) greater trochanteric periprosthetic fracture around her previous right total hip arthroplasty (THA). Her fracture is classified in the Vancouver classification as AG, and is best treated with open reduction internal fixation (ORIF) using a trochanteric claw plate.
The Vancouver hip periprosthetic classification system is one of the most useful classifications in the field of orthopaedic surgery, as it can reliably guide decision-making regarding fixation versus revision of the femoral component. Vancouver A fractures confer fractures about the femoral trochanters, with AG and AL fractures depicting greater and lesser trochanters, respectively. While nondisplaced and minimally displaced (<2cm) Vancouver AG fractures may be managed nonoperatively with protected weight-bearing, displaced AG fractures should be treated with ORIF using wires, cables, or claw plates.
Sariyilmaz et al. performed a biomechanical study to compare fixation techniques (cables, trochanteric grip plates, and locking plates) in Vancouver
type AG periprosthetic femoral fractures. They reported that locking plate versus cable fixation and grip plate fixation versus cable fixation showed statistically significant superior results in axial distraction tests. They concluded that Vancouver type AG fractures may be treated with either grip plate fixation or locking plates, with the former ensuring more stable osteosynthesis.
Masri et al. published a review article on the evaluation and management of periprosthetic fractures. They reported that the best outcome is achieved when the surgeon has a thorough understanding of the principles of treatment of periprosthetic fractures with access to various fixation and prosthetic devices. They concluded that the Vancouver classification offers a reproducible description of these factors and easily guides treatment.
Figure A depicts a displaced greater trochanteric periprosthetic fracture. Illustration A depicts the Vancouver periprosthetic hip classification system. Illustration B depicts an example of a Vancouver AG fracture treated with ORIF using a claw plate.
Incorrect Answers:
Answers 1 & 3: Displaced Vancouver AG fractures should be treated with ORIF using either trochanteric cables, claw plates, or locking plates.
Answer 4: A Vancouver AL fractures occur at the lesser trochanter, not the greater trochanter.
Answer 5: Vancouver B1 fractures occur around or just below a well-fixed stem. They are treated with ORIF.
OrthoCash 2020
-
-
A 79-year-old man sustains a fall and presents with the injury depicted in Figures A and B. He underwent total knee arthroplasty (TKA) 5 days ago and had been doing well prior to his recent fall. What is the TKA implant design and what is the most appropriate treatment?
-
Cruciate-retaining; Open reduction internal fixation with lateral locking plate
-
Cruciate-retaining; Retrograde femoral nail
-
Cruciate-retaining; Open reduction internal fixation with medial locking plate
-
Posterior-stabilized; Open reduction internal fixation with lateral locking plate
-
Posterior-stabilized; Femoral component revision Corrent answer: 4
The TKA prosthesis in question is a posterior-stabilized (PS) design. Open reduction and internal fixation (ORIF) with a lateral locking plate is a viable treatment option for a periprosthetic femur fracture around the femoral component of a well-fixed PS TKA.
Femoral periprosthetic fractures after TKA may occur following low-energy trauma in osteopenic bone. Nondisplaced fractures with a stable prosthesis may be treated nonoperatively in a cast or brace. The decision for revision of the femoral component is guided by component stability versus loosening, with ORIF and revision arthroplasty indicated, respectively. ORIF options include locked femoral plating or retrograde intramedullary nailing (IMN).
Retrograde IMN is not a viable option for stemmed femoral prosthesis and posterior-stabilized (PS) TKA systems without an open box design.
Haidukewych et al. published an instructional course lecture on periprosthetic
fractures of the hip and knee. They reported internal fixation is indicated for the majority of periprosthetic distal femoral fractures. Both locked plates and retrograde IMNs can provide good outcomes, and that revision arthroplasty is indicated in fractures around loose components, nonunions, or fractures for which internal fixation attempts are likely to fail.
Su et al. published a review on periprosthetic femoral fractures above total knee replacements. They reported that periprosthetic femoral fractures above TKAs can be managed by a variety of methods, including casting, ORIF, external fixation, or revision arthroplasty. They highlighted that classification based on fracture location helps guide treatment. They concluded that IMNs are best for proximal fractures, fixed-angle devices for fractures originating at the component, and revision arthroplasty for very distal fractures or those with component loosening.
Figures A and B depict a periprosthetic femoral fracture originating at the anterior flange of the femoral component of a PS-TKA. Illustrations A and B depict radiographs of the periprosthetic femur fracture after ORIF with a lateral locking plate.
Incorrect Answers:
Answers 1, 2 & 3: The femoral component has a box, making the cruciate-retaining (CR) design incorrect.
Answer 5: Femoral component revision is unnecessary as the femoral component appears well fixed.
OrthoCash 2020
-
-
A 67-year-old woman presents with chronic right hip pain, exacerbated by long walks. She has limited hip range of motion, particularly in flexion and internal rotation. Radiographs are depicted in Figure A. This is her first time seeking treatment. What is the mechanism of action of a medication strongly recommended for short-term pain relief according to the most recent (2013) AAOS Clinical Practice Guidelines?
-
Direct action on hypothalamic regulating center with anti-pyretic effects
-
Binds to cannabinoid receptors in neural tissues
-
Binds directly to nuclear receptors to interrupt the inflammatory and immune cascade via mRNA changes
-
Maintains synovial fluid viscosity and supports articular cartilage shock absorption
-
Inhibits sodium ion channels to inhibit sensory nerve impulse initiation and conduction
Corrent answer: 3
This patient presents with right hip osteoarthritis. In the 2013 AAOS Clinical Practice Guidelines (CPG), intraarticular corticosteroids usage in improving function and pain reduction in the short-term for patients with symptomatic osteoarthritis of the hip was strongly recommended. Corticosteroids function by direct binding to nuclear steroid receptors to interrupt the inflammatory cascade through mRNA changes.
The 2013 AAOS CPG for the treatment of symptomatic arthritis discuss both operative and non-operative treatment options and scrutinizes the literature for each modality. Amongst the strong recommendations are weight loss, low impact physical activity, and non-narcotic analgesia including nonsteroidal anti-inflammatory drugs (NSAIDs). Numerous modalities were not supported including the use of prescription opioids, acupuncture, needle lavage, hyaluronic acid, glucosamine and chondroitin, and arthroscopic lavage.
Dieppe et al. published a review article on the management of hip osteoarthritis. They reported that shock absorbing shoe insoles and walking sticks can be of great benefit, while physiotherapy and hydrotherapy should be considered for more severe cases. They recommended simple analgesics such as paracetamol and NSAIDs as first-line treatment, with joint replacement considered in patients with severe pain or disability.
Quinn et al. published a review article on the management of hip osteoarthritis using the AAOS Appropriate Use Criteria (AUC). Evidence-based information, in conjunction with the clinical expertise of physicians, was used to develop the criteria to improve patient care and obtain the best outcomes while considering the subtleties and distinctions necessary in making clinical decisions.
Figure A depicts right hip osteoarthritis with evidence osteophytes, narrowed joint space, and subchondral sclerosis.
Incorrect Answers:
Answer 1: Tylenol is not strongly supported by the AAOS CPG.
Answer 2: Cannabinoids like marinol is not strongly supported by the AAOS CPG.
Answer 4: Hyaluronic acid is not strongly supported by the AAOS CPG. Answer 5: Lidocaine is not strongly supported by the AAOS CPG.
OrthoCash 2020
-
-
A 70-year-old man presents with chronic persistent right knee pain and erythema which has been present for 7 weeks after having undergone total knee arthroplasty (TKA) 7 years ago. He is referred after completing a course of oral antibiotics prescribed by his primary care physician, which did not improve his symptoms. His current radiograph is shown in Figure A. Laboratory testing reveals a serum C-reactive protein (CRP) of 50mg/L and an erythrocyte sedimentation rate (ESR) of 67 mm/h. Arthrocentesis is performed and reveals a synovial WBC of 1,500 WBC/uL, with 85% polymorphonuclear cells (PMNs), and negative final cultures. The alpha-defensin test is positive. What is the next best step?
-
Repeat knee arthrocentesis after 2-week antibiotic holiday
-
Revision of femoral component without antibiotic therapy
-
One-stage revision arthroplasty with intravenous antibiotic therapy for 4-6 weeks
-
Two-stage revision arthroplasty with intravenous antibiotic therapy for 4-6 weeks
-
Revision of tibial component without antibiotic therapy
Corrent answer: 4
Based on the 2018 revised Musculoskeletal Infection Sociecty (MSIS) criteria, the elevated ESR, serum CRP, positive alpha defensin, and elevated PMNs convey a score of 1, 2, 3, and 2, respectively, for a combined score >6. This is diagnostic of a chronic prosthetic joint infection (PJI), for the most supported management strategy is two-stage revision arthroplasty with intravenous (IV) antibiotic therapy for 4-6 weeks.
PJI after TKA is estimated to be 2.5%. The chronicity of the infection determines management. Non-MRSA PJI within 4 weeks of surgery is considered acute and may be treated with irrigation, debridement, polyethylene exchange, component retention, and IV antibiotics, as the organism has purportedly had too little time to form a robust biofilm. PJI occurring more than 4 weeks after TKA is considered chronic and, due to a high likelihood of biofilm formation, requires two-stage revision arthroplasty with IV antibiotic therapy.
Ting et al. reviewed an algorithm-based approach for diagnosis of PJI. They reported that the diagnosis of PJI is made in 90% of patients by ESR and CRP, followed by arthrocentesis if the results are high, with a focus on synovial WBC count, differential, and cultures.
Everhart et al. developed and validated a preoperative surgical site infection (SSI) risk score for primary or revision TKA and hip arthroplasty (THA). They reported that patient comorbidities composing the risk significantly influence SSI risk for primary or revision TKA and THA. They concluded that preoperative SSI risk can be objectively determined by the proposed SSI risk score.
Parvizi et al. most recently presented the 2018 updated MSIS evidence-based criteria for diagnosis of periprosthetic hip and knee infections (Illustration A). The authors updated the original crtieria, expanding and refining the contributions from each of the minor criteria. The authors supported that a score >6 was diagnostic of periprosthetic infection. They concluded that this criteria was 97.7% sensitive and 99.5% specific for diagnosis of prosthetic joint infection.
Figures and Illustrations:
Figure A shows the AP and lateral radiographs of a TKA prosthesis with evidence of osteolysis and marked loosening of the femoral and tibial components.
Illustration A is the updated 2018 MSIS criteria for diagnosis of a PJI.
Incorrect Answers:
Answer 1: A repeat knee arthrocentesis is not indicated given the clinical history and objective laboratory data suggestive of a PJI.
Answer 2: Revision of loosened femoral component without antibiotic therapy would be indicated in TKA with aseptic femoral loosening, not with a PJI. Answer 3: A one-stage revision is not the accepted standard of treatment and is not as successful in the clearance of pathogens in PJI compared to two-stage revision. While there is increasing support in the literature for one-stage revision, the current gold standard for a chronic PJI remains a two-stage revision.
Answer 5: Revision of loosened tibial component without antibiotic therapy would be indicated in TKA with aseptic tibial loosening, not with a PJI.
OrthoCash 2020
-
-
Figure A depicts the intraoperative findings during a revision total hip arthroplasty (THA) in a patient with chronic pain for the last two years after undergoing his index THA 10 years ago. Workup for infection was negative. The acetabular and femoral components are assessed to be well-fixed intraoperatively. Which of the following describes the best treatment option?
-
Suppressive intravenous antibiotics
-
Two-stage revision arthroplasty
-
Irrigation and debridement
-
Acetabular component revision
-
Revision to new ceramic femoral head with titanium sleeve Corrent answer: 5
This patient's symptoms and intraoperative image are consistent with trunnionosis. As the components appear well-fixed, the best treatment is a revision to a ceramic head with a titanium sleeve.
Metal-on-metal (MoM) total hip arthroplasty (THA) has been associated with complications from metal debris and toxicity. Although morse taper technology allows machined taper trunnion to fit with the femoral head, complications have been reported including corrosion at the trunnion, which results in pain and is often associated with adverse local tissue reactions (ALTR). During revision surgery, a ceramic head with a titanium sleeve adaptor is advocated, as an exchange of the metal femoral head to another metal femoral head may result in recurrence of ALTR. Lastly, while no cutoffs for serum cobalt chromium ion levels have been identified as pathognomonic for trunnion corrosion, a serum cobalt level of 1.6 ng/mL (ppb) and greater has been suggested as a threshold for mechanically-assisted crevice corrosion.
Weiser et al. published a current concepts review of trunnionosis in THA. They stated that the complication of trunnionosis in THA is likely underreported since it often causes concurrent osteolysis and loosening, which are more universally accepted diagnoses. They recommended analysis of serum cobalt and chromium ions as well as metal artifact reduction MRI during workup. In revision surgery, they advocated for head and liner exchange, with retention
of the acetabular and femoral components.
Raju et al. published a case series on trunnionosis in metal-on-polyethylene (MoP)THA. They reported three failures (two dissociations of the femoral head from the neck), with the most likely contributing factors to failure being a large femoral head size, high horizontal offset, a low angled neck, and a titanium alloy taper with a cobalt-chromium head. They recommended high vigilance for any alteration of alignment between the femoral head and neck in follow-up radiographs after THA.
Figure A is an intraoperative image depicting severe corrosion at the trunnion in a metal-on-metal THA.
Incorrect Answers:
Answer 1: Intravenous antibiotics are not indicated as a workup for periprosthetic joint infection is negative.
Answer 2: Two-stage revision may be appropriate in the setting of infection but a single stage revision is appropriate for trunnionosis.
Answer 3: Irrigation and debridement would not address the corrosion at the taper, resulting in recurrent ALTR.
Answer 4: Acetabular component revision is not indicated given the acetabular component is well fixed.
OrthoCash 2020
-
-
Figure A depicts the current radiograph of a 66-year-old man with significant right groin pain after undergoing right total hip arthroplasty (THA) 10 years ago. Revision surgery is planned after infection workup is negative. What is the classification of his diagnosis and what would the most appropriate treatment for the acetabulum?
-
Paprosky I; cementless hemispheric cup with screw fixation
-
Paprosky I; cemented hemispheric cup without screw fixation
-
Paprosky IIB; cementless hemispheric cup with screw fixation
-
Paprosky IIIA; cup/cage construct
-
Paprosky IIIA; triflange reconstruction Corrent answer: 3
This patient demonstrates superior acetabular rim loss and superolateral migration that can be characterized as Paprosky IIB in the Paprosky classification for acetabular bone loss. This may be managed with a hemispheric acetabular cup with screw fixation.
Acetabular bone loss poses a technical challenge in THA. The Paprosky classification for acetabular bone loss helps guide treatment for revision THA. Broadly speaking, Paprosky Type I and II defects may be managed with a porous-coated hemisphere cup secured with screws, and Type III defects managed with reconstruction cages protected with cups, structural augments, or custom triflange implants.
Sheth et al. published a review article on the evaluation and management of acetabular bone loss in revision THA. They reported that appropriate radiographs are key in quantifying acetabular bone loss, and specific classification schemes can assist in identifying bone loss patterns which guide available treatment options. They concluded that depending on the severity of bone loss, treatment may include impaction grafting and acetabular cementation, cementless hemispheric acetabular reconstruction, structural allograft reconstruction, cementless reconstruction with modular porous metal
augments, ring and cage reconstruction, cup-cage reconstruction, and triflange reconstruction.
Paprosky et al. performed a 6-year follow up evaluation study on acetabular defect classification and surgical reconstruction in revision THA. They typed acetabular defects from 1 to 3 and reconstructed with bulk or support allograft depending on the type. They concluded that the size, orientation, and method of fixation of the allografts utilized during revision THA play a pivotal role in the integrity of structural allografts, and stressed the importance of adequate host-bone to ensure solid bone ingrowth.
Dennis et al. published on the outcomes after Paprosky Type III acetabular bone loss reconstructed using custom triflanged acetabular components. They reported stable fixation and reconstruction of periacetabular bone in over 80% of patients at short-term follow up. They recommended that this technique be used with caution in cases of preoperative hemipelvis dissociation unless additional column plating is performed.
Figure A demonstrates Paprosky IIB acetabular bone loss with superolateral migration of the acetabular component. Illustration A depicts the Paprosky classification.
Incorrect Answers:
Answers 1 & 2: This patient has at least Paprosky II acetabular bone loss as the acetabular rim is not intact.
Answer 4 & 5: This patient's acetabular rim is partially supportive, making a IIIA defect incorrect.
OrthoCash 2020
-
-
A 72-year-old male presents with worsening left hip pain 12 years after total hip arthroplasty. On examination, the patient has a Trendelenburg gait with a 3.5 cm leg length discrepancy. The patient denies any fevers or chills. Current radiographs are shown in figure A. Recent ESR and CRP are 21 mm/hr and 1.2 mg/L, respectively. What is the preferred treatment option to address these findings?
-
Large porous hemispheric cup with particulate bone graft and augmented with screw fixation
-
Custom triflanged acetabular component
-
Cemented large porous hemispheric cup
-
Metal augments with large porous hemispheric cup and bone grafting combined with screw augmentation
-
2-stage revision arthroplasty Corrent answer: 4
The patient is presenting with a Paprosky type IIIA acetabular defect with the migration of the hip center in a superolateral direction. The preferred treatment option would be to provide structural stability of the cup with metal augments combined with bone grafting and cement reinforcement and screw fixation.
Acetabular bone loss can make revision total hip arthroplasty challenging due to lack of structural support of the acetabular cup as well as concerns for bony ingrowth potentially compromising implant longevity. The Paprosky classification was designed to identify the location and degree of acetabular bone loss and thereby to guide treatment of the respective defects. In type IIIA defects there is bone loss of the superolateral acetabulum with greater than 3 cm migration of the center of the femoral head, also described as "up and out." Intraoperatively structural support must be reestablished for the revision cup by either structural allograft or metal augments. Further bone grafting is performed to enhance long term bone ingrowth of the prosthesis.
Sheth et al. performed a literature review on the evaluation and management of acetabular bone loss in revision total hip arthroplasty. The authors reviewed the Paprosky classification for acetabular bone loss and recommended the use of noncemented, porous-coated, hemispheric cups with adjunctive screw fixation in type I, IIA, and IIB defects. For type IIC defects, the authors recommended highly porous, noncemented, hemispheric cups with screw fixation and bone grafting of the medial wall defect. For type IIIA defects, they advocated for the use of metal augments or structural allograft combined with porous hemispheric cups and augmentation with screw fixation and cement.
Lastly, for type IIIB defects, they recommended the use of noncemented acetabular devices combined with structural allograft, structural augments, and a reconstruction cage.
Paprosky et al. performed a retrospective study of patients undergoing revision total hip arthroplasty and proposed a classification system of acetabular bone loss, recommending treatment options for each type. The authors found that of the 147 implants included in the study, only 6 required repeat revision, all of which were type IIIB defects. The authors concluded that adherence to this classification system and the recommended reconstruction techniques can produce acceptable and predictable results in acetabular revision surgery.
Dennis et al. performed a retrospective review of twenty-four patients with Paprosky type IIIB acetabular defects treated with a custom triflanged acetabular component (CTAC). The authors found that of the twenty-four patients treated, three (87.5%) were considered to have radiographic and clinical signs of failure, with one requiring resection arthroplasty. There were two hip dislocations necessitating only closed reduction. The authors concluded that CTAC is an effective means to treat type IIIB acetabular defects, but should be used with caution in cases of pelvic discontinuity unless additional column plating performed.
Figure A is the AP radiograph of a pelvis with a Paprosky type IIIA acetabular defect. Illustration A is the post-op radiograph after reconstruction with metal augments and large porous hemispheric cup. Illustration B is a table with the description of the Paprosky classification. Illustration C is a diagram depicting the Paprosky classification.
Incorrect Answers:
Answer 1: The use of particulate bone graft with a large porous hemispheric cup would be more appropriate for a type IIA or IIB defect. The use of particulate bone graft does not afford enough stability to support the cup long enough for bone ingrowth in type IIIA defects.
Answer 2: Custom triflanged acetabular components would be preferred for the treatment of type IIIB defects where there is disruption of the medial wall. Answer 3: The use of cement fixation in acetabular revision is not recommended due to high failure rates. Cement is strongest in compression, but the acetabular cement mantle is exposed to shear forces, which weaken the cement-bone and cement-prosthesis interface over time.
Answer 5: The patient does not have clinical or laboratory findings suggestive of a prosthetic joint infection, which would make a 2-stage revision inappropriate in this patient.
OrthoCash 2020
-
-
A 67-year-old woman with poliomyelitis presents with quadriceps weakness and chronic right knee pain for the last 2 years. She is scheduled to undergo right total knee arthroplasty (TKA) after failing nonoperative modalities. Her preoperative radiographs are shown in Figures A and B. What technique should be utilized to optimize her function and to prevent the recurrence of her deformity?
-
Posterior stabilized design with under-resection of distal femur
-
Posterior stabilized design with under-resection of proximal tibia
-
Posterior stabilized design with under-resection of posterior femur
-
Varus-valgus constrained design
-
Rotating hinge design
Corrent answer: 5
This patient with neuromuscular disease has genu valgum with recurvatum (hyperextension). Of the techniques listed above, utilization of a rotating hinge TKA design would most likely optimize her function and prevent recurrence of her recurvatum deformity because of the implant’s extension stop.
Genu recurvatum is associated with deformities such as genu valgum, ligamentous laxity, and neuromuscular diseases, which are often accompanied by equinus ankle contractures. Knee hyperextension is likely to recur after TKA in patients with neuromuscular disorders such as poliomyelitis due to the bony deformity, muscle weakness, and paralysis seen in these patients. Several strategies to correct knee hyperextension at the time of primary TKA have been described and include posterior capsular plication, proximal and posterior transfer of collateral ligaments, under-resection of distal femur and proximal tibia, and the use of thicker components. In severe cases where ligament integrity is likely compromised, utilization of more constrained prostheses is recommended.
Giori et al. performed a retrospective study of patients with poliomyelitis involving a limb that underwent primary TKA. Complications reported included two periprosthetic fractures, one peroneal nerve palsy, one patellar tendon avulsion, and four cases of recurrent instability, all attributable to the poor bone quality, valgus deformity, patella baja, poor musculature, and attenuated soft tissues observed in knees with poliomyelitis. They concluded that pain and knee scores improved following TKA this cohort of patients and recurrence of instability and progressive functional deterioration is possible postoperatively.
Paratte et al. published an AAOS Instructional Course Lecture on instability after TKA. They reported knee hyperextension before TKA is seen in <1% of patients and is most commonly seen in patients with neuromuscular disease like poliomyelitis. They recommended solutions to be considered for such patients such as distal femur under-resection, distal femoral augmentation blocks with the knee left with a slight flexion contracture, translation of the femoral origins of the medial collateral ligament and lateral collateral ligaments proximally and posteriorly to recreate the normal tightening action during full extension of the knee, and the use of a rotating-hinge total knee prosthesis with an extension stop.
Meding et al. published a review article on the etiology and surgical treatment of genu recurvatum during TKA. They emphasized the importance of diagnosing and elucidating the etiology of the hyperextension deformity prior to surgery since the deformity is known to recur in patients with certain neuromuscular disorders. They recommended a meticulous approach and
avoiding even mild degrees of residual instability in the coronal plane at surgery since this is associated with increased extension in the postoperative period.
Figures A and B depict severe right knee osteoarthritis with valgus and recurvatum deformities. Illustration A depicts a rotating-hinge TKA prosthesis.
Incorrect Answers:
Answers 1, 2 & 3: While a posterior stabilized design would increase the level of constraint, this alone would be insufficient for the recurvatum deformity, even with conservative distal femur and proximal tibia resections.
Answer 3: While a posterior stabilized design would increase the level of constraint, this alone would be insufficient for the recurvatum deformity. Furthermore, under-resection of the posterior femur selectively influences the flexion space and would result in a knee tight in flexion.
Answer 4: A varus-valgus constrained design alone would not address the recurvatum deformity.
OrthoCash 2020
-
-
A 57-year-old man with a history of chronic lower back pain and right hip arthritis is postoperative day 2 from an uncomplicated right total hip arthroplasty with a spinal block. Since the procedure, he has reported persistent pain in his right leg with a focal point in the proximal lateral leg. He has had difficulty getting out of bed for physical therapy due to pain reproduced in his leg. He is voiding but has not yet had a bowel movement. Physical exam is only significant for decreased ankle dorsiflexion strength on the right. Plantarflexion strength remains 5/5 bilaterally. No point tenderness was elicited and Homan's sign is negative. His wound is unremarkable with typical post-operative swelling of the leg and no significant drainage. He has
been receiving ASA 81mg PO daily since surgery and has been wearing compression stockings full-time. Postoperative repeat radiographs of the hip are unremarkable and his hemoglobin is stable. Which of the
following etiologies is most likely responsible for this patient's symptoms?
-
Residual effect of the spinal block
-
Lumbar lateral recess stenosis
-
Gluteal hematoma
-
Acute post-operative infection
-
Venous thomboembolism
Corrent answer: 2
During total hip arthroplasty, a "double crush" injury can occur to the sciatic nerve in the presence of pre-existing degenerative lumbar spondylosis, leading to persistent pain and post-operative motor weakness. The best study would be a lumbar MRI to evaluate for lumbar spinal stenosis.
Nerve injuries following total hip arthroplasty are rare and usually affect the sciatic nerve. The peroneal branch appears to be the most commonly affected due to its more superficial and lateral position, more the tightly packed fascicles, and greater adherence to the surrounding tissues compared to the tibial division. Lumbar degenerative disc disease commonly occurs in the presence of coxarthrosis, with spinal stenosis being exacerbated by traction neurapraxia during the procedure.
DeHart et al. reviewed nerve injuries and postoperative management. The authors stated that the sciatic nerve is the most commonly injured nerve, with up to 70% of cases have subclinical electrodiagnostic changes. The cause of this is multifactorial, but the vast majority of studies reviewed reported complete spontaneous recovery by 6-12 months. The authors recommend observation of the nerve deficit with ankle-foot-orthosis and follow-up EMG to determine the level of the injury.
Pritchett performed a review of 21 patients that presented with a foot drop after total hip arthroplasty. All patients reported prior back and leg pain prior to the procedure, with post-op MRI demonstrating severe spinal stenosis. The author postulated there to be a double crush phenomenon, with patients undergoing lumbar laminectomy having improvement or complete resolution. The author concluded that select patients presenting with foot drop following THA may benefit from a lumbar laminectomy.
Incorrect Answers:
Answer 1: Spinal blocks with monitored anesthesia care is a popular anesthetic plan for arthroplasty as it reduces postoperative pneumonia rates, decreases blood loss, and decreases postoperative admission. Unilateral motor deficits
and paresthesias are uncommon after a spinal block but have been reported with regional anesthesia, such as a sciatic nerve block. Furthermore a spinal would be unlikely to have any residual effect still post-operative day 2.
Answer 3: A gluteal hematoma may cause sciatic nerve palsy which could present as a foot drop. However, without sufficient swelling at the surgical site, this is unlikely to be the case.
Answer 4: Acute post-operative infection is very unlikely post-operative day 2, especially in the absence of any clinical signs or symptoms of aggressive pathology.
Answer 5: Deep vein thrombosis usually presents with swelling, edema and sometimes erythema of the affected extremity.
OrthoCash 2020
-
-
A 65-year-old female presents to the clinic with isolated medial-sided left knee pain. She has since exhausted conservative management but remains persistently symptomatic. The physical exam and radiographic work-up demonstrates isolated medial tibiofemoral compartment involvement. After discussion of the surgical options, she undergoes the procedure shown in Figure A. She initially does well but returns to clinic 3 months post-operatively with significantly increased medial-sided knee pain and the injury shown in Figure B. All of the following technical errors likely contributed to this complication EXCEPT?
-
Excessive force impacting the tibial component
-
Penetration of the posterior tibial cortex with proximal guide pin
-
Placement of a peripheral medial cortical guide pin
-
Tibial resection guide replacement with re-drilling of the two proximal guide holes
-
Under-sizing of the tibial component Corrent answer: 1
The patient presents with a periprosthetic tibial stress fracture following a medial unicompartmental knee arthroplasty. Excessive force used when impacting the tibial component could potentially lead to intra-operative fracture, but this would be recognized in the acute post-operative period.
With continuing advances in surgical technique, UKA has demonstrated increasingly promising midterm outcomes. However, there are number of technical considerations that directly impact survivorship and the potential for post-operative complications. While aseptic loosening remains the most common mode of early failure necessitating conversion to TKA, literature is replete with reports of early failure secondary to proximal tibia stress fractures. Stress fractures have been linked to a number of largely non-modifiable patient characteristics to include bone quality, but technical errors remain a controllable contributing factor. Recent studies have found that excessive guide pin number and suboptimal placement for the tibial resection guide as well as tibial component undersizing are associated with increased proximal tibial mechanical stress and may result in fracture.
Brumby et al. described a series of tibial plateau stress fractures subsequent to UKA. The authors attributed these to mechanical weakening of the proximal tibia from the guide pin and lug holes drilled for the guide and tibial component, respectively. They noted that this even occurred in some cases with penetration of the medial tibial cortex by a single pin. Stress fractures presented at a median of 8 weeks post-operatively and in all cases required revision to TKA. They recommended post-operative monitoring of patients in whom a guide with 3 or more pins was utilized, or with any peripheral pins that breach the medial tibial cortex.
Vince et al. present a review of the evolution, indications, and outcomes following UKA. The authors specifically highlight guide pin holes as a major contributing factor to periprosthetic fractures. They recommend limiting the number of holes drilled for placement of the tibial resection guides and paying careful attention to placement, specifically with regard to violation of the medial cortex. Furthermore, the authors advocated that a single proximal pin, placed centrally, and the guide secured distally by the ankle clamp would be
sufficient for stability and alignment while avoiding increased stress on the proximal tibia.
Figure A is an AP weight-bearing radiograph of the left knee demonstrating isolated medial tibiofemoral arthritis.
Figure B is a post-operative radiograph of the left knee significant for a medial UKA. Figure C is a follow-up AP radiograph of the left knee demonstrating a periprosthetic fracture involving the medial UKA with significant varus collapse.
Incorrect answers:
Answer 2: Penetration of the posterior tibial cortex with guide pins is not recommended, as this has been found to increase proximal tibial mechanical stress and potentially lead to periprosthetic stress fracture.
Answer 3: Peripheral or medial guide pins should be avoided as well, given that placement results in mechanical weakening of the medial cortex and can contribute to stress fracture. Studies have found stress fractures related to even a single pin hole that violated the medial cortex.
Answer 4: Replacing a tibial resection guide and re-drilling the guide holes entails a minimum of 3-4 holes within the proximal tibia. A greater number of holes, especially 3 or more, has been suggested to convey an increased risk of stress fracture.
Answer 5: Undersizing the tibial component results in greater stress concentration over a smaller area, leading to possible subsidence of the implant, loosening, or stress fracture.
OrthoCash 2020
-
-
A 65-year-old male presents to your clinic for evaluation of right hip pain. He underwent a right total hip arthroplasty (THA) 20 years prior and was doing very well until 2 years ago. He admits to groin pain when getting up from a seated position. He denies any fevers or chills. Radiograph is shown in Figure A. Which of the following would preclude the patient from undergoing a single-stage surgical intervention without further workup?
-
Elevated serum cobalt
-
Metallosis noted intra-operatively
-
Significantly higher serum cobalt then serum chrome levels
-
Elevated ESR and CRP
-
Pseudotumor noted on MRI Corrent answer: 4
An elevated ESR and CRP are screening labs used to determine if further workup is required to rule out a periprosthetic joint infection (PJI). A patient with an elevated ESR and CRP should thus undergo further workup including a joint aspiration prior to consideration of a single-stage revision THA.
The differential diagnosis of pain after THA encompasses a number of etiologies. While radiographs may point to loosening of the stem or the cup, osteolysis or a stress fracture, the first step in management needs to evaluate for a PJI. Accordingly, ESR and CRP are logical next screening steps in the workup. If elevated, additional studies need to be obtained such as joint aspiration with manual diff (to look for elevated synovial WBC, synovial PMNs or a positive culture), immunoassays (alpha-defensive, leukocyte esterase colorimetric strip) and serum IL-6. If the additional workup for infection is negative, it is safe to proceed with a single-stage revision THA. Metal on metal THA have their own set of modes of failure leading to bony erosion and pain.
The majority of these are aseptic and allow for a single-stage revision including metallosis, pseudotumor, and metal hypersensitivity.
Parvizi et al. performed a retrospective cohort study to examine the effectiveness of surgical treatment in treating hip and knee PJI caused by MRSA. They looked at 127 patients with a minimum of 2 years follow-up or until recurrence of PJI. In 35 patients, only an I&D with prosthetic retention was performed while a 2 stage explantation and reimplantation was performed in the other 92 patients. Of those who underwent an I&D and implant retention, only 37% of cases had successful eradication of the infection
whereas two-stage exchange arthroplasty controlled the infection in 75% of hips and 60% of knees in the other 92 patients. Furthermore, cardiac disease was associated with a higher likelihood of failure to control infection in all treatment groups.
Shukla et al. evaluated 87 hips with a PJI that were treated with explantation, antibiotic spacer and 6 weeks of antibiotics. The authors looked at ESR and CRP before reimplantation and obtained synovial WBC at the time of reimplantation. The authors noted 9 hips (10.1%) had persistent infections at the time of re-implantation. The mean ESR, CRP, and synovial fluid WBC count had significantly decreased between stages; however, the ESR remained elevated in 50 patients (62.5%) and the CRP remained elevated in 22 patients (27.5%) in whom the infection had been eradicated. The authors noted that the synovial fluid WBC count was the best test for identifying persistent infection, with an optimum cutoff of 3528 WBCs/microL (sensitivity, 78%; specificity, 96%).
Browne et al. evaluated 37 patients with metal on metal THA or resurfacing arthroplasties who underwent revision to determine the clinical, radiographic, laboratory, intraoperative, and histopathologic findings to determine the cause of failure. Of the 37 patients, 10 were revised due to metal on metal hypersensitivity, 8 due to chronic inflammation with lymphocytic infiltration, 8 with aseptic loosening, 2 with iliopsoas impingement, 3 with femoral neck fracture after resurfacing arthroplasty and 6 due to infection, instability, and periprosthetic fracture. The authors stressed increased awareness of the wide variety of modes of failure associated with metal-on-metal articulations.
Figure A is an AP pelvis showing a characteristic appearance of a metal-on-metal THA with a large femoral head.
Incorrect Answers:
Answer 1: Elevated serum cobalt can be indicated of metal on metal wear but this would not preclude single-stage intervention.
Answer 2: Metallosis can be a common finding intra-operatively in a patient with a prior metal-on-metal THA. This would not prevent one from performing a single-stage intervention.
Answer 3: A significantly elevated Co/Cr ratio has been shown to point towards corrosion of the trunion (Trunnionosis). This would not prevent a single-stage intervention.
Answer 5: A pseudotumor is a non-infected, non-neoplastic lesion that can form in the vicinity of a metal-on-metal THA that can be associated with pain and or bony erosion.
OrthoCash 2020
-
-
The use of a high-offset femoral stem leads to which of the following changes with regard to total hip arthroplasty?
-
Increased joint reactive forces
-
Increased leg length
-
Increased risk of acetabular component loosening
-
Increased soft tissue tension
-
A higher rate of dislocation Corrent answer: 4
A high-offset femoral stem leads to increased soft tissue tension without affecting leg length.
Femoral offset is defined as the distance from the femoral head center of rotation to the center of the long axis of the femur. Restoration of offset improves overall arthroplasty biomechanics with decreased cup strain and polyethylene wear, decreased dislocation risk, increased hip abductor strength, and lower rates of postoperative limp. The drawback of too much femoral offset is an increased risk of lateral prominence and subsequent trochanteric bursitis.
Lecerf et al. reviewed the femoral offset with regard to total hip arthroplasty. They state femoral offset does correlate closely with the hip abductor lever arm and hip abductor strength. They conclude that femoral offset is important for improved hip function and longevity after total hip arthroplasty.
Flecher et al. reviewed limb lengthening as it pertains to total hip arthroplasty. They discuss methods of assessing limb length including EOS and CT imaging as well as intraoperative robotic or computer assistance. They comment that the expectations of limb function after total hip make it crucial for surgeons to understand the three-dimensional geometry and placement of prostheses.
Illustration A shows an example of a standard and high offset stem and how it maintains leg length while increasing offset.
Incorrect Answers:
Answer 1: A high-offset stem leads to decreased joint reactive forces. Answer 2: Using a high-offset stem does not affect the leg length.
Answer 3: The use of a high-offset stem does not increase the risk of acetabular component loosening. A lateralized acetabular liner does increase the risk of acetabular component loosening.
Answer 5: A high-offset stem leads to increased soft tissue tensioning and a lower rate of dislocation
OrthoCash 2020
-
-
You are currently evaluating a 68-year-old woman who has met indications for a total knee arthroplasty. You finally have finished documenting the patient’s extensively detailed social and family history. As you finish, the patient tells you that she has spoken with her friends regarding her knee and tells you that she wants a prescription for preoperative physical therapy, would like a drain placed, an order for a cryotherapy device and use of a patient-controlled analgesia (PCA), in addition to being mobilized with physical therapy on the day of surgery. As an astute resident you inform the patient that based on the current AAOS clinical practice guidelines (CPG), there is strong evidence against using which of the patient’s request?
-
Preoperative physical therapy
-
Drain placement
-
Cryotherapy device
-
Postoperative day 0 mobilization
-
Patient-controlled analgesia
Corrent answer: 2
Based on the most recently published AAOS CPG, there is strong evidence to support not using a drain with total knee arthroplasty (TKA) because there is not any difference in complications or outcomes.
The use of a drain was postulated to aid in decreasing postoperative infection, swelling, blood transfusions, hematoma formation, pain, length of hospital stay, and re-operation rates. In addition, their use was meant to improve
postoperative range of motion. However, after review of the high- and moderate-quality studies and with input from the multiple orthopaedic and medical societies, there has not been any clear advantage to the use of drains after unilateral total knee arthroplasty when comparing complication profiles and outcomes.
McGrory et al. published a systematic review on the surgical management of osteoarthritis of the knee. In an effort to improve the surgical management of patients with osteoarthritis, the authors provide 38 evidence-based recommendations on topics specific to the preoperative, perioperative and postoperative treatment of such patients. Additionally, each topic was further classified based on the level of evidence available (limited, moderate, and strong) to support or not support a give recommendation.
MacDonald et al. completed a prospective randomized clinical trial using continuous passive motion (CPM) following TKA. The patients were separated into two separate treatment groups plus a control group. The patients were followed over the course of one year from surgery. Similar to postoperative drain placement following TKA, MacDonald et al. were unable to demonstrate any significant difference in outcomes for CPM versus no CPM.
Incorrect Answers:
Answer 1: There is limited evidence to support that supervised exercise before TKA might improve pain and physical function after surgery.
Answer 3: There is moderate evidence to support that the use of cryotherapy devices after TKA does not improve outcomes.
Answer 4: There is strong evidence to support that rehabilitation started on the day of TKA reduces the length of hospital stay. Also, there is moderate evidence to support that when rehabilitation is started on the day of TKA there is a reduction in pain and an improvement in function it comparison to starting on postoperative day 1.
Answer 5: There are not any current recommendations regarding PCA use.
OrthoCash 2020
-
-
Figure A is the radiograph of a male who fell down the stairs. He is 8 years status post right total hip arthroplasty. All of the following are indications for a proximal femoral replacement EXCEPT?
-
<4cm of diaphyseal cortical bone
-
Age <50 years
-
Extensive metadiaphyseal proximal bone loss with <4cm of intact isthmic bone
-
Nonunion of the proximal femur with multiple failed attempts at osteosynthesis
-
Paprosky IV femoral bone loss Corrent answer: 2
The radiograph demonstrates a Paprosky type IV femoral deficiency. Given the substantial bone loss with limited proximal femoral support, a proximal femoral replacement is recommended.
Postoperative fractures around a total hip prosthesis has an incidence of 0.1% and occurs most commonly at the tip of the stem. Proximal femoral support is important to evaluate following a periprosthetic fracture of the hip. The Paprosky classification of femoral bone loss helps guide treatment. A Paprosky type IIIb or IV femoral deficiency would benefit from either an allograft prosthetic component or a megaprosthesis/modular oncology component.
Additionally, impaction bone grafting is indicated with a large canal and thin cortices for Paprosky IIIb and IV defects. Revising the femur to a proximal
femoral replacement would allow early mobility and provide better fixation, given the substantial bone loss for proximal support and is typically reserved for the elderly or sedentary patient.
Parvizi et al. review the use of a proximal femoral replacement (megaprosthesis) in revision hip surgery. They report that with the increased use of cortical strut grafts to augment host bone, the indications for the use of megaprostheses have narrowed. They conclude that currently, the use of megaprostheses is reserved for elderly or sedentary patients with massive proximal femoral bone loss that cannot be reconstructed by other reconstructive procedures.
Brown et al. tests the inter-observer and intra-observer reliability of this Paprosky classification of femoral bone loss. They report an inter-observer reliability of 0.61, indicating substantial agreement between surgeons. They also show a high intra-observer reliability, indicating substantial to almost perfect agreement. They conclude that there is substantial agreement among experienced arthroplasty surgeons when using the Paprosky Classification to characterize femoral bone loss.
Figure A is the AP radiograph of the right hip which demonstrates a Paprosky type IV femoral deficiency. Illustration A is the Paprosky classification of proximal femoral bone loss.
Incorrect Answers:
Answer 1: A loose stem with extensive metadiaphyseal proximal bone loss with non-supporting diaphyseal cortical bone (<4cm of cortical bone) is an indication for a proximal femoral replacement
Answer 3: A loose stem with extensive metadiaphyseal proximal bone loss with non-supporting diaphyseal cortical bone is an indication for proximal femoral replacement
Answer 4: A proximal femoral nonunion which has failed multiple attempts at obtaining osteosynthesis favors use of a proximal femoral replacement Answer 5: A Paprosky IV femoral bone loss involves extensive metadiaphyseal bone loss and a nonsupportive diaphysis
OrthoCash 2020
-
-
A healthy, active, 65-year-old male underwent a total knee arthroplasty 1 year ago. He presents to the emergency room after a ground-level fall earlier in the day. On exam, the patient is unable to perform a straight leg raise. Figure A is his current lateral radiograph. What is the most appropriate treatment for this patient?
-
Immediate active and passive range of motion in a hinged brace
-
Immobilization for 2 weeks followed by aggressive physical therapy
-
Cylinder cast for 6 weeks
-
Open Reduction Internal Fixation
-
Partial patellectomy
Corrent answer: 4
This patient has a displaced patella fracture with a disrupted extensor mechanism; therefore, (4) open reduction internal fixation is the correct answer.
Risk factors for patella fracture following total knee arthroplasty include trauma, the performance of a lateral release, damage to the blood supply, and excessive resection at the time of resurfacing. Indication for non-operative management includes both implant stability and extensor mechanism competency. If the implant is unstable or the extensor mechanism is disrupted a variety of surgical options exist including open reduction internal fixation (ORIF), component revision, partial or complete patellectomy or extensor mechanism allograft.
Konan et al. reviewed the management of periprosthetic total knee fractures. They report that non-operative treatment for periprosthetic patella fractures with an intact extensor mechanism leads to better outcomes when compared to surgery. Surgery is associated with high complication rates, including infection and nonunion. They conclude that regardless of the treatment employed, the goal should be early patient mobilization.
Kuyzk et al. reviewed the management of periprosthetic total knee fractures. They note that periprosthetic patella fractures are the least common type of fracture in this population. Their review concludes that host bone stock is one of the most important factors when determining the revision implant type.
Figure A demonstrates a displaced patella fracture in a patient with a previous total knee arthroplasty.
Incorrect answers
Answer 1: Immediate active and passive range of motion would not allow for the restoration of the extensor mechanism.
Answer 2: Two weeks of immobilization would not allow adequate time for fracture healing. Additionally, early range of motion before fracture healing could promote laxity of the retinacular tissues resulting in an extensor lag and weakness.
Answer 3: 6 weeks of cylinder casting would result in an unacceptable extensor lag.
Answer 5: Partial patellectomy is reserved for cases with severe comminution or pole fractures with small bony fragments. This patient is displaced at the midpoint of the patella.
OrthoCash 2020
-
-
A 91-year-old, minimally ambulatory male presents with acute on chronic progressive right thigh pain. Fifteen years ago, he underwent a right total hip replacement and he had been having progressive thigh start-up pain over the prior 5 months. He sustained a ground-level fall yesterday and he is now unable to bear any weight on the right leg. His current radiograph is shown in Figure A. His labs, including CBC, ESR, and CRP are all within normal limits. Which of the following represents the most appropriate next step in definitive management?
-
Revision to a proximal femoral replacement
-
Open reduction and internal fixation with proximal femoral locking plate and cerclage cables
-
Revision to a cemented long femoral stem
-
Open reduction and internal fixation with iliac crest bone grafting
-
Revision to a cementless long porous-coated femoral stem Corrent answer: 1
The patient has a Vancouver B3 periprosthetic femur fracture and requires revision. Given the extensive amount of bone loss in the proximal segment as well as a loose stem, a proximal femoral replacement would be an appropriate treatment
Post-operative periprosthetic femur fractures are classified based on the Vancouver classification which classifies fractures based on the location of the fracture, implant stability, and bone loss. Vancouver B3 periprosthetic femur fractures are defined as fractures at, around, or just below the stem with significant proximal bone loss/osteopenia or comminution which is unable to allow for reconstruction. In the setting of these injuries, treatment would consist of endoprosthetic proximal femur replacement or replacement with a large proximal femur allograft.
Della Valle et al. discuss the challenges associated with pre-operative planning for femoral revision total hip arthroplasty. The authors provide a classification of femoral bone loss that guides the surgeon in selecting an appropriate method of reconstruction. They conclude that appropriate pre-operative planning is required for the management of femoral implant revision in the setting of fracture, osteolysis, and instability.
Brown et al. reviewed the indications for revision THA to include instability, aseptic loosening, osteolysis, infection, periprosthetic fracture, component malposition, and catastrophic implant failure. They note that femoral component revision presents a complex challenge to the arthroplasty surgeon because of modern implant design as well as bone loss in the proximal femur. They conclude that knowledge of various reconstructive options and the indications for each is necessary to achieve a successful outcome.
Figure A is the radiograph of a periprosthetic femur fracture with extensive proximal femoral bone loss consistent with a Vancouver B3 periprosthetic femur fracture. Illustrations A and B are the radiographs revealing a revision total hip arthroplasty with proximal femur endoprosthetic reconstruction.
Incorrect Answers:
Answer 2, 3, 4 and 5: The patient has sustained a periprosthetic femur fracture and extensive osteopenia with bone loss and a loose stem. He requires revision with a proximal femoral replacement or a proximal femoral allograft
OrthoCash 2020
-
-
A 64-year-old male is 6 months out from left total knee arthroplasty. He has had at least two months of pain and swelling to the operative joint. In your initial workup, he is found to have a well-healed surgical incision, a serum CRP of 13mg/L and an ESR of 19mm/h. You perform arthrocentesis, which results in a negative alpha-defensin, synovial WBC of 1000 cells/µL, synovial PMNs of 90%, and synovial CRP of 4mg/L. What is the next best step in management?
-
Corticosteroid injection
-
Proceed to OR for histologic examination
-
Proceed with two stage revision
-
Proceed with single stage polyethylene exchange with irrigation and debridement
-
6 weeks of IV antibiotics
Corrent answer: 2
The patient has a score of 4 (2 points for elevated serum CRP, 2 points for elevated synovial PMN %) according to Parvizi et al's "The 2018 Definition of Periprosthetic Hip and Knee Infection: An Evidence-Based and Validated Criteria". This results in an ‘inconclusive’ determination of whether the patient has a periprosthetic joint infection. The best next step is to proceed to the OR in order to obtain tissue for histologic examination, cultures, and to determine whether purulence is present.
Periprosthetic joint infections can be devastating. Chronic PJI of the hip and knee is typically treated with two-stage revision arthroplasty. The first stage involves removal of the orthopaedic implants, placement of an antibiotic spacer, and at least 6 weeks of intravenous antibiotics. Once there is evidence that the infection has cleared (i.e. serum and synovial analysis), the second stage involves removal of the antibiotic spacer and placement of revision components.
Parvizi et al. updated their definition of PJI in 2018. They kept the major criteria for a chronic periprosthetic joint infection the same (an infection is indicated when 2 positive cultures of the same organism are isolated, or a sinus tract is present), but made several changes to the minor criteria.
Namely, they assign a point criteria in which a score of ≥6 is reliably infected, 2-5 is inconclusive, and 0-1 is not infected. In the case of infection, their recommendation is to proceed with a two-stage revision. If an inconclusive score is met, the recommendation is to proceed to the OR for histologic examination, cultures, and to determine whether purulence is present. The update to the MSIS criteria provides a sensitivity of 97.7% and a specificity of 99.5%.
Parvizi et al. reviewed 54 consecutive THA two-stage revisions. In their review, they determined MRSA to be the most common causative organism: MRSA 27.7%, S.epidermidis 18.4%, followed by MSSA 14.8%. They followed patients for a mean of 32 months, during which 26% had a recurrent infection and 8% developed mechanical failure (loose acetabular or femoral components).
Della Valle et al. made 15 CPGs (clinical practice guidelines) in JAAOS 2010. Among their recommendations: 1) they recommended utilizing serum CRP and ESR in assessing for PJI, 2) they recommended aspiration if serum CRP/ESR are abnormal, 3) if the initial aspiration has unexpected results they recommend repeat aspiration, and 4) 2 week antibiotic holiday before attaining cultures.
Illustration A shows the scoring criteria for the 2018 PJI update.
Incorrect Answers:
Answer 1: It would be inappropriate to perform a corticosteroid injection when you have some concern that a patient may have a periprosthetic joint infection.
Answer 3: Though this patient may have a periprosthetic joint infection, it would be wise to gather more data in order to maximize certainty that this patient does or does not have a periprosthetic joint infection.
Answer 4: If this were an acute periprosthetic joint infection a polyethylene exchange with irrigation and debridement would be appropriate.
Answer 5: It would be inappropriate to treat a patient who may have a periprosthetic joint infection with solely IV antibiotics.
OrthoCash 2020
-
-
A 66-year-old male is undergoing a total knee arthroplasty using a fixed bearing posterior stabilized component. During the intraoperative trialing of the components, it is noted that the flexion gap is loose, and the extension gap is appropriate. Compared to a patient with appropriate flexion and extension gaps, this patient would be at an increased risk for which of the following?
-
Manipulation under anesthesia
-
Knee hyperextension
-
Posterior knee dislocation
-
Anterior knee dislocation
-
Patella fracture
Corrent answer: 3
A loose flexion gap in a posterior stabilized primary total knee arthroplasty can lead to an atraumatic posterior knee dislocation during knee flexion.
A posteriorly stabilized knee has a post built into the polyethylene bearing that articulates with the box of the femoral component in flexion to act as a cam mechanism. It is felt to occur when the patient is in moderate to deep flexion and tibia is able to translate posteriorly. The post becomes locked behind the cam and the patient is unable to extend the knee. The etiology is felt to be related to implant design, soft tissue laxity over time, and surgeon technique.
Clarke et al. review flexion instability as a mode of failure in knee replacements. They describe how this is usually due to a lack of adequate balance at the time of surgery. They also report that revision surgery is usually the only way to correct symptomatic flexion instability.
Lombardi et al. reviewed 15 cases of dislocation in three different implant designs given the statistically significant difference in the rate of dislocation. They found the only significant difference between the group of patients with dislocation was increased flexion. They state the majority of the dislocations occurred prior to a modification in the tibial polyethylene insert of the specific knee design they studied.
Illustration A shows an example of a posteriorly dislocated total knee arthroplasty following a "jumped post" dislocation.
Incorrect Answers:
Answer 1: An excessive flexion gap would be more likely to lead to instability rather than stiffness and subsequent manipulation
Answer 2: A loose extension gap would lead to knee hyperextension, this patient has an appropriate extension gap.
Answer 4: The more common direction of dislocation in this scenario is a posterior knee dislocation when the cam is able to jump the post.
Answer 5: The risk of patella fracture is increased with over resection of the patella bone leaving a thickness of patella <13mm.
OrthoCash 2020
-
-
A 78-year-old woman who has a history of an uncomplicated right total hip arthroplasty presents after a fall. Figure A is the radiograph obtained in the emergency department. What is the most appropriate treatment for the femoral component?
-
Retained femoral stem with open reduction internal fixation
-
Revision femoral stem to an uncemented long stem with strut allograft
-
Revision femoral stem to a cemented long stem with open reduction internal fixation
-
Revision femoral stem to an uncemented long stem with open reduction internal fixation
-
Proximal femoral replacement Corrent answer: 4
In this scenario, the stem is loose with good proximal bone stock. The best option would be revision femoral stem to an uncemented long stem with an open reduction of the fracture.
Operative treatment of periprosthetic fractures is directed by the location of the fracture, the stability of the implant, and remaining bone stock. In circumstances where the femoral implant has loosened or subsided revision to a long stem is recommended. Cemented fixation is less ideal in a fracture scenario given cement interference with fracture healing. After bypassing the fracture with a long stem the fracture is reduced and stabilized to provide an environment for healing and long term durability.
Ko et al. reviewed 14 patients who underwent revision to a Wagner stem
following a B2 periprosthetic fracture. They found all 12 patients who followed up went on to union with 10 achieving good or excellent outcomes. They concluded the Wagner revision stem is a satisfactory prosthesis for Vancouver B2 periprosthetic femur fractures.
Kwong et al. reviewed 143 patients who underwent revision total hip for a proximally compromised femur to a modular cementless femoral stem.
Roughly ~10% of these patients were revised for periprosthetic fracture. They found a 97.2% survival rate and an average Harris hip score of 92. They concluded that the modular cementless diaphyseal engaging revision stem allows for adequate revisions THA for the proximally compromised femur.
Figure A shows a loose femoral component with subsidence and a periprosthetic fracture. Illustration A shows an AP of the right hip following open reduction internal fixation as well as a revision to an uncemented long stem and revision of the acetabular components.
Incorrect Answers:
Answer 1: ORIF while retaining the loose implant would lead to continued pain and instability.
Answer 2: There is adequate proximal bone stock so there is no indication for a strut allograft.
Answer 3: Revision to a cemented stem is not indicated as cement can impede fracture healing.
Answer 5: Proximal femoral replacement would not be indicated in this patient given adequate bone stock for a revision stem and ORIF.
OrthoCash 2020
-
-
A 78-year-old female with end-stage arthritis of the left hip is schedule for a total hip arthroplasty. Her contralateral hip was replaced 4 years prior and a current radiograph is shown in Figure A. Which of the following would be the most effective at preventing the complication shown in the Figure?
-
Indomethacin treatment for 2 days postoperatively
-
Leaving 2 drains in place until at least 4 days post-operatively
-
Postoperative administration of ethylhydroxydiphosphonate
-
Preoperative administration of radiation therapy 1 week before surgery
-
Postoperative administration of radiation therapy 8 hours following the surgery
Corrent answer: 5
This patient has developed heterotopic ossification (HO) following a total hip arthroplasty. Radiation therapy administered within 24-48 hours postoperatively would best prevent this from forming.
Heterotopic ossification following arthroplasty is relatively rare, but is more common following total hip arthroplasty (THA) as compared to total knee arthroplasty (TKA). Symptoms can range from subtle pain to complete loss of motion. The trigger for HO formation is unknown but this process involves mesenchymal cell stimulation within the muscle and fascia to form osteoblast and osteoid formation. This process begins within 16 hours of the insulting event and continues for over 1 year, maturing into a ossified mass of lamellar bone.
Iorio and Healy discussed management of HO following arthroplasty. They note the most effective treatment at preventing HO is radiation therapy given 6 hours or sooner pre-operatively, or within 4 days post-operatively.
Indomethacin is an alternative to radiation, with variable regimens showing relatively equivalent results. They make a point to note that despite appropriate prophylaxis, 2-5% of individuals still develop HO.
Pelligrini et al. evaluated the outcomes of 2 groups undergoing THA who received either 800 cGy or fractionated 1000 cGy radiation therapy postoperatively. Equivalent outcomes were shown between the groups, as evidenced by 79% disease-free at 6 month follow-up. Of note, the implants used were all cemented which do not rely on bone ingrowth.
Pelligrini and Gregoritch then prospectively followed 2 groups undergoing THA that were randomized to receive either pre- or post-operative radiation therapy. At final 6 month followup, both groups showed similar disease-free rates (73% vs. 76%). This demonstrated that pre-operative radiation was as effective as post-operative, at a time when it's utility was unknown and unstudied.
Figure A shows a radiograph with HO formation nearly completely bridging the hip joint.
Incorrect Answers:
Answer 1: Indomethacin is effective in preventing HO formation but duration of treatment usually requires at least 10 days, and usually longer.
Answer 2: Drains have not been proven effective in preventing HO.
Answer 3: Postoperative administration of ethylhydroxydiphosphonate results in delay of mineralization of osteoid but HO formation is not decreased.
Answer 4: Radiation must be done either right before the procedure or immediately following.
OrthoCash 2020
-
-
A 62-year-old woman with a valgus knee as seen in Figures A and B who underwent a primary total knee arthroplasty with a tourniquet presents 5 hours postoperatively with severe pain in the extremity and inability to dorsiflex or plantarflex the ankle. Narcotic pain medication does not improve her symptoms. The knee is flexed and the bandage is loosened and she is re-examined one hour later. On examination, the patient is unable to dorsiflex or plantarflex the foot and the pulses are asymmetric. What is the next most appropriate step in management?
-
Serial neurologic examinations and EMG in 3 months
-
Pain service consultation for adductor canal block
-
Return to the OR for peroneal nerve exploration
-
Vascular surgery consultation
-
Thrombectomy
Corrent answer: 4
The patient's asymmetric pulses, pain, and loss of motor function are evidence of an acute vascular injury for which an immediate vascular surgery consultation should be obtained.
Acute vascular injury following TKA is a rare but devastating complication that can result in wound healing complications, permanent neurologic injury, and loss of limb. Preoperative knowledge and evaluation of patients at risk for vascular injury allows optimization and potential modifications of surgical techniques to limit the chances of a vascular injury. Early recognition of an injury and consultation with a vascular surgeon is paramount to limit the chances of loss of limb. Patients with pre-existing vascular disease may be better served to have a TKA without the use of a tourniquet. If an injury occurs aggressive revascularization may be indicated for limb salvage.
Calligaro et al. report on their experience with vascular injuries following total hip and total knee arthroplasty in 32 patients, 24 TKA and 8 THA, for a rate of 0.13%. They found 44% of these vascular injuries were noted after the day of surgery and late-diagnosed injuries tended to have a higher rate of fasciotomies and foot drop. They also found thrombectomy alone was only successful in 28% of patients. They concluded that arterial injury after TKA and THA is rare, and aggressive revascularization is often needed for limb salvage.
Smith et al. review arterial injuries following total knee arthroplasty. They state preoperative risk factors of a history of vascular disease, intermittent claudication, ischemic ulcers, rest pain, asymmetric pulses, suspected popliteal aneurysm, radiographic evidence of vascular disease, or prior vascular surgery should alert the orthopedic surgeon of an increased risk for vascular complications following total knee arthroplasty. If performing a TKA on a patient with a history of vascular disease they recommend evaluation by a vascular surgeon and consideration of not using a tourniquet intraoperatively versus a tourniquet with an intravenous dose of heparin.
Ninomiya et al. reviewed the anatomy of the popliteal artery and when it could be injured during various portions of total knee arthroplasty. They state the popliteal artery is lateral to the midline at the tibial plateau in 95% of cadavers. To avoid vessel injury they recommend a preoperative vascular workup for high risk patients, careful placement of posterior retractors, and avoidance of hyperextension of the knee.
Figure A shows an AP and lateral of the knee respectively demonstrating a valgus knee with degenerative changes as well as posterior arterial calcifications.
Incorrect Answers:
Answer 1: Serial neurologic examinations would leave her with an untreated vascular injury.
Answer 2: Pain service consultation would not correct her underlying and limb-threatening vascular injury.
Answer 3: There is no evidence of an isolated peroneal nerve injury as the patient exhibits global motor dysfunction.
Answer 5: Thrombectomy for acute vascular injuries should be decided on by a vascular surgeon after consultation.
OrthoCash 2020
-
-
A 70-year-old female patient on chronic steroids for severe lupus presents with worsening bilateral hip pain over the last several years. She has been on chronic corticosteroids for p-ANCA vasculitis. The current radiograph is shown in figure A. Which treatment will have the most reliable pain relief and return of function in this patient?
-
Vascularized fibular graft
-
Proximal femoral osteotomy
-
Core decompression
-
Total hip arthroplasty
-
Hip resurfacing arthroplasty Corrent answer: 4
Total hip arthroplasty would provide the most reliable pain relief and return of function in this patient with bilateral femoral head avascular necrosis with collapse.
Hip avascular necrosis, also known as osteonecrosis, leads to progressively worse hip pain and femoral head collapse. Idiopathic avascular necrosis is most common, which is the result of intravascular coagulation. In 80% of
cases, it is bilateral. Core decompression, vascularized fibular autograft, and rotational osteotomies are treatment options for pre-collapse osteonecrosis. Once Total hip arthroplasty or hip resurfacing arthroplasty are treatment options for lesions with femoral head collapse. Hip resurfacing is reserved for young male patients with good femoral bone stock. Total hip arthroplasty is a better option for older, female patients and those with chronic steroid use as there is poorer bone quality.
Jawad et al. reviewed the Ficat classification system for hip osteonecrosis. They described stage 0 as preclinical disease that is suspected when the contralateral hip is affected, stage 1 as preradiographic disease with groin pain, stage 2 as increased femoral head density or cystic lesions with or without a crescent sign, stage 3 occurs when there is the loss of the femoral head contour and stage 4 with complete collapse of the femoral head with associated osteoarthritis. The authors concluded that the Ficat classification system is the most widely used, but has limitations with prognostication of outcomes and reliability among researchers.
Zalavras and Lieberman reviewed the diagnosis and management of hip osteonecrosis. They identified risk factors for developing hip osteonecrosis including trauma, corticosteroid use, excessive alcohol consumption, coagulation disorders, hemoglobinopathies, dysbaric phenomena, autoimmune diseases, storage diseases, smoking, and hyperlipidemia.
Figure A is the AP pelvis radiograph demonstrating bilateral hip avascular necrosis with the collapse of the femoral head. Illustration A is a diagram of a vascularized fibular autograft used for hip avascular necrosis. Vascularized fibular autograft involves the harvesting of a portion of the peroneal artery pedicle. The femoral neck and head are reamed to accommodate the graft, which provides structural support to the subchondral bone. This is fixed with a K-wire or screw. The graft is anastomosed with the lateral femoral circumflex artery. Illustration B is a table of the Ficat classification system.
Incorrect Answers:
Answer 1: Vascularized bone grafting provides mechanical support of the subchondral bone and enhances the revascularization of the femoral head. However, this procedure is much less predictable in patients > age 40.
Answer 2: Proximal femoral osteotomies are aimed at preventing femoral head collapse by transferring the necrotic bone from the weight-bearing region. This patient already has femoral head collapse, which excludes an osteotomy as a treatment option.
Answer 3: Core decompression is reserved for early-stage osteonecrosis without femoral head collapse by decreased intraosseous pressure and
increasing vascular flow. It is not effective once femoral head collapse has occurred. It can be augmented with bone morphogenic proteins or bone marrow aspirate.
Answer 5: Resurfacing arthroplasty preserves proximal femoral bone stock and does not affect later conversion to total hip arthroplasty. However, this is typically reserved for young male patients with good bone stock. Patients that are obese, female, femoral head/neck cysts are at higher risk for a subsequent femoral neck fracture. Chronic steroid use results in poor bone quality that excludes resurfacing arthroplasty as a treatment option.
OrthoCash 2020
-
-
Medial knee osteoarthritis is associated with which biomechanical change?
-
Decreased knee flexion moment
-
Increased knee abduction moment
-
Increased knee adduction moment
-
Increased knee extension moment
-
None of the above Corrent answer: 3
An increased adductor moment during gait is associated with progression of medial knee osteoarthritis.
Altered joint loading during ambulation contributes to the onset and progression of knee osteoarthritis. The external knee adduction moment is considered a surrogate measure for the medial tibiofemoral contact force. An adduction moment is experienced by the knee when the ground reaction force passes medial to the center of the joint. This is seen when the knee has a varus mechanical alignment. An abnormally large peak adduction moment has been linked to increased pain and rate of disease progression. Strategies to decrease the knee adduction moment have been developed such as offloader braces.
Chehab et al. performed a biomechanical, clinical and MRI study on the progression of knee osteoarthritis. They found that the knee adduction and flexion moments were associated with decreases in medial femoral and tibial cartilage thickness, with the knee adduction moment being most associated with diminished medial femoral cartilage thickness and the knee flexion moment being most associated with diminished medial tibial cartilage thickness. They concluded that both knee adduction moment and knee flexion moment increase with medial arthritis and varus alignment, and must be taken into consideration when designing interventions that address knee osteoarthritis progression.
Manal et al. performed a biomechanical study in which they determined knee compartment loading utilizing video-based motion capture and electromyographic (EMG) recordings. They determined that peak adduction moment was the leading predictor of peak medial loading, and that peak knee flexor moment was also a significant predictor of peak medial joint loading.
They concluded that the combined use of peak knee adductor and flexor moments provides a more accurate estimate of peak medial joint loading than the peak adduction moment alone.
Illustration A is a diagram that shows how a knee adduction moment is produced by a varus mechanical alignment.
Incorrect Answers:
Answer 1 and 4: These are two ways of stating the same thing. An increased knee flexion moment is associated with increased medial compartment loading.
Answer 2: An increased abduction moment is not associated with increased medial compartment loading.
Answer 5: This is incorrect. Both an increased adduction moment and an increased flexion moment are associated with increased medial compartment loading.
OrthoCash 2020
-
-
With regard to a mobile-bearing unicompartmental knee arthroplasty (UKA), which of the following is the most common cause of late (>10 years) failure?
-
Aseptic loosening
-
Progression of osteoarthritis
-
Unexplained pain
-
Instability
-
Infection
Corrent answer: 2
The most common cause of late failure and revision of unicompartmental knee arthroplasty (UKA) is the progression of osteoarthritis.
Late failure (>10 years) of UKA is most commonly caused by progression of arthritis. Other causes of failure include aseptic loosening, instability, infection, unexplained pain, and polyethylene wear. Interestingly, while TKA saw improved survival with improved polyethylene from the 1990s to the 2000s, UKA did not see a large change in survivorship suggesting that progression of osteoarthritis may be an unmodifiable mode of failure of these implants.
Jennings et al. review medial UKA. They state early medial UKA failures (<5 years) were from aseptic loosening (25%) with the progression of osteoarthritis second at 20%. However midterm and late failures were more commonly due to the progression of osteoarthritis (38 to 40%) with aseptic loosening (29%) and polyethylene wear (10%) as the next most common modes of failure.
Borus et al. reviewed UKA with regard to the evolution of the procedure. They found that at 10 years, one registry study found the progression of arthritis (51%) to be the most common cause of failure. They also cite a study that showed that mobile-bearing UKA tended to fail more from the progression of arthritis while fixed-bearing UKA failed due to tibial component failure.
Incorrect Answers:
Answer 1: Aseptic loosening is a major cause of failed UKA however it's more common in the early period (<5 years).
Answer 3: Unexplained pain can occur and cause the failure of UKA, but it does not occur as often as the progression of arthritis.
Answer 4: Instability can occur and cause the failure of UKA, but it does not occur as often as the progression of arthritis.
Answer 5: Infection can occur and cause failure of UKA, but it does not occur as often as the progression of arthritis.
OrthoCash 2020
-
-
Placement of an acetabular cup with a high inclination angle decreases the stability benefit of which of the following?
-
Larger femoral head size
-
Smaller femoral head size
-
Acetabular cup medialization
-
Increased femoral offset
-
Femoral stem anteversion Corrent answer: 1
Placement of an acetabular cup with a high inclination (abduction, theta) angle decreases the benefit of a larger head size by allowing dislocation with minimal translation.
Dislocations following THA occurs in approximately 1-3% of cases with an increased risk following revision surgery. Increased femoral head size improves stability by increasing jump-distance. In addition, an increased head-neck ratio allows a greater arc of motion prior to neck-socket impingement.
However, the increased stability provided by larger head sizes can be negated by other factors, including poor component positioning, poor soft tissue tensioning, and abductor deficiency. The use of a larger femoral head will not compensate for instability caused by a vertically positioned cup or abductor deficiency.
Burroughs et al. performed an in-vitro study evaluating the range of motion and stability in THA with 28, 32, 38, and 44-mm femoral head sizes. They found femoral heads >32-mm provide greater ROM and decreased component impingement. The authors conclude that large femoral heads offer potential in providing greater hip ROM and joint stability.
Kung et al. studied the effect of femoral head size and abductor integrity on dislocation rates in 230 patients who underwent revision THA. Four groups were identified: 1) intact abductor mechanism and 28-mm head, 2) absent abductor mechanism and 28-mm head, 3) intact abductor mechanism and 36-mm head, and 4) absent abductor mechanism and 36-mm head. The dislocation rate was higher with a 28-mm head compared to a 36-mm head when abductors were intact (groups 1 & 3). Dislocation rates were also higher if the abductors were absent, regardless of head size. The authors conclude the use of a large-diameter head does not reduce the rate of dislocation if the abductor mechanism is absent.
Illustration A is a low AP pelvis demonstrating an inclination (abduction, theta) angle of 63º.
Incorrect Answers
Answer 2: Smaller femoral heads have decreased head-neck ratios and therefore increased risk of dislocation.
Answer 3: Acetabular cup medialization decreases joint reactive force by
increasing the moment arm of the abductors, which is not biomechanically dependent on cup position.
Answer 4: Increased femoral offset increases soft-tissue tension, which is not decreased with a vertical cup.
Answer 5: The stability provided by femoral stem anteversion is not influenced by a vertical cup position.
OrthoCash 2020
-
-
A 65-year-old woman who underwent left total hip arthroplasty 10 years ago now reports groin pain over the past year. An immediate postoperative (left image) and current radiograph (right image) are shown in Figure A. Laboratory studies show an elevated ESR and CRP. Aspiration results from one day prior shows 500 WBC with 50% neutrophils. She is presently taking oral antibiotics for a upper respiratory tract infection. What is the next most appropriate step in management?
-
Triple phase bone scan
-
Repeat aspiration today sent for cell count, gram stain, and culture
-
Open biopsy and frozen sections
-
Revision arthroplaty of acetabular component
-
Repeat aspiration today sent alpha-defensin synovial fluid immunoassay Corrent answer: 5
Oral antibiotics can decrease the yield of aspiration. In this circumstance, the aspiration should be repeated after 2 weeks off antibiotics or a repeat aspiration can be sent for alpha-defensin synovial fluid immunoassay.
The diagnosis of periprosthetic joint infection remains a challenge with no single test with 100% accuracy. The highest accuracy can be achieved with a combination of tests and the knowledge of the pretest probability given the clinical history. Elevated inflammatory markers are an indication to aspirate the joint. Aspiration can yield a false-negative result if the patient is on antibiotics at the time of aspiration (often for UTI, URI, or cellulitis). A repeat aspiration after 2 weeks off antibiotics can yield more accurate information, or an alpha-defensin synovial fluid immunoassay can be performed as these results are not affected by antibiotics.
Della Valle et al. reviewed preoperative and intraoperative evaluation for periprosthetic infection. They state that patients should be off antibiotics for 2 weeks prior to aspiration as being on antibiotics can lead to false-negative results. They state there is no gold standard single test to diagnose a periprosthetic infection so a combination of tests will lead to the highest proportion of correct diagnoses.
Lachiewicz et al. reviewed 142 patients who underwent revision total hip arthroplasty and had a preoperative aspiration. They found no patients were infected if their implant had been in for >5 years and they had a normal ESR. They conclude that all patients with a painful total hip should be aspirated if
<5 years have elapsed from the index surgery or they have an elevated CRP.
Kelly et al. looked at synovial alpha-defensin in cases of an unclear diagnosis of PJI. They retrospectively reviewed 41 cases of possible PJI with prior aspiration and found in patients with recent antibiotic use alpha-defensin correctly diagnosed 83% of patients. They concluded alpha-defensin may be a useful data point in patients with and unclear diagnosis in cases of recent antibiotic use, equivocal laboratory findings, or suspected false-positive or false-negative cultures.
Figure A shows a left total hip with immediate post-op and current radiographs with interval loosening of the acetabular component.
Incorrect Answers:
Answer 1: Triple phase bone scan is not useful in determining septic versus aseptic loosening but rather if loosening is present. This implant is clearly loose on plain films.
Answer 2: Repeat aspiration would likely lead to similar aspiration results given that the patient is currently on antibiotics.
Answer 3: The patient has a suspected chronic periprosthetic joint infection. Preoperative optimization including adequate aspiration should be performed prior to revision surgery.
Answer 4: Revision arthroplasty may be indicated, however, a periprosthetic joint infection hasn't sufficiently been worked up prior to surgery and may change the plan for revision surgery.
OrthoCash 2020
-
-
An otherwise healthy 62-year-old woman presents 6 months postop from a TKA. For the past 6 weeks, she has noted pain and swelling with one week of drainage from the knee as seen in Figure A. Inflammatory markers and aspiration cell counts are elevated. Which of the following treatments give her the highest chance of eradication of infection?
-
Chronic lifelong suppressive antibiotic therapy
-
Urgent debridement, antibiotics, and implant retention (DAIR)
-
Urgent irrigation and debridement with modular component exchange
-
1-stage revision of all components
-
2-stage revision of all components Corrent answer: 5
The patient has a confirmed chronic postoperative periprosthetic joint infection (PJI) based on positive aspiration and a draining sinus. A 2-stage revision would give her the best chance of successful eradication.
Chronic PJI are difficult to eradicate without the removal of components due to biofilm on the prosthesis surface. A 2-stage revision with the removal of all components and placement of an antibiotic-eluting spacer and 6 weeks of culture directed IV antibiotics is the gold standard for chronic infections.
Parvizi et al. reviewed the literature on diagnostic strategies for assessing PJI and compared these data to a single large-volume institution's findings. Joint fluid aspirates with high cell counts and high percentage-neutrophils as well as positive FDG-PET scans have high sensitivities for diagnosing periprosthetic infections. While properly performed aspirates and intraoperative cultures
have near-perfect specificities (0.97, 1.0 respectively), they found a 10-14% false-negative rate. They go on to describe their institutional diagnostic protocol defining numerical cutoffs based on predictive value thresholds.
Koyonos et al. performed a single-institution retrospective review of irrigation and debridement alone for various time intervals from index surgery (acute, acute delayed, and chronic). They found irrigation and debridement was an ineffective way to definitively treat PJI for acute post-op infections (<4 wks from surgery 69% failure), acute delayed infections (>4 wks from surgery, 56% failure), and chronic infections (months after index surgery, 72% failure). They concluded that I&D should be reserved for acute onset symptoms with a non-staphylococcal infection in an optimized host with a previously normal total joint arthroplasty.
Figure A shows a draining sinus tract over a standard midline TKA incision. Incorrect Answers:
Answer 1: Lifelong antibiotic suppression would be less likely to lead to
successful infection eradication compared to 2-stage revision.
Answers 2 and 3: Debridement and modular component exchange would be less likely to lead to successful eradication compared to 2-stage revision.
Answer 4: 1-stage revision is an area of active research with promising data thus far, but the gold standard for chronic PJI management remains a 2-stage revision with our current body of evidence.
OrthoCash 2020
-
-
A 62-year-old female is referred to you by your partner for continued groin pain after undergoing an uneventful total hip arthroplasty utilizing the direct anterior approach. On exam, her pain is reproduced with resisted hip flexion. Workup for infection and loosening were negative. Radiographs and CT show well-placed implants. An ultrasound-guided lidocaine injection eliminated her pain completely for one day but returned. When arthroscopically addressing the pathologic structure, in what position should the hip be placed to avoid damaging the nearest major neurovascular structure?
-
Internal rotation to avoid the sciatic nerve
-
Internal rotation to avoid the ascending medial femoral circumflex artery
-
Internal rotation to avoid the femoral neuromuscular bundle
-
External rotation to avoid the ascending medial femoral circumflex artery
-
External rotation to avoid the femoral neurovascular bundle
Corrent answer: 5
This patient has iliopsoas tendinitis following total hip arthroplasty (THA). The leg should be externally rotated when performing extra-capsular tenotomy to protect the femoral neurovascular bundle.
Hip arthroscopy was a previously daunting procedure given the deep location of the hip joint, limitations of available instrumentation, and close proximity of the peri-articular anatomy. Significant advances in these areas have allowed for safer and more reproducible results in treating common hip pathology.
Iliopsoas tenotomy is now commonly performed but relies heavily on a thorough understanding of the surrounding anatomy. The two most common iliopsoas tenotomy techniques are the trans-capsular and extra-capsular releases. Trans-capsular release involves making a capsulotomy between the anterior labrum and zona orbicularis to access the iliopsoas tendon (Illustration A). The extra-capsular tenotomy is more commonly done following total hip arthroplasty and involves accessing the iliopsoas at the level of the lesser trochanter (Illustration C). Both techniques require slight flexion of the hip and external rotation to bring the tendon towards the surgeon and away from the femoral neurovascular structures.
Robertson and Kelly performed a cadaveric study demonstrating which neurovascular structures are at risk with common arthroscopic portals. The traditional anterior portal (AP) was an average of 15mm from the branches of the lateral femoral cutaneous nerve. They concluded that by shifting the anterior portal further lateral to a longitudinal line drawn from the ASIS, the LFCN is less in danger.
Ilizaliturri et al. performed a prospective trial following two cohorts undergoing either the extra-capsular or trans-capsular iliopsoas tenotomy. They found both cohorts had significant improvements in both groups without significant differences between them. This shows arthroscopic/endoscopic iliopsoas tenotomy to be a safe and reliable procedure.
Illustration A shows the trans-capsular iliopsoas tenotomy. Illustration B is a panel intra-operative photos of this release, showing the subsequent release of the bright white psoas tendon (PT) to reveal the iliac muscle fibers (ZO= zone orbicularis, AHC= anterior hip capsule). Illustration C is an axial MRI at the level of the central compartment, with the femoral neurovascular bundle (arrow) and psoas labelled.
Illustrations D and E demonstrate the extra-capsular iliopsoas tenotomy. Again, note the bright appearance of the psoas tendon before release.
Illustration F is an axial MRI at the level of the lesser trochanter with the femoral neurovascular bundle (arrow) and psoas tendon labelled.
Incorrect Answers:
Answers 1 and 2- Internal rotation would be beneficial when exploring the deep gluteal space. Both the sciatic nerve and MFCA are posterior structures at risk when in this area.
Answer 3- Internal rotation would draw the iliopsoas and lesser trochanter away from the surgeon.
Answer 4- The LFCA is at risk when instrumenting through the DALA portal, the MFCA is a posterior structure.
OrthoCash 2020
-
-
A 65-year-old male presents to your clinic with right hip pain. He underwent right metal-on-metal total hip arthroplasty (THA) 20 years prior and had been doing well until 5 years ago. He notes groin pain with ambulation and thigh pain when getting up from a seated position. His radiographs are shown in Figure A. MRI studies with metal artifact reduction sequence reveal a pseudotumor noted in the periprosthetic soft tissues. Which of the following is true with respect to this patient's clinical picture?
-
The majority of patients with metal-on-metal THA have a pseudotumor
-
The presence of a pseudotumor is associated with increased implant linear wear rate
-
The pseudotumors are composed of both macrophages and lymphocytes, with macrophages being predominant
-
The presence of a pseudotumor is associated with relatively high ratio of serum cobalt to chromium
-
Observation is recommended given the absence of loosening on radiographs Corrent answer: 2
The patient has a metal-on-metal (MoM) THA with a pseudotumor present on MRI studies. The presence of a pseudotumor is associated with increased linear wear of both the acetabulum and the femoral components.
A pseudotumor is a mass-forming tissue reaction caused by metal-on-metal wear most notably seen in MoM total hip arthroplasty. These lesions are noted in 10-15% of patients with MoM THAs. The etiology is unclear but is thought to be associated with local high wear debris as well an increased hypersensitivity to metal wear. The lesions are lymphocyte-predominant although macrophages are also present throughout. Treatment of patients with hip pain and pseudotumor noted on MRI studies is generally revision to a ceramic-on-polyethylene THA.
Kwon et al. compared the in vivo wear rate of MoM THAs revised due to a pseudotumor compared to those MoM THAs that were revised for other reasons. The authors noted that the pseudotumor group (n = 8) had median linear wear of the femoral component of 8.1 microns/year and the non-pseudotumor group (n =22) had a linear wear rate of 1.79 microns/year. A similar discrepancy was noted for acetabulum wear with 7.36 microns/year in the pseudotumor group compared to 1.28 microns/year in the non-pseudotumor group. The authors conclude that the presence of a pseudotumor is associated with increased wear at the metal-on-metal articulation.
Daniel et al. performed a review of pseudotumors associated with MoM THA. They reviewed the potential causes to include foreign-body reaction, hypersensitivity and wear debris. They discussed that patients with pseudotumors associated with a MoM THA should undergo prompt revision to a nonmetal-on-metal bearing THA since prolonged delay can lead to soft tissue envelope compromise, recurrent dislocations, nerve palsies, and femoral artery stenosis. Furthermore, they also noted that hip function scores for patients who had revision for pseudotumors were significantly worse than those for patients who had MoM THA revision for other reasons.
Figure A is an AP pelvis showing the characteristic appearance of an MoM THA with a large metallic femoral head.
Incorrect Answers:
Answer 1: Studies show that only around 10-15% of patients with a MoM THA will have a pseudotumor present on MRI studies.
Answer 3: Pseudotumors are composed of both macrophages and lymphocytes, with lymphocytes being predominant.
Answer 4: Pseudotumors are not typically associated with a high cobalt to chromium ratio (this can be seen in trunnionosis).
Answer 5: In the presence of a MoM THA and pseudotumor, revision THA should be entertained as progressive soft tissue damage and more difficult reconstructions can result with delayed care.
OrthoCash 2020
-
-
A 66-year-old male undergoes the procedure shown in figures A and B. After 4 years, he develops progressive pain and limitations in his daily function that is refractory to conservative measures. He is indicated for conversion to a total knee replacement with almost complete relief of his symptoms postoperatively. What preoperative factor likely led to the subsequent failure?